Common Law Admission Test-2018 Held on 13-5-2018 Question Paper With Answer Key

Common Law Admission Test-2018 Held on 13-5-2018
Common Law Admission Test-2018 Held on 13-5-2018 Question Paper With Answer Key

Common Law Admission Test-2018 Held on 13-5-2018

111. Directions- (Q. 111-112) Read the information below and answer the Questions.

A neurosurgeon, an orthopeadist, a plastic surgeon and an ophthalmologist visited the patient after a surgery.

The time of each visit is mentioned below :

Neurosurgeon at 7 : 45

Orthopedist 8 : 30

Plastic surgeon 9 : 15

Ophthalmologist  9 : 50

Each time mentioned above may be either A.M. or P.M.

If the ophthalmologist generally goes for his rounds before the surgeons, but not unless any other doctor does, and the neurosurgeon is not the last to go for his rounds, what is the correct sequence of their visit?

(A) Orthopaedist, Neurosurgeon, Plastic surgeon, Ophthalmologist

(B) Neurosurgeon, Orthopaedist, Plastic surgeon, Ophthalmologist

(C) Orthopaedist, Ophthalmologist, Neurosurgeon, Plastic surgeon

(D) Ophthalmologist, Orthopaedist, Neurosurgeon, Plastic surgeon

Answer: (C)

112. Who were the doctors to visit the patient in the evening?

(A) Neurosurgeon and Plastic surgeon

(B) Plastic surgeon Ophthalmologist and Neurosurgeon,

(C) Orthopaedist and Neurosurgeon

(D) Ophthalmologist and Orthopaedist and Plastic surgeon

Answer: (A)

113. Samantha is your father’s mother’s grandson’s daughter.

(A) niece

(B) sister

(C) uncle

(D) brother

Answer: (A)

114. Look at this series : 7, 10, 8, 11, 9, 12, …

What number should come next?

(A) 7

(B) 10

(C) 12

(D) 13

Answer: (B)

115. In an interview for selection of two District Judges, four candidates P, Q, R, and S were short listed. All of them had 10 years standing as Lawyers. Who among them are most suitable?

(I) P-Very academic not much interested in litigation practice, very honest and impatient.

(II) Q-A Patient listener, average in academics, thorough

(III) R-Jobial, Patient, good academic records and actively involved in politics.

(IV) S-Shrewd, Quick tempered, punctual and Published several Legal articles

(A) P and Q

(B) Q and S

(C) R and S

(D) P and S

Answer: (B)

116. Find the odd one :

(A) Bat

(B) Vulture

(C) Eagle

(D) Owl

Answer: (A)

Directions- (Q. 117-118) Read the information below and answer the questions:

    On the farewell day five best friends, namely, Vishal, Anirudh, Gaurav Mansi and Sapna, clicked their photo, facing the photographer who is on the west.

They were standing as per the following conditions:

(I) Sapna stands to the right of Mansi and Gaurav

(II) Gaurav stands to the left of Vishal and Sapna

(III) Mansi stands between Vishal Anirudh

(IV) There are two people between Anirudh and Gaurav

117. Who will be standing at the 2nd position from the right in the photograph?

(A) Vishal

(B) Mansi

(C) Gaurav

(D) Anirudh

Answer: (A)

118. Who stands to the left of Anirudh?

(A) Sapna

(B) Gaurav

(C) Mansi

(D) Vishal

Answer: (C)

119. Find the equivalent for ‘?’

Sphygmomanometer : Blood Pressure : : Pyrometer : ?

(A) Temperature

(B) Blood Flow

(C) Urine Flow

(D) Atmospheric Pressure

Answer: (A)

120. From morning 11 O’ clock to Evening 7 O’ clock, how many times the hour hand and minutes hand will overlap each other?

(A) 9

(B) 8

(C) 7

(D) 6

Answer: (C)

121. Find the odd one out :

(A) Police station

(B) Railway station

(C) Supermarket

(D) Airport

Answer: (A)

122. Direction- Read the following carefully and answer the question given below :

Although women now constitute 30 per cent of the global industrial force, legal hurdles and traditional barriers still hinder the vast majority of women in their efforts to achieve parity with men.

The paragraph best support the statement :

(A) The Author claims that Feminism rules

(B) Equality of men and women is still a myth

(C) Women should surpass men in all the fields

(D) The Author claims that women have to take more efforts

Answer: (B)

123. If TEACHER is coded as CAHEETR and STUDENT is coded as DUETNST, then what is the code for SUCCESS?

(A) CECSUSS

(B) CCEUSSS

(C) SCUSCES

(D) UCSECSS

Answer: (B)

Directions- (Q. 124-129) Read the following and answer the questions.

   Chetan invited five of his friends, namely Ritwik, Avantika, Darshan, Parul and Shweta to his birthday party. Each of the five friends gifted Chetan a different article – a jacket, a watch, a shirt, a video game and a headphone. As a return gift, Chetan gifted each of them a different article-goggles, a bag, a pen, a mobile case and a diary. The following is additional information about the gifts gives by the friends and the return gifts they received.

(I) Avantika gifted the shirt but did not receive a bag in return.

(II) Shweta did not gift the headphone but received the mobile case in return.

(III) Neither Ritwik nor Darshan gifted the jacket but the person who did gift it, received the goggles in return.

(IV) The person who gifted the video games received a diary or a bag.

(V) Darshan did not gift a watch but received a pen in return.

(VI) Neither Shweta gifted a video game nor did Ritwik received a diary in return.

124. Who gifted the watch?

(A) Shweta

(B) Darshan

(C) Ritwik

(D) Parul

Answer: (A)

125. Who received goggles as a return gift?

(A) Avantika

(B) Ritwik

(C) Parul

(D) Darshan

Answer: (C)

126. Which among the following combination is correct?

(A) Avantika – Bag

(B) Ritwik – Video Game

(C) Ritwik – Diary

(D) Parul – Watch

Answer: (B)

127. Who gifted the head phones?

(A) Shweta

(B) Ritwik

(C) Darshan

(D) Parul

Answer: (C)

128. Which among the following combinations is correct?

(A) Avantika-Diary

(B) Headphones-Diary

(C) Ritwik-Goggles

(D) Watch-Bag

Answer: (A)

129. The one who received the bag in return, gifted which among the following items?

(A) Watch

(B) Video game

(C) Headphones

(D) Jackets

Answer: (B)

130. Directions- Examine the following expressions/activities/processes, and arrange them in the most logical sequence.

(1) Design               (2) Need

(3) Launching       (4) Research

(5) Testing              (6) Identify

(A) 4, 1, 6, 2, 3, 5

(B) 2, 6, 4, 1, 5, 3

(C) 3, 5, 4, 1, 6, 2

(D) 2, 4, 1, 5, 6, 3

Answer: (B)

131. The value of expression  is

(A) 2/15

(B) 2

(C)  

(D) 15

Answer: (D)

132. The fractions  are arranged in ascending order of magnitude as :

(A)   

(B)   

(C)   

(D)   

Answer: (B)

133. Let x be the greatest number of 4 digits, which when divided by 15, 20 and 28 leaves in each case the remainder 2. The sum of digits of x is:

(A) 19

(B) 21

(C) 23

(D) 25

Answer: (C)

134. A student was asked to simplify the expression :

 his answer was 1/65. What is the difference between his answer and the correct answer?

(A) 1/65

(B) 1/130

(C) 1/20

(D) 1/13

Answer: (B)

135. When 13511, 13903 and 14589 are divided by’ the greatest number ‘n’. The remainder in each case is ‘m’. the value of (n + m) is:

(A) 183

(B) 182

(C) 181

(D) 179

Answer: (A)

136. ABCD is a quadrilateral in which measures of angle D and angle C are 60 degree and 100 degree respectively. If the internal bisectors of angle A and angle B meet at P, then measure of angle APB is:

(A) 80°

(B) 90°

(C) 100°

(D) 110°

Answer: (A)

137. Suman saves 10% of her monthly salary. Now her expenditure increases by 30% and savings increases by 50%. The present increase in her salary is :

(A) 25%

(B) 30%

(C) 32%

(D) 40%

Answer: (C)

138. A sum amounts to Rs 9,680 2 years and to Rs 10,648 in 3 years respectively at compound interest. What will be the amount if the same sum is invested for  at the same rate of compound interest?

(A) Rs 9,025

(B) Rs 9,152

(C) Rs 9,215

(D) Rs 9,251

Answer: (B)

139. The marked price of an article is 20% more than the cost price. If the article is sold at a discount of 15% on its market price, then the gain per cent is :

(A) 5

(B)  

(C)  

(D) 2

Answer: (D)

140. Reshma sells an article to Rekha at 37.5% profit, Rekha sells it to Madhu at  Again Madhu sells it to Mitu at 25% loss. If Mitu pays Rs 342 for the article, then what is the cost price of the article to Reshma?

(A) Rs 304

(B) Rs 266.50

(C) Rs 380

(D) Rs 384.75

Answer: (A)

141. The number of children a camp is x and their average weight is 20 kg. If 5 children each weighing 12 kg. join the camp or if 10 children each weighing 21 kg leave the camp, the average weight in both the cases remain the same. The value of x is :

(A) 18

(B) 16

(C) 15

(D) 14

Answer: (C)

142. In a 120 litre of solution of acid and water, acid is 75%. A person takes out 20 litres of this solution and added 16.2 litres of acid and 3.8 litres of water in the remaining solution. What is the percentage of water in the final solution?

(A) 22

(B) 24

(C) 25

(D) 28

Answer: (B)

143. Twelve men and 5 women can complete a work in 2 days where as 4 men and 3 women can complete the same work in 5 days. In how many days can 8 men complete the work?

(A) 3

(B) 4

(C) 5

(D) 6

Answer: (B)

144. a and b are inversely proportional to each other and are positive. If a increases by 100%, then b decreases by:

(A) 50%

(B) 75%

(C) 100%

(D) 200%

Answer: (A)

145. Four pipes A, B, C and D can fill a tank with water in 15, 20, 30 and 60 hours respectively. Pipe A is opened at m., B at 5 a.m., C at 6 a.m. and D at 7 a.m. When is the tank filled up completely?

(A) 9 : 30 a.m.

(B) 10 : 00 a.m.

(C) 10 : 30 a.m.

(D) 11 : 00 a.m.

Answer: (D)

146. If a person travels at a speed of 40 km/h, he will reach his destination on time. He covered half the journey in 2/3 of the time. At what speed (in km/h) should he travel to cover the remaining journey to reach the destination on time?

(A) 48

(B) 50

(C) 60

(D) 72

Answer: (C)

147. The parallel sides of a trapezium shaped field are 25 m and 10 m and non-parallel sides are 14 m and 13 m. What is the area (in m2) of the field?

(A) 204

(B) 196

(C) 156

(D) 144

Answer: (B)

148. x2 (a – b) + a2(b – x) + b2(x – a) is factored as :

(A) (a – b) (x – a) (x – b)

(B) (a – b) (x – a) (x + b)

(C) (b – a) (x + a) (x – b)

(D) (a + b) (x – a) (x + b)

Answer: (A)

149. Three fair dice are thrown simultaneously. What is the probability that the sum of numbers on their tops, is at leas 6?

(A) 5/108

(B) 1/24

(C) 103/108

(D) 17/18

Answer: (C)

150. Six dice are stacked on the floor as shown in the figure below. On each dice, the sum of numbers on opposite faces is 7, i.e., if 1 is written on one face, then 6 is written on the face opposite it and so on.

What is the maximum possible sum of numbers on the 21 visible faces?

(A) 88

(B) 89

(C) 96

(D) 147

Answer: (*)

151. Article 51-A on Fundamental Duties was inserted into the Constitution of India through the :

(A) 41st  Constitution (Amendment) Act, 1978

(B) 44th Constitution (Amendment) Act, 1978

(C) 40th Constitution (Amendment) Act, 1976

(D) 42nd Constitution (Amendment) Act, 1976

Answer: (D)

152. The highest law officer in India is the :

(A) Attorney General

(B) Advocate General

(C) Solicitor General

(D) Chief Justice of the Supreme Court of India

Answer: (A)

153. How many languages are there in the Eighth Schedule of the Constitution of India?

(A) 21

(B) 22

(C) 19

(D) 18

Answer: (A)

154. A Panchayat Samiti at the block level in India is only a/an :

(A) Administrative authority

(B) Coordinating and Supervisory authority

(C) Consultative Committee

(D) Advisory body

Answer: (B)

155. Which of the following is not a fundamental duty as per the provisions of the Constitution of India?

(A) To  uphold and protect the sovereignty, unity and integrity of India

(B) To join the defence of Forces of India

(C) To defend the country and render national service when called upon to do so

(D) To value and preserve the rich heritage of our composite culture

Answer: (B)

156. The Right to Education Act, 2009 (RTE) providers for free and compulsory education to-

(A) All illiterate children of India

(B) All citizens of India

(C) Children aged between 6and 14 years

(D) All children up to the age of 10 years

Answer: (C)

157. Direction- The question below consists of two statements, one labeled as ‘Assertion (A) and another as ‘Reason’ (R) Examine these two statements carefully and select the answer to these items from the codes given below :

Assertion (A) : It is the legal and constitutional duty of the State to provide legal aid to the poor.

Reason (R) : No one should be denied by reason of his poverty.

Codes :

(A) Both (A) and (R) are individually true and (R) is the correct explanation to (A)

(B) Both (A) and (R) are individually true but (R) is not the correct explanation (A)

(C) (A) is true but (R) is false

(D) (A) is false but (R) is true

Answer: (A)

158. Which was the law introduced by Sir William Bentinck to prohibit the practice of Sati?

(A) Bengal Sati Regulation, 1829

(B) U.P. Sati Prohibition Act, 1828

(C) Indian Sati Prohibition Act, 1827

(D) Rajasthan Sati (Prevention) Act, 1830

Answer: (A)

159. Legal Principle: Nothing an ‘offence’, if committed by a child below seven years of age.

Fact Situation : Adil, ages six years, is a student of class one. He placed his sharpened pencil on the bench with its pointed end up when his classmate Ajay stood up to answer a question from the teacher. Ajay gets hurt when he sits on the pencil and Adil and his friends have a good laugh. Ajay’s father, on seeing his son injured when he returns home, wants action against Adil. Which of the following statements is the most appropriate in relation to the legal principle stated above?

(A) Adil has committed an ‘offence’.

(B) Adil has not committed any ‘offence’.

(C) Childish pranks cannot be investigated by the police

(D) The class teacher must be arrested

Answer: (B)

160. Legal Principle : The doctrine of basic structure in Constitutional jurisprudence means that the Constitution of India has certain basic features that cannot be taken away through amendments by the Parliament. The power of judicial review is a part of the basic structure and it helps the constitutional Courts to determine whether an amendment is against the basic structure or not.

Fax Situation : Parliament proposes an amendment to limit the power of appeal against conviction for the offence of Sedition, to be exercised only by the Supreme Court of India.

Which of the following statements is the most appropriate in relation to the legal principle stated above?

(A) The proposed amendment is against doctrine of basic structure.

(B) The proposed amendment is unconnected to the doctrine of basic structure

(C) The proposed amendment can be reviewed and struck down by the Constitutional Courts.

(D) The proposed amendment is beyond the power of the Parliament

Answer: (C)

161. Who among the following was the first Chairman of the Constituent Assembly in India?

(A) Dr. Rajendra Prasad

(B) Dr. Sachchidananda Sinha

(C) Harendra Coomar Mookerjee

(D) Dr. B. R. Ambedkar

Answer: (B)

162. Legal Principle : It is an offence to obstruct a public servant in the due discharge of his duty. Right of private defence is available to protect one’s person and property.

Fact Situation : Sidhu comes to the rescue of his uncle who is sought to be taken into a car by some men. In the process, he causes injury to some of them. Later, it turns out that the men were police persons in plain clothes trying to enforce a warrant against his uncle.

Which of the following statements is the most appropriate in relation to the legal principle stated above ?

(A) Sidhu has committed the offence of obstructing a public servant in due discharge of his duty.

(B) Sidhu has not committed an offence since he did not know that the men were from the police

(C) Sidhu’s uncle has resisted arrest and should be proceeded against

(D) Sidhu should not have tried to help his uncle without ascertained the fact

Answer: (B)

163. Legal Principle : An employer is liable for the Act of his servant performed during the course of employment

Fact Situation : While working as a driver for Verma. Alok sometimes used to earn some side income by carrying parcels for others in Verma’s car without his knowledge or permission. While going to pick Verma from the airport one day, Alok stopped to deliver a parcel he was carrying with him. While he was delivering the parcel, which unknown to him was one of contraband goods, the police arrested Alok.

Which of the following statements is the most appropriate in relation to the legal principle stated above ?

(A) Verma is liable for the act of Alok since he is Verma’s driver

(B) Verma is liable for the act of Alok since he had gone to pick Verma from the airport

(C) Verma is not liable for the act of Alok since Alok himself did not know that he was carrying contraband goods.

(D) Verma is not liable for the act of Alok since carrying the parcel was not in the course of his employment

Answer: (D)

164. Which law introduced the system of diarchy in India during the British reign?

(A) The Government of India Act, 1858

(B) The Government of India Act, 1909

(C) The Government of India Act, 1919

(D) The Government of India Act, 1935

Answer: (C)

165. Legal Principle : The Latin maxim nemo bis punitur pro eodem delictomeans that nobody can be punished twice for the same offence.

Fact Situation : Sajan, a petty thief, is caught and trashed thoroughly by the people before being handed over to the police. Sajan pleads before the Magistrate that since he was already thrashed by the people he should not be again punished by the State.

Which of the following statements is the most appropriate in relation to the legal principle stated above?

(A) Sajan is right since nobody should be punished for the same offence twice.

(B) Thrashing given by the people does not amount to legal punishment and so Sajan can be punished by the State.

(C) Giving a good thrashing to the thief is the best form of punishment to prevent future theft.

(D) The Magistrate should take into consideration the thrashing received by Sajan while fixing his punishment.

Answer: (B)

Common Law Admission Test-2018 Held on 13-5-2018 Question Paper With Answer Key

Common Law Admission Test Held on 13-5-2018
Common Law Admission Test-2018 Held on 13-5-2018 Question Paper With Answer Key

Common Law Admission Test 2018 Held on 13-5-2018

1. Directions- In the paragraph below there are five sentences. In each sentence there are pairs of highlighted words (A, B). Select the most appropriate words to form correct sentence. Then from the options given select the correct option.

Internationally high rise walls serve as street (A) paintings (B) canvasses. In Delhi and Gurgaon artists have (A) earmarked (B) earmark district and after civic authority (A) compliance, (B) grant transformed it into a throbbing art district. A walk through the (A) installment (B) Installations for public interaction is (A) instituted (B0 enivsaged. It is hoped that this wil discourage defiling city’s public places.

The correct sequence should be :

(A)  ABABB

(B) BABAB

(C)  BAABB

(D)  ABAB

Answer: (C)

2. Directions – In the paragraph below there are five sentences. In each sentence there are pairs of highlighted words. (A, B). Select the most appropriate words to form correct sentence. Then from the options given select correct option.

Pie charts are another extremely effectively (A) visual (B) graph presentation of data. They show what proportions make up a whole. Each segment should be (A) cleared (B) differentiated by shading cross-hatching or colour and should be (A) labeled (B) articulated horizontally. (A) Initially (B) Conventionally, the largest slice begins at the 12 o’clock position then slices are sequence clockwise in (A) descending (B) clarified positions.

(A)  ABABA

(B) BBABA

(C)  BABBA

(D)  BAABA

Answer: (A)

3. Directions : In the paragraph below there are five sentences. In each sentence there are pairs of highlighted words (A, B).

Select the most appropriate words to form correct sentences. Then from the options given select the correct option.

One of the predictable  difficulties you can expect to (A) encounter (B) counter when you go to a different country to study or work is language. But difficulties may also result from (A) culture (B)

Cultural differences, which are offen less (A) conscious (B) obvious at first and can be unexpected. A Dutch academic was one of the first persons to propose an (A) influential (B) dire theory of cultural (A) diversity (B) division.

(A)  ABABA

(B) ABBAA

(C)  AABBA

(D)  ABABB

Answer: (B)

4. Directions – Identify the incorrect sentence/ sentences:

(A)  It is important to monitor the groups.

(B) Check that the instructions are clear, and tell the students to begin.

(C)  Inefficient bus service are a hazard

(D)  Traffic the city has reached unsustainable levels

(A)  A

(B)  B

(C)  C

(D)  D

Answer: (C)

5. Directions – Identify the incorrect sentence/ sentences.

(A) The college has organized a science fare

(B) This is a to notify a leather wallet has been lost

(C) Shortly blood donation will begin a camp

(D) The agenda of the meeting is available on the subside

(A)  B

(B) C

(C)  D

(D)  A

Answer: (B)

6. Directions – Identify the incorrect sentence/ sentences.

(A) It is vastly raiding is significant

(B) Reading for pleasure is linked to parental support at an early age.

(C) We can read intensively and extensively

(D) Basic literacy among the prison population is insignificant.

(A)  C

(B) D

(C)  B

(D)  A

Answer: (D)

7. Directions- Fill up the blanks numbered (a) (b) (c) (d) (e) (f) in the passage given below with the most appropriate word from the options given for each blank.

The main objective of art and (a) living is to develop (b) sensibilities and skills of healthful living besides providing a (c) ground for love of a labour, (d) social attitudes and moral values so as to enable the child to be (e) to the ideas of others with humility and sincerity in thought, word and deed. Love for mankind and helping the needy would (f) at this stage and its culmination would be in terms of attainment of selfless service.

‘a’ is :

(A)  profitable

(B) promising

(C)  productive

(D)  praises

Answer: (C)

Directions- (Q. 8-12) Fill up the blanks numbered (a) (b) (C) (d) (e) (f) in the passage given below with the most appropriate word from the options given for each blank.

The main objective of art and (a) living is to develop (b) sensibilities and skills of healthful living besides providing  (c) ground for love of labour, (d) social attitudes and moral values so as to enable the child to be (e) to the ideas of others with humility and sincerity in thought, word and deed. Love for mankind and helping the needy would (f) at this stage and its culmination would be in terms of attainment of selfless services.

8. ‘b’ is :

(A)  nature

(B) healthy

(C)  aesthetic

(D)  genuine

Answer: (C)

9. ‘c’ is :

(A)  nurturing

(B) fruitful

(C)  attractive

(D)  interesting

Answer: (A)

10. ‘d’ is :

(A)  clear

(B) positive

(C)  growing

(D)  negative

Answer: (B)

11. ‘e’ is :

(A)  welcome

(B) clear

(C)  receptive

(D)  illegal

Answer: (C)

12. ‘f’ is :

(A)  disappear

(B) reveal

(C)  link

(D)  germinate

Answer: (D)

13. Direction- For alternative summaries are given in the text. Choose the option that best captures the essence of the text.

Develop Critical Thinking

The way we see the world and relate to others is intrinsically connected to our own set of values that govern the way we decide to live. However, the influence of fashion, consumerism, pop culture, broken homes, social unrest, and the media is all-pervasive. For any people, teachers and students alike, this influence goes unquestioned. Critical thinking, if successfully taught at this level, becomes the antidote for individual and social illiteracy. For the authors, critical thinking should constitute an indivisible part of the overall educational process. Facione (1995) comments : ‘Critical thinking lies at the root of civilization. It is a cornerstone in the journey humankind is taking from beastly savagery to global sensitivity’.

Supporting the development of these skills involves reflective teaching and learning, which is highly complex and which some students may find difficult, or interpret as weakness on the part of the teacher. But in the long run, with patience on the part of the teacher, it will develop students who can view old or new material, from a variety of sources, through new eyes, using their skills to define their own stance and express it, often better in the second language, with an open-minded confidence.

(A)  The world view of the human being today is influenced by a lot of things. Critical thinking cannot be taught as it involves reflective teaching and could be misunderstood.

(B) Critical thinking must be part of an individual’s education but it is a difficult concept because it very often reflects a teacher as a dominating individual.

(C)  Critical thinking is an important aspect of an individual’s literacy level and his social knowhow. It is part of civilization and eventually leads to global sensitivity. Students can look at information in a variety of ways.

(D)  Reflective teaching is another aspect of initial thinking and it could lead to individual and social illiteracy.

Answer: (C)

14. Direction- Four alternatives are given in the text. Choose the best captures the essence of the text.

The construct of democracy rests on informed choice and governance on informed policy. Answers to critical questions are stranded in the arena of claims and counter claims. Measures of progress are simply not known or available. The debate to job creation is a mystery of sorts. The public are not aware that the Centre and States spend crores on education, health and social services. Take education, Barely a few of Class V students can read a Class II text, the poor are switching to private schools and over 33% of million plus schools and over 33% of million plus schools do not maintain a feasible pupil teacher ratio.

(A)  Being informed is important aspect of any democracy. This information is not available especially in the area of education, health and social services.

(B) Answers are often given but they need to deciphered accurately. Job clarity is available but sometimes it is a mystery. There is concusion about information.

(C)  Health and social services is clearly communicated but education remains an area of ignorance. Claims and counter claims create even more confusion.

(D)  Crores are spent on many areas of growth but there is no accountability for information. Parents are switching to private schools as the information on education is available there.

Answer: (A)

Directions- (Q. 15-19) The word given in each of the sentences is used contextually. Pick the word from the alternatives given that is most inappropriate in the given context.

15. VENERATE – We venerate aged priests and martyrs.

(A)  respect  

(B) revere

(C)  spurn

(D)  honor

Answer: (C)

16. PRUDENCE – Prudence and sacrifice help us in meeting the future.

(A)  caution

(B) reckless

(C)  foresight

(D)  judgement

Answer: (B)

17. OSTENATION-Ostenation in wealth and living styles can sometimes turn vulgar.

(A)  pageantry

(B) display

(C)  modesty

(D)  flourish

Answer: (C)

18. DOCTRINE-A doctrine is often refined by reasoning.

(A)  offense

(B) dogma

(C)  precept

(D)  principle

Answer: (A)

19. BENEVOLENCE : Every religion believes in the concept of benevolence.

(A)  generosity

(B) malignity

(C)  liberality

(D)  benignity

Answer: (B)

20. Directions-(Q. 20-21) Answer the question, based on the following information. Indicate which of the statements given with that particular question, is consistent with the information given in the passage below :

A Holistic Viewpoint

It is now recognized by modern science that the universe at the subatomic level does not have solid material objects, but consists of only wave like patterns which represent probabilities of interconnections between other interconnections, all of which together constitute an inseparable web of inter-relationships constituting the entire universe. Fritj of Capra therefore, views the universe not as “an assemblage of independent parts” but as “a dynamic web of inter-related events” in which each of the web determines the structure of the whole. Geoffrey Chew views such inter-penetrating and interdependent relationship in the universe in terms of a “boo-strap” theory which implies that all forces in the universe are inseparably linked together, every part affects every other part, and the whole world is held together so to say, by bootstraps. David Bohm refers to a holographic concept which implies not only that every part is connected with every other part within the whole but also that, in a sense, each part contains the whole. This, according to David Bohm, recognizes the “Un-divided wholeness” of the entire universe instead of the classical idea of analyzability of the world into separately and independently existent parts. Choose the appropriate option :

(A)  there are many solid material objects in the universe

(B) the entire universe a web of inter-relationships

(C)  the universe comprises of a number of independent parts

(D)  these independent parts determine the structure of the web

Answer: (B)

21. 

(A)  the holographic theory speaks of the exclusiveness of each part

(B) the word can be analysed into separate independent parts.

(C)  the classical idea recognizes the concept of wholeness.

(D)  the boot strap theory speaks of interdependent relationships

Answer: (D)

Directions – (Q. 22-24) In the following sentence a part of sentence is underlined. Beneath each sentence four different ways of paraphrasing the underlined part are indicated. Choose the best alternative among the four options.

22. The police searched everywhere but they drew a blank.

(A)  upset

(B) did not find any information

(C)  came to an end

(D)  drew a line

Answer: (B)

23. She was accused of being close-minded and intolerant.

(A)  did not agree

(B) kept of herself

(C)  not open to new ideas

(D)  unwilling to discuss

Answer: (C)

24. Rehman likes to have a finger in every pie.

(A)  likes cooking

(B) wants to do everything

(C)  object to everything

(D)  cannot take decisions

Answer: (B)

Directions- (Q. 25-27) In the question below, a related phrase is followed by a group of words. Select the group of words that best expresses a relationship similar to the one expressed in the original phrase.

25. At one’s wits end

(A)  cannot say anything funny

(B) not able to think logically

(C)  not able to work

(D)  clarity of thought

Answer: (B)

26. Gift of the gab

(A)  clever at work

(B) flatter someone

(C)  promise blatantly

(D)  fluent at speaking

Answer: (D)

27. Die in harness

(A)  die while riding

(B) die at the hight of one’s success

(C)  die with help

(D)  die when the work has been completed

Answer: (B)

Directions- (Q. 28-37) Read the passage given below. Choose the best options for the Question.

       IOT has had an impact across all fields, be it industries, government, small or large business and even for Personal Consumption.

      What is IOT (Internet of things) you might ask. It’s been a growing topic of conversation for some time now. Put in the simples term it means anything that has an on and off button and is connected to the internet for receiving, analyzing, storing or sending data. This could mean anything, from the watch that you wear to airplanes that can be controlled from a remote according to the analyst firm Gartner, by the year 2020 we’ll have over 26 billion connected devices. That could mean people to people, people connected to things and things connected to things. The new rule of the future is going to be “Anything that can be connected will be connected”. Take for example that when you set an alarm, to wake up and that alarm goes off it not only wakes you up but also brews your coffee, sets the right temperature of water for your bath, put on the television to bring you the latest updates from around the globe and all this before you even put a foot out of your bed. This is all done by simply getting the network and all this before you even put a foot out of our bed. This is all done by simply getting the network of interconnected things/devices that have embedded sensors, network connectively, software and necessary electronics that collect and exchange data.

   To show how far we have come with technology and connectively, we have smart watches such a Fitbit, Garmin to name a few that have changed the way we look at time. We have one device that not only tells us the time but also tracks the number of steps, calories and our heart rate. This watch is actually connected to our phone so with just one turn of the wrist one can tell who is calling or what messages have been received without having to dig through pockets or handbags.

    IOT is making its presence felt in health care as well. Doctors can now remotely monitor and communicate with their patients and health care providers can benefit from this. Whether data comes from foetal monitors, electrocardiograms, temperature monitors or blood glucose levels, tracking this information is vital for some patients. Many of this requires follow up interaction with healthcare professionals. With smarter devices that deliver more valuable data it can reduce the need for direct patient-physical interaction.

    Take for instance in the sporting field, minute chips are being attached to balls and bats which will transmit information of how fast the ball is travelling and a baseman’s moves, the time, the angles, the pressure on the bat at different positions, data of the muscle stretch if he’s hit a six so on and so forth. Formula one cars are also being fitted with these sensors which relays information on the minute moves being made by the driver. Chips are also being put into wearable devices of sportsmen to detect sub-optimal action of any body parts to show signs of stress or strain which will help in the early detection of injuries and take preventive measures.

     IOT had an impact across all fields, be it industries, government, small or large business and even for personal consumption. IBM, Google, Intel, Microsoft and Cisco are some of the top players in the IOT spectrum.

    With billions of devices connected security becomes a big issue. How can people make sure that their data is safe and secure ? This is one of the major concerns in the IOT that becomes a hot topic. Another issue is with all these billions of devices sharing data companies will be faced with the problem of how to store, track, analyse and make vase sense of the information being generated. Companies are monitoring the network segment to identify anomalous traffic and to take action if necessary.

    Now that we have a fair understanding of IOT let’s see what impact it has on the education sector. The only constant in our lives is change and learning. From the get go we learn, be it to the walk or run. We adapt to the changing times and constantly learn from it. Education or learning as we know it in the broader sense is the most important of all and the one that decides which way we handle those changes to impact us an the world.

    Today’s world is fast paced and to keep with we need an infusion of speed with learning. From the class-room assignments, lectures, black-boards and chalk we have come a long way to what is now known as e-learning (electronic learning) or m-learning (mobile learning). With GenNext it is imperative to provide the right kind of education.

   The rise of technology and IOT allows schools to improve the safety of their campuses, keep track of resources and enhance access to information. It ensure data quality being the top priority but also facilitates development of content allowing teachers to use this technology to create smart lesson plans and ensuring the reach of this content to any corner of the world.

28. How is IOT a concept of connecting?

(A)  It is the internet in a different form.

(B) It is the internet but connected to anything.

(C)  It is an analysing machine.

(D)  It is a data collection term.

Answer: (B)

29. It is said that we look at time differently now, why?

(A)  It is connected different devices.

(B) It is an a different format.

(C)  It is connected to the aeroplane

(D)  It connects people.

Answer: (A)

30. IOT will help in simplifying patient-physician interaction because :

(A)  It helps reduce datas.

(B) Doctor can remotely connected to patients

(C)  It improves conductivity

(D)  It is a simple process

Answer: (B)

31. How will IOT work in the sporting field?

(A)  Through sport fields with chips

(B) Through special machine

(C)  Through minute wearable chips which are connected.

(D)  Through special moves by the sportsmen

Answer: (C)

32. Preventive measures is imperative because :

(A)  it can detect anything

(B) It identifies treatment needed

(C)  It keeps them in touch

(D)  It helps to identify stress and strain

Answer: (D)

33. One of the major concerns of IOT is :

(A)  Security and privacy, data loss and tracking data

(B) Only security and privacy

(C)  Only data loss

(D)  Only tracking data

Answer: (A)

34. Technology and IOT has benefited education considerably because it has :

(A)  Helped in improving the lesson.

(B) Helped keep track of all resources

(C)  Helped teachers to track

(D)  Helped with keeping up with the change in learning

Answer: (D)

35. Teachers can use this data :

(A)  Only to teach with more resources

(B) Only to create a smart class-room

(C)  Only to reach out to the world

(D)  To reach with more resources, create a smart classroom and reach out to the world

Answer: (D)

36. Select a title that is most suitable for the passage :

(A)  IOT Curse of Boon

(B) IOT and Our Life Tomorrow

(C)  IOT is Changing the Way We Learn

(D)  IOT is a All Pervading

Answer: (B)

37. To conclude, IOT has an impact on :

(A)  Education, sports, medicine and our daily lives

(B) Education, sports only

(C)  Education only

(D)  Education and medicine only

Answer: (A)

Directions- (Q. 38-40) Choose the option closest to the Capitalized word.

38. ANNIHILATE

(A)  devastate

(B) delay

(C)  investigate

(D)  retaliate

Answer: (A)

39. FACILITATE

(A)  lend

(B) share

(C)  support

(D)  serve

Answer: (C)

40. CAMPAIGN

(A)  lobby

(B) run

(C)  agree

(D)  work

Answer: (A)

41. Who wrote the memoir book ‘My History’?

(A)  Dr. G. Madhavan Nair

(B) Stephen Hawking

(C)  Angeline Merkel

(D)  Rosalind Franklin

Answer: (B)

42. What is the Motto of the Commonwealth Games 2018?

(A)  People, Place, Passion

(B) Share the dream

(C)  Come out and play

(D)  United by the moment

Answer: (B)

43. The 2024 Summer Olympic will be held in Paris, France for third time. How many years before that, did the city host the Summer Olympic for the second time ?

(A)  100 years

(B) 96 years

(C)  92 years

(D)  99 years

Answer: (A)

44. Who won the men’s Singles title in Australian Open 2018?

(A)  Dylan Alcott

(B) R. Nadal

(C)  Roger Federer

(D)  Marin Cilic

Answer: (C)

45. In which election was the Electronic Voting Machines tried out on an experimental basis in India?

(A)  The 1982 Kerala Legislative Assembly Elections

(B) The Karnataka State for the1984, Legislative Assembly Elections.

(C)  The U.P. State for the 1989, Lok Sabha Elections

(D)  The 1987, Delhi Corporation Election in Karol Bagh constituency

Answer: (A)

46. Recently a star, lcarus was seen by NASA’s Hubble Space Telescope. What are the approximate time taken by lcrus’ light to reach the Earth?

(A)  Nine billion years

(B) Ten million years

(C)  Sven trillion years

(D)  One hundred years

Answer: (A)

47. Who won the first Gold Medal for India at the Commonwealth Games 2018?

(A)  Punam Yadav

(B) Mirabai Chanu

(C)  Jitu Rai

(D)  Sanjita Chanu

Answer: (B)

48. Who is the first Indian to hold the post of Deputy Director General for Programmes at the World Health Organisation?

(A)  Rasha Omer

(B) Vijayalakshmi Pandit

(C)  Soumya Swaminathan

(D)  Kiran Bedi

Answer: (C)

49. Who is the first female fighter pilot of India?

(A)  Punit Arora

(B) Avani Chaturvedi

(C)  Mitali Madhumita

(D)  Priya Jhingan

Answer: (B)

50. The reserve Bank of India issued the 100 rupee coin to mark the birth centenary of which Carnatic Music Legend?

(A)  Dr. Balamurli Krishna

(B) Dr. M.s. Subbulakshmi

(C)  Muthuswami Deekshithar

(D)  Chembai Vaidunatha Bhagavathar

Answer: (B)

51. Which type of mango from Andhra Pradesh received a Geographical Indication (GI) Tag in 2017?

(A)  Alphonso

(B) Banganapalle

(C)  Totapuri

(D)  Himsagar

Answer: (B)

52. The International Day of the Girl Child is observed b y the UN on:

(A)  March 8

(B) August 12

(C)  October 16

(D)  October 11

Answer: (D)

53. Where is the temple of Angkor Wat located?

(A)  Singapore

(B) Cambodia

(C)  Thailand

(D)  Indonesia

Answer: (B)

54. Which one of the following States lifted the Santosh Football title 2018?

(A)  Kerala

(B) Punjab

(C)  Karnataka

(D)  Uttar Pradesh

Answer: (A)

55. IRDA stands for :

(A)  Insurance Regulatory and Development Authority

(B) International Resources Development Agency

(C)  Indian Roads Development Authority

(D)  Indian Rural Development Authority

Answer: (A)

56. The first Defence Minister of India was :

(A)  K. M. Cariappa

(B) V. K. Krishna Menon

(C)  Baldev Singh

(D)  Kailash Nath Katju

Answer: (C)

57. Who is the legendary figure who won both a Noble Prize and an Oscar?

(A)  Satyajit Ray

(B) William Goldman

(C)  Gabriel Garcia Marquez

(D)  George Bernard Shaw

Answer: (D)

58. The discipline that deals with map making is known as :

(A)  Cryptography

(B) Carpology

(C)  Cartography

(D)  Lithography

Answer: (C)

59. The prize established, based on a donation from the Sveriges Riksbank in memory of Alfred Nobel, is the :

(A)  Prize in Medicine

(B) Prize in Economic Sciences

(C)  Peace Prize

(D)  Prize in Literature

Answer: (B)

60. What is the significance of May 13, 1952 with regard to Indian polity?

(A)  First sitting of the Lok Sabha

(B) First sitting of the Rajya Sabha

(C)  First sitting of the Supreme Court of India

(D)  Formation of the Election Commission of India

Answer: (B)

61. Which among the following is the earliest literary work in Tamil language?

(A)  Manimegalai

(B) Silapadikaram

(C)  Thirukkural

(D)  Tolkappiyam

Answer: (D)

62. Who has been appointed in 2018 as the Principal Scientific Advisor to the Government of India?

(A)  S. N. Joshi

(B) Rakesh Aggarwal

(C)  K. Vijay Raghavan

(D)  Vinod Kumar

Answer: (C)

63. Escape velocity of a rocket fired from the earth towards the moon is a velocity to get rid of the :

(A)  Centripetal force. due to Earth’s revolution

(B) Earth’s gravitational pull

(C)  Centripetal force due to Earth’s rotation

(D)  Moons gravitational pull

Answer: (B)

64. From which animal is catgut usually made of?

(A)  Sheep

(B) Camel

(C)  Cat

(D)  Rabbit

Answer: (A)

65. Who wrote the book Der Ursprung der Familie, des Privat-eigenthums and des staats/The Origin of the Family, Private Property, and the State?

(A)  Friedrich Engels

(B) Lewis H. Morgan

(C)  Karl Marks

(D)  Friedrich Carl von Savigny

Answer: (A)

66. Name the part of the human body reclassified as an organ due to research conducted in the past decade.

(A)  Alimentary

(B) Mesentery

(C)  Epithelium

(D)  Spleen

Answer: (B)

67. The World’s first ‘negative emissions’ power plant has been built in-

(A)  Iceland

(B) Poland

(C)  Finland

(D)  China

Answer: (A)

68. Which platform has been launched by the Union Government on the occasion of the 2018 International Women’s Day?

(A)  Women Entrepreneurship Platform

(B) Women Combat Platform

(C)  Women Power Platform

(D)  Women Literacy Platform

Answer: (A)

69. Who is the author of ‘The Coalition Years 1996-2012’?

(A)  Manmohan Singh

(B) Pratibha Patil

(C)  Sharad Pawar

(D)  Pranab Mukherjee

Answer: (D)

70. What does ‘NITI’ in Niti Ayog stand for?

(A)  National Invitation to Invest In India

(B) New India Transformation Initiative

(C)  National Institution for Transforming India

(D)  Nayi India Turant India

Answer: (C)

71. In commemoration of which of the following events, is May 1 celebrated in many places as the ‘International Workers’ Day?

(A)  The Haymarket Affair, Chicago.

(B) Foundation Day of International Labour Organization

(C)  Commencement of the First International Labour Conference

(D)  Concluding Day of the Russia Revolution

Answer: (A)

72. The National Social Security Board functioning under the Ministry of labour and Employment is meant for recommending formulation of social security schemes for :

(A)  Industrial workers

(B) Women and Children

(C)  Government employees

(D)  Unorganised workers

Answer: (D)

73. The scarcity definition of economics is credited to:

(A)  Alfred Marshall

(B) Lionel Robbins

(C)  Adam Smith

(D)  Dennis Robertson

Answer: (B)

74. Name the digital payment app introduced by Google in 2017 for India.

(A)  BHIM

(B) Tez

(C)  Citrus Pay

(D)  Mobikwik

Answer: (B)

75. Which of the following countries runs bullet trains with the highest operational speed?

(A)  China

(B) Germany

(C)  South Korea

(D)  USA

Answer: (A)

76. Who became the Chief Justice of the Federal Court of India on 14 August, 1947?

(A)  Maurice Gwyer

(B) Patanjali Shastri

(C)  H.J. Kania

(D)  Srinivas Varadachariar

Answer: (C)

77. In which country outside India has the Energy Efficiency Services Limited, under the Ministry of Power, Government of India, launched the UJALA scheme in 2017 ?

(A)  Malaysia

(B) Srilanka

(C)  Indonesia

(D)  Maldives

Answer: (A)

78. Which is the organization that initiates and presents the National Awards in India?

(A)  Film and Television of India

(B) Central Board of Film Certification

(C)  Directorate of Film Festivals

(D)  Prasar Bharati

Answer: (C)

79. In 2017, eBay India was merged with which of the following Companies?

(A)  Flipkart

(B) Alibaba

(C)  Naaptol

(D)  Snapdeal

Answer: (A)

80. The world’s first fully Solar-powered Airport is :

(A)  Cochin International Airport

(B) Shanghai Pudong International Airport

(C)  Dubai International Airport

(D)  Singapore Changi Airport

Answer: (A)

81. The winner of the Jnanpith Award 2017 is :

(A)  G. Sankara Kurup

(B) Shankha Ghosh

(C)  Raghuvir Chaudhari

(D)  Krishan Sobti

Answer: (D)

82. What do the paintings of Ajanta caves primarily depicts?

(A)  Ramayana

(B) Panchatantra  

(C)  Jataka Tales

(D)  Mahabharata

Answer: (C)

83. Which country joined as the eight member of the South Asia Sub-regional Economic Cooperation (SASEC) in February 2017?

(A)  Myanmar

(B) Nepal

(C)  Bhutan

(D)  Bangladesh

Answer: (A)

84. What is the theme of the 2048 World Wildlife Day?

(A)  Stop wildlife crime

(B) Listen to the young voices

(C)  Bit cats : predators under threat

(D)  The future of wildlife is in our hands

Answer: (C)

85. In which athletics event did Bob Beamon hold the world record for 23 years?

(A)  Triple Jump

(B) High Jump

(C)  Long Jump

(D)  Steeple Chase

Answer: (C)

86. Which State government has come up with a Water ATM Policy in April 2018?

(A)  Haryana

(B) Maharashtra

(C)  Uttar Pradesh

(D)  Madhya Pradesh

Answer: (A)

87. The military operation which annexed Hyderabad into the Indian Union was code-named as :

(A)  Operation Meghdoot

(B) Operation Polo

(C)  Operation Viraat

(D)  Operation Vijay

Answer: (B)

88. Which of the following movies won the Best Movie Oscar Award, 2018 ?

(A)  The Shape Water

(B) Call Me by Your Name

(C)  Darkest Hour

(D)  Dunkirk

Answer: (A)

89. Who is the first woman Defence ministry of Independent India?

(A)  Nirmala Sitharaman

(B) Indira Gandhi

(C)  Sushma Swaraj

(D)  Sheela Dixit

Answer: (B)

90. What is the motto of the Supreme Court of India?

(A)  Satyameva Jayate

(B) Dharmo Rakshati Rakshitah

(C)  Sarve Jana Sukhno Bhavanthu

(D)  Yato Dharmastato Jayah

Answer: (D)

91. Find the missing group in the sequence :

X7D, V11G, T13J, ……, P19P

(A)  Q15M

(B) R17M

(C)  Q17L

(D)  R15M

Answer: (B)

92. Find the missing number in the sequence :

9, 3, …….., 1/3, 1/9

(A)  2

(B) 6

(C)  1/2

(D)  1

Answer: (D)

93. Which word does not belong with the others?

(A)  Flute

(B) Clarinet

(C)  Saxophone

(D)  Violin

Answer: (D)

94. Find the next group of alphabets :

CFI, DHL, ILO, LPT, …..?

(A)  ORU

(B) RUW

(C)  OQT

(D)  OSV

Answer: (A)

95. If ‘VOLKSWAGEN’ is written as ‘UQKMRYZIDP’ then ‘SWITZERLAND’ is written as ……?

(A)  RVHSYDFPMOD

(B) TXJUAFSMBOE

(C)  RYHYVGQNZPE

(D)  RYHVYGQNZPC

Answer: (D)

96. In an archery match, Suresh’s team got more scores than Mahes’s team but not as many as Yogesh’s team. Yogesh’s team gor more scores than Mukesh’ team. Mukesh team got less score than Mahesh’s team.

Which team is in second place in the descending order of scores?

(A)  Yogesh’s team

(B) Mukesh’s team

(C)  Suresh’s team

(D)  Maheshs’s team

Answer: (C)

97. Choose the word that is not necessary part of the bold word.

Movie.

(A)  Scene

(B) Video

(C)  Duration

(D)  Dialogue

Answer: (D)

98. The doctor is a person who looks after the sick people and prescribes medicines so that the patient recovers fast. In order to become a doctor, a person has to study medicine. Doctors lead a hard life. Their life is very busy. They get up early in the morning and go to the hospital. They work without taking a break. They always remain polite so that patients feel comfortable with them. Since doctors work so hard we must realise their value.

Based on the above paragraph, who among the following is/are doctors(s)?

(i) Kumar Sharma wakes up early in the morning everyday and treats everyone with patience. He usually gives emergency aid to the persons who are injured in accidents.

(ii) Rakesh studied medicine in Russia and practiced medicine there for 2 years. Later, stopped medical practice and turned into some business

(iii) Shalini works in Apollo Hospitals. She always remains polite so that patients feel comfortable. She is a specialist in providing quality care for the cancer patients.

(A)  All are doctors

(B) Rakesh and Shalini are the doctors

(C)  Kumar Shrma and Rakesh are doctors.

(D)  Only Rakesh is a doctor.

Answer: (D)

Directions- (Q. 99 to 103) Read the passage given below and answer the Question.

        Ten friends attended a marriage, five of them Venkat, Manohar, Ravi, Parasnath and Tilak are sitting in a row facing North and other five friends Vidya, Maya, Divya, Keerthi and Anu are sitting in a row facing South not exactly in the same order, Each friend in a row is facing exactly one from the other row. One of them is using a Skoda Car. Manohar sits opposite to his friend who is using a Ford Car and sits at one of the extremes. The friend who uses a Honda car, is not Ravi who sits second to the right of Manohar. Venkat sita exactly in the middle of Ravi and the friend who uses a Nissan car is not Manohar. Keerthi does not sit at the extreme end but uses a Tata car and sits opposite the friend who uses a Fiat Car. The one who uses a Maruthi car sits opposite to the friend who is to the immediate left o Keerthi. The friend who uses a Toyota Car but not Anu, sits opposite Prashant. Tilak does not sit at any of the extremes, but sits opposite the friend who used a Chevrolet car, and is sitting adjacent to Divya and the friend who uses a  Mahindra car, Vidya neither uses Chevrolet nor Toyota cars.

99. Who uses the Mahindra Car?

(A)  Venkat

(B) Anu

(C)  Vidya

(D)  Divya

Answer: (C)

100. Keerthin is how many places away from the one who uses the Ford car?

(A)  One

(B) Three

(C)  Four

(D)  Two

Answer: (B)

101. Who sits third to the left to the one who uses the Chevrolet?

(A)  Anu

(B) Maya

(C)  Divya

(D)  Vidya

Answer: (B)

102. Which one does not belong to the group?

(A)  Maruthi

(B) Skoda

(C)  Honda

(D)  Mahindra

Answer: (D)

103. Which is the car used by Maya?

(A)  Toyota

(B) Chevrolet

(C)  Mahindra

(D)  Ford

Answer: (A)

104. In a queue. I am the last person while my friend is seventh from the front. If the person exactly between me and my friend is on the 23rd position from the front, what is my position in the queue?

(A)  37

(B) 36

(C)  38

(D)  39

Answer: (D)

Directions-(Q. 105-106) Read the statement and on the basis of that, choose the most appropriate course of action(s) given below the statement.

105. Statement : Most of those who study in premier Medical colleges in India migrate to developed nations for better prospects in their professional pursuits.

(I) All the students joining these colleges should be asked to sign a bond at the time of admission to the effect that they will remain in India t least for ten years after they complete their medical education.

(II) All those students who desire to settle in the developed nations should be asked to pay the entire cost of their education which the government subsidised.

(A)  Only I follows

(B) Only II follows

(C)  Both I and II follow

(D)  Neither I nor II follows

Answer: (B)

106. Statement : Official data show more people died on Indian roads in 2016 than in 2015; UP and Tamil Nadu accounted for the largest numbers of fatalities.

(I) Government should make a policy regulating the manufacturing of automobiles for private use.

(II) Government should take steps to creat awareness among the public about road safety.

(III) Accidents can b e avoided if the Governments takes steps to make good roads.

(IV) To eliminate accidents completely the Government should impose stringent punishments for traffic violations.

(A)  Only II and IV follow

(B) Only II follows

(C)  Only II and IV follow

(D)  Only IV follows

Answer: (B)

Directions- (Q. 107-108) Read the statement and presume that whatever statements given are true. On the basis of that, choose the most appropriate conclusion(s) given below :

107. Statements : Some rats are cows.

All cows are animals.

Conclusions:

(I) All rats are animals

(II) Some animals are rats

(A)  Only conclusion I follows

(B) Only conclusion II follows

(C)  Both conclusion I and II follow

(D)  Neither conclusion I nor conclusion II follows

Answer: (B)

108. Statements : All the students are young. All the teens are young. Some men are teens.

Conclusions:

(I) Some students are teens.

(II) Some young are students

(iii) Some young are men

(A)  Only (I) follows

(B) Only (II) and (III) follow

(C)  Only (I) and (II) follow

(D)  Only (I) and (III) follow

Answer: (B)

109. ‘X’ started walking towards the west, after travelling 5 kilometres turned exactly to his right and walked 3 km. Then the decided to walk exactly to his left. On reaching 2 kilometers, he heard a voice and walked back. Then he noticed that, his friend ‘Y’ was walking towards him from his right side. In which direction ‘Y’ walking?

(A)  East

(B) West

(C)  North

(D)  South

Answer: (C)

110. If ‘DECEMBER’ is ‘AUGUST’, ‘MARCH’ is ‘MAY’, ‘SEPTEMBER’ is ‘SEPTEMBER’ and ‘OCTOBER’ is ‘JULY’ then JUNE is :

(A)  February

(B) April

(C)  January

(D)  November

Answer: (B)

Common Law Admission Test (CLAT) Examination Held on May 8, 2016 Question Paper With Answer Key

Common Law Admission Test (CLAT) Examination Held on May 8, 2016
Common Law Admission Test (CLAT) Examination Held on May 8, 2016 Question Paper With Answer Key

Common Law Admission Test

CLAT Examination Held on May 8, 2016

English Including Comprehension

 

Direction for Questions 1-10: Fill in the blanks by choosing the most appropriate option:

1. I like reading journals _______ novels.

(a)  The best

(b)  more than

(c)  most than

(d)  the less than

Answer: (b)

2. There isn’t _______ water in the overhead tank.

(a)  Lot of

(b)  any

(c)  little

(d)  something

Answer: (b)

3. They always give the available seats to _______ comes first.

(a)  Whomever

(b)  whom

(c)  whichever

(d)  whoever

Answer: (d)

4. A fire broke _____ in the neighbourhood.

(a)  Off

(b)  out

(c)  away

(d)  from

Answer: (b)

5. If you promise _______ angry with me, I’ll tell you what I broke.

(a)  not to get

(b)  get out

(c)  not getting

(d)  not get

Answer: (a)

6. A thief does not ______ the door.

(a)  Knock at

(b)  knock on

(c)  knock to

(d)  knock

Answer: (a)

7. There is a lot of work ______ hand. Let’s cancel ______ picnic.

(a)  Upon, the

(b)  in, a

(c)  in, the

(d)  on, a

Answer: (c)

8. It’s unfortunate that he died _____ cancer.

(a)  By

(b)  of

(c)  from

(d)  with

Answer: (b)

9. Professor Ahmed ____ teaching us _____ August, 2012.

(a)  Was, for

(b)  had been, since

(c)  has been, since

(d)  has been, for

Answer: (c)

10. The method and practice of teaching is called ____.

(a)  Paediatrics

(b)  pedagogy

(c)  training

(d)  philately

Answer: (b)

Direction for questions 11-15: Complete the sentences with the correct options:

11. Her parent have arrived ________ the airport.

(a)  to

(b)  at

(c)  on

(d)  in

Answer: (b)

12. They returned home _____ a taxi.

(a)  in

(b)  from

(c)  on

(d)  with

Answer: (a)

13. I have never come _______ nay one as rude as him.

(a)  across

(b)  to

(c)  around

(d)  at

Answer: (a)

14. I can’t bear _______ late.

(a)  been

(b)  be

(c)  to

(d)  being

Answer: (d)

15. Mani, along with his friends, _______ for basket ball practice every morning.

(a)  were going

(b)  are going

(c)  goes

(d)  go

Answer: (c)

Direction for questions 16-20: Choose the correct spellings in questions given below:

16. Choose the correct spellings

(a)  Sacribegous

(b)  saereligious

(c)  sacrilegious

(d)  sacrilegiuos

Answer: (c)

17. Choose the correct spellings

(a)  Deceive

(b)  deceeve

(c)  diecieve

(d)  decieve

Answer: (a)

18. Choose the correct spellings

(a)  Collaborate

(b)  colaborate

(c)  collabortae

(d)  collaborrate

Answer: (c)

19. Choose the correct spellings

(a)  Integrity

(b)  integerety

(c)  intigerity

(d)  integerity

Answer: (a)

20. Choose the correct spellings

(a)  Berrister

(b)  barrister

(c)  barisster

(d)  berister

Answer: (b)

Directions for Questions 21-24 : Choose the most appropriate option for each of the following questions:

21. “Faux pas” means:

(a)  Expected to happening

(b)  flase

(c)  social blunder

(d)  fake identity

Answer: (c)

22. “Ab initio” means:

(a)  In continuation with

(b)  from the beginning

(c)  in defence of a comment

(d)  in connection with the future

Answer: (b)

23. “To bury the hatchet” means:

(a)  In continuation with

(b)  from the beginning

(c)  in defence of a comment

(d)  in connection with the future

Answer: (a)

24. “Amicus curiae” means:

(a)  One of the judges of the court

(b)  A friend of the court

(c)  One among the parties to the dispute

(d)  One among the expert witnesses in a case.

Answer: (b)

Direction for Questions 25-28: Choose the correct option for each of the following questions:

25. Choose the correct option

(a)  Please stop interfering into my romantic life.

(b)  Please stop interfering in my romantic life.

(c)  Please stop interfering with my romantic life.

(d)  Please stop interfering for my romantic life.

Answer: (b)

26. Choose the correct option

(a)  The Titanic did not reach their destination

(b)  The Titanic did not reach her destination

(c)  The Titanic did not reach his destination

(d)  The Titanic did not reach it’s destination

Answer: (b)

27. Choose the correct option

(a)  The Film Star climbed off ther car and smiled at the people.

(b)  The Film Star come out of the car and smiled at the people.

(c)  The Film Star got out of the car and smiled at the people.

(d)  The Film Star got off the car and smiled at the people.

Answer: (c)

28. Choose the correct option

(a)  Your grammar is very good, but you need to work on correcting your pronunciation.

(b)  Your grammar is very good, but you need to work on managing your pronunciation.

(c)  Your grammar is very good, but you need to work on repairing your pronunciation.

(d)  Your grammar is very good, but you need to work on modifying your pronunciation.

Answer: (a)

Direction for Questions 29-30) : Complete the proverb, in the following questions:

29. When _____ is bliss, it is _____ to be wise.

(a)  Romance, boring

(b)  ignorance, folly

(c)  knowledge, better

(d)  bachelorhood, single

Answer: (b)

30. ______ waters run _____.

(a)  Hot, fast

(b)  Muddy, slow

(c)  Stagnant, leisurely

(d)  Still, deep

Answer: (d)

Direction for Question 31-40: Read the given passage carefully and answer the questions given after the passage:

1. Often, we passionately pursue matters that in the future appear to be contradictory to our real intention or nature; and triumph is followed by remorse or regret. There are numerous examples of such a trend in the annals of history and contemporary life.

2. Alfred Nobel was the son of Immanuel Nobel, an inventor who experimented extensively with explosives. Alfred too carried out research and experiments with a large range of chemicals; he found new methods to blast rocks for the construction of roads and bridges; he was engaged in the development of technology and different weapons; his life revolved around rockets and cannons and gun powder. The ingenuity of the scientist brought him enough wealth to buy the Bofors armament plant in Sweden.

3. Paradoxically, Nobel’s life was a busy one yet he was lonely; and as he grew older, he began suffering from guilt of having invented the dynamite that was being used for destructive purposes. He set aside a huge part of his wealth to institute Nobel Prizes. Besides honouring men and women for their extraordinary achievements in physics, chemistry, medicine and literature, he wished to honour people who worked for the promotion of peace.

4. It’s strange that the very man whose name was closely connected with explosives and inventions that helped in waging wars willed a large part of his earnings for the people who work for the promotion of peace and the benefit of mankind. The Nobel Peace Prize is intended f or a person who has accomplished the best work for fraternity among nations, for abolition or reduction of war and for promotion of peace.

5. Another example that comes to one’s mind is that of Albert Einstein. In 1939, fearing that the Nazis would win the race to build the world’s first atomic bomb, Einstein urged President Franklin D Roosevelt to launch an American programme on nuclear research. The matter was considered and a project called the Manhattan Project was initiated. The project involved intense nuclear research the construction of the world’s first atomic bomb. All this while, Einstein had the impression that the bomb would be used to protect the world from the Nazis. But in 1945, when Hiroshima was bombed to end World War II, Einstein was deeply grieved and he regretted his endorsement of the need for nuclear research.

6. He also stated that had he known that the Germans would be unsuccessful in making the atomic bomb, he would have probably never recommended making one. In 1947, Einstein began working for the cause of disarmament. But, Einstein’s name still continues to be linked with the bomb.

Man’s fluctuating thoughts, changing opinions, varying opportunities keep the mind in a state of flux. Hence, the paradox of life: it’s certain t hat nothing is certain in life.

31. The Manhattan Project was initiated ______.

(a)  in honour of Einstein.

(b)  to protect the Nazis.

(c)  to bomb Hiroshima.

(d)  to carry out nuclear research

Answer: (d)

32. 32 Alfred established the Nobel Prizes to ______.

(a)  remind people of his achievements.

(b)  ease his guilt and promote work for the betterment of mankind.

(c)  use his wealth for hard working people.

(d)  honour only those people who are intelligent

Answer: (b)

33. In paragraph 4, the word ‘accomplished’ means ___________.

(a)  completed successfully.

(b)  worked hard.

(c)  won awards

(d)  made an effort to do something

Answer: (a)

34. In the fifth paragraph, the word ‘endorsement’ means

(a)  expressing one’s opposition.

(b)  expressing one’s approval or support.

(c)  making a promise to do something.

(d)  expressing one’s regret.

Answer: (b)

35. Working with arms and ammunition helped Alfred to amass _______.

(a)  enemies

(b)  intelligence

(c)  wealth

(d)  popularity

Answer: (c)

36. Immanuel’s interest in dynamites influenced Alfred’s inclination for working ___________.

(a)  for humanity

(b)  with explosives

(c)  for the Nobel Peace Prize

(d)  with contradiction

Answer: (b)

37. One of the paradoxes in Alfred’s life was that he was ___________.

(a)  lonely yet rich

(b)  hard working but a failure

(c)  intelligent yet lonely

(d)  occupied yet lonely

Answer: (d)

38. Einstein had the impression that the Germans would __________.

(a)  bomb Hiroshima.

(b)  be successful in making the world’s first atomic bomb.

(c)  be unsuccessful in making the atomic bomb.

(d)  work for humanity.

Answer: (b)

39. The passage is _____________.

(a)  an argumentative essay

(b)  an expository essay.

(c)  a process essay.

(d)  a descriptive essay.

Answer: (d)

40. The paradox, ‘it’s certain that nothing is certain in life’, indicates the writer’s

(a)  hatred for scientists

(b)  analytical mind

(c)  scientific mind

(d)  persuasive nature

Answer: (b)

Section : General Knowledge and Current Affairs

Direction for Questions 41-90: Choose the most appropriate option:

41. India signed the “Paris Agreement on Climate Change” in April, 2016 at:

(a)  New York

(b)  Paris

(c)  New Delhi

(d)  Geneva

Answer: (a)

42. The country which cloned a buffalo calf first is:

(a)  India

(b)  U. S. A.

(c)  Sweden

(d)  Denmark

Answer: (a)

43. Marks is also known as the:

(a)  Blue planet

(b)  Brown  planet

(c)  Red planet

(d)  Green planet

Answer: (c)

44. In 2015, Prime Minister Mr. Narendra Modi launched a new campaign, “Start up India, Stand up India”. The campaign is aimed at:

(a)  Making India Digital

(b)  Promoting bank financing for start ups and offer incentives to boost entrepreneurship and job creation.

(c)  Promoting Swatch Bharat

(d)  Promoting free education to all children below the age of 14 years and assuring them jobs after obtaining higher education.

Answer: (b)

45. Who is the author of “Crime & Punishment”?

(a)  Geoffrey Chaucer

(b)  Fyodor Dostoevsky

(c)  Harold Joseph Laski

(d)  Karl Marx

Answer: (b)

46. Power to summon the Houses of the Parliament is vested with:

(a)  Vice President

(b)  President

(c)  Prime Minister

(d)  Speaker and Vice President

Answer: (b)

47. Which country is the first developed nation to default on debt of IMF?

(a)  USA

(b)  UK

(c)  Japan

(d)  Greece

Answer: (d)

48. Which State of India was declared in January, 2016 as the “first organic farming State of India”?

(a)  Punjab

(b)  Sikkim

(c)  Kerala

(d)  Haryana

Answer: (b)

49. Punjabi has become the ______________ most common language in the Parliament of Canada.

(a)  fourth

(b)  fifth

(c)  second

(d)  third

Answer: (d)

50. Which State has become the first State to introduce compulsory gender education at the graduate level?

(a)  Telangana

(b)  Kerala

(c)  Punjab

(d)  Andhra Pradesh

Answer: (a)

51. The highest peace time gallantry award Ashok Chakra was awarded posthumously during 2016 to:

(a)  Mohan Nath Goswami

(b)  Mahendra Singh

(c)  Jagdish Chand

(d)  Rajesh Atra

Answer: (a)

52. The Constitution (One Hundredth Amendment) Act, 2015 was enacted to give effect to:

(a)  the transfer of certain territories by India to Bangladesh and transfer of certain territories from Bangladesh to India

(b)  the acquiring of territories by India from Bangladesh.

(c)  National Judicial Appointment Commission

(d)  transfer of certain territories by India to Bangladesh.

Answer: (a)

53. The Currency of Thailand is:

(a)  Baht

(b)  Rupee

(c)  Ringgit

(d)  Peseta

Answer: (a)

54. Parliament of India consist of:

(a)  Council of States

(b)  House of the People

(c)  House of the People and Council of States

(d)  President, House of the People and Council of States

Answer: (d)

55. The direction to hold floor test to prove majority in the Legislative Assembly of Uttarakhand , to be held on 10th May, 2016 has been give n on 6th May, 2016 , by

(a)  The Supreme Court of India

(b)  The High Court of Uttarakhand

(c)  The President of India

(d)  The Governor of Uttarakhand

Answer: (a)

56. Which organization has the motto ‘Be Prepared’?

(a)  National Disaster Management Authority

(b)  National Cadet Corps

(c)  Boys’ Scout

(d)  National Service Scheme

Answer: (c)

57. Under the Constitution of India the official language of the Union is:

(a)  English and Hindi

(b)  English

(c)  Hindi

(d)  Hindi in Devanagari Script

Answer: (d)

58. At the Asian Indoor Athletics Championships held at Doha in February, 2016, who among women won the gold medal in Long Jump?

(a)  Irina Ektova of Khazagstan

(b)  Mayooka Johny of India

(c)  Prajusha of India

(d)  Olga Rypakova of Khazakstan

Answer: (b)

59. The “Paris Agreement” was adopted in the twenty first session of Conference of Parties in the month of:

(a)  December, 2015

(b)  January, 2016

(c)  November, 2015

(d)  February, 2016

Answer: (a)

60. Who was conferred the Rajiv Gandhi Khel Ratna award during

(a)  M. S. Dhoni

(b)  Virat Kohli

(c)  Saina Nehwal

(d)  Sania Mirza

Answer: (d)

61. Who is the President of Sri Lanka?

(a)  Jayavardane

(b)  K. Sripavan

(c)  Ranil Wickramasinghe

(d)  Maithripala Sirisena

Answer: (d)

62. Who has scored the fastest century in a Test Cricket match?

(a)  A B de Villiers, South Africa

(b)  Misbah Ul Haq, Pakistan

(c)  Brendon McCullum, New Zealand

(d)  Vivian Richards, West Indies

Answer: (c)

63. Who is the Chairperson of NITI Aayog?

(a)  Ms. Sushma Swaraj

(b)  Sh. Narendra Modi

(c)  Sh. Arun Jately

(d)  Ms. Sushmita Mahajan

Answer: (b)

64. Who is the Chairperson of National Human Rights Commission?

(a)  Justice B.S. Chauhan

(b)  Justice Balakrishnan

(c)  Justice D.K. Jain

(d)  Justice H.L. Dattu

Answer: (d)

65. Who was the Constitutional Advisor to the Constituent Assembly in framing the Indian Constitution?

(a)  Dr. Rajendra Prasad

(b)  Dr. B.R. Ambedkar

(c)  Sir B.N. Rau

(d)  Pandit Jawaharlal Nehru

Answer: (c)

66. In April 2016, The Duke and Duchess of Cambridge visited India. Their names are:

(a)  Prince Robert and Kateler

(b)  Prince Charles and Cathy

(c)  Prince William and Kate Middleton

(d)  Prince Hillery and Victoria.

Answer: (c)

67. In case the President of India wants to resign, he shall address his resignation to the:

(a)  Chief Justice of India

(b)  Vice President

(c)  Chief Election Commissioner

(d)  Prime Minister

Answer: (b)

68. Name the Indian Gymnast who has become the first female Indian Gymnast to qualify for the Olympics.

(a)  Dipa Karmakar

(b)  Anjubala

(c)  Bisweshwari Nandi

(d)  Rupinder Kaur

Answer: (a)

69. In case of death of both the President and Vice-President of India, who shall act as the President of India?

(a)  Prime Minister

(b)  The Parliament will nominate from among its Members

(c)  Chief Justice of India

(d)  Attorney General of India

Answer: (c)

70. Who is the Union Finance Minister of India?

(a)  Mr. D.V. Sadananda Gowda

(b)  Ms. Sushma Swaraj

(c)  Mr. Arun Jaitly

(d)  Mr. Rajnath Singh

Answer: (c)

71. Who is the Chairman of National Green Tribunal?

(a)  Justice Balakrishna Reddy

(b)  Justice R.C. Lahoti

(c)  Justice Swatanter Kumar

(d)  Justice Bhandhari

Answer: (c)

72. Which State has the largest number of foreign students in India?

(a)  Uttar Pradesh

(b)  Maharashtra

(c)  Haryana

(d)  Karnataka

Answer: (d)

73. Who lit the torch from the sun’s rays reflected in a parabolic mirrorduring the Olympic flame lighting ceremony for the Rio 2016 Olympic Games at the site of ancient Olympia in Greece on 22nd April, 2016?

(a)  Katerina Lehou

(b)  Ronaldo

(c)  Katerina Kaif

(d)  Joanie Laureh aka Chyna

Answer: (a)

74. The Parliament of which country became first Parliament in the world to run entirely on Solar Power?

(a)  UK

(b)  India

(c)  Pakistan

(d)  Japan

Answer: (c)

75. Prime Minister, Mr. Narendra Modi, in March, 2016, launched an ambitious programme “Setu Bharatam”. The programme is aimed at:

(a)  Linking all major cities and towns by rail, air and road by 2019.

(b)  Making all National Highways railway level crossing free by 2019

(c)  Providing travel concession to senior citizens throughout India.

(d)  Providing travel concession to women and children throughout India

Answer: (b)

76. How many Indian Universities/Institutes figure among the top 200 in BRICS rankings?

(a)  6

(b)  16

(c)  26

(d)  20

Answer: (b)

77. Which day is celebrated as “World Consumer Rights Day”?

(a)  15th March

(b)  5th September

(c)  5th June

(d)  15th July

Answer: (a)

78. Who was crowned as the winner of Femina Miss India 2016 and who will represent India at the Miss World pageant?

(a)  1Sushruthi Krishna

(b)  Pankhuri Gidwani

(c)  Sushobita Kapoor

(d)  Priyadarshini Chatterjee

Answer: (d)

79. __________ is the fastest planet to revolve around the Sun.

(a)  Mercury

(b)  Mars

(c)  Jupiter

(d)  Venus

Answer: (a)

80. Who has written the book “Indomitable Spirit”?

(a)  Dr. A.P.J. Abdul Kalam

(b)  Justice Krishna Iyer

(c)  Ms. Arundhati Roy

(d)  Sh. Narendra Modi

Answer: (a)

81. The Rajya Sabha in April, 2016 passed two Bills, which had already been passed by the Lok Sabha earlier, repealing certain outdated/old laws. The Bills intended to repeal:

(a)  315 laws

(b)  1053 laws

(c)  153 laws

(d)  513 laws

Answer: (b)

82. “Beyond the Lines – An Autobiography” is authored by:

(a)  Mr. Arun Shourie

(b)  Mr. Kuldip Nayar

(c)  Justice Krishna Iyer

(d)  Mr. Soli Sarab Ji

Answer: (b)

83. The maximum number of ‘Smokeless Villages’ are found in the State of:

(a)  Punjab

(b)  Rajasthan

(c)  Telangana

(d)  Karnataka

Answer: (d)

84. In an attempt to curb black money, the Government has made PAN mandatory for all financial transactions exceeding Rupees:

(a)  4 lakhs

(b)  3 lakhs

(c)  2 lakhs

(d)  1 l akhs

Answer: (c)

85. In which of the following States / Union Territories, the Election Commission has decided to hold election in a single phase on May 16, 2016?

(a)  Assam, Kerala and Pondicherry.

(b)  West Bengal, Kerala and Tamil Nadu.

(c)  Kerala, Tamil Nadu and Pondicherry.

(d)  West Bengal, Kerala and Assam.

Answer: (c)

86. Name the country that has six Deputy Prime Ministers

(a)  Nigeria

(b)  Nicaragua

(c)  Nepal

(d)  Maldives

Answer: (c)

87. Country’s first ‘visually challenged friendly’ railway station is:

(a)  New Delhi

(b)  Varanasi

(c)  Tirupathi

(d)  Mysuru

Answer: (d)

88. The largest diaspora in the world is from:

(a)  Mexico

(b)  South Africa

(c)  India

(d)  China

Answer: (c)

89. A Japanese maglev, which is the fastest passenger train in the world, has broken its own record in 2015. The train reached ___________ kmph in the test run.

(a)  503

(b)  403

(c)  603

(d)  453

Answer: (c)

90. In January, 2016, at the first stage, the Union Ministry of Urban Development unveiled the list of ________ cities for smart cities mission out of 98 shortlisted cities:

(a)  30

(b)  15

(c)  20

(d)  25

Answer: (c)

Section : Elementary Mathematics ( Numerical Ability )

Direction for Questions 91-110: Choose the most appropriate option:

91. The number of ‘three digit numbers’ which are multiples of 9 are:

(a)  98

(b)  101

(c)  100

(d)  99

Answer: (c)

92. The value of a machine depreciates every year at the rate of 10% on its value at the beginning of that year. If the present value of the machine is Rs. 729, its worth three years ago was:

(a)  Rs. 947.70

(b)  Rs. 1,000

(c)  Rs. 750.87

(d)  Rs. 800

Answer: (b)

93. The angle subtended by the Minor segment of a circle at the center is __________ the angle subtended by the Major segment at the center of the circle.

(a)  greater than

(b)  not related to

(c)  equal to

(d)  lesser than

Answer: (d)

94. What is the sum of all the natural numbers from 1 to 100?

(a)  5000

(b)  5050

(c)  6000

(d)  5052

Answer: (b)

95. A part of monthly hostel charges is fixed and the remaining depends on the number of days one has taken food in the mess. When a student A takes food for 20 days, she has to pay Rs. 1000 as hostel char ges whereas a student B, who takes food for 26 days, pays Rs. 1180 as hostel charges. Find the fixed charges and the cost of food per day.

(a)  300, 30

(b)  400, 40

(c)  200, 20

(d)  400, 30

Answer: (d)

96. A library has an average of 510 visitors on Sundays and 240 on other days. What is the average number of visitors per day in the month of June beginning with a Sunday?

(a)  250

(b)  280

(c)  285

(d)  276

Answer: (c)

97. 12 defective pens are accidentally mixed with 132 good ones. It is not possible to just look at a pen and tell whether or not it is defective. One pen is taken out at random from this lot. Determine the probability that the pen taken out is a good one.

(a)  7/12

(b)  11/12

(c)  10/12

(d)  9/12

Answer: (b)

98. A man earns Rs. 20 on the first day and spends Rs. 15 on the next day. He again earns Rs. 20 on the third day and spends Rs. 15 on the fourth day. If he continues to save in this way, how soon will he have Rs. 60 in hand?

(a)  on 27th day

(b)  on 24th day

(c)  on 12th day

(d)  on 17th day

Answer: (d)

99. 60 kg of an alloy X is mixed with 100 kg of an alloy Y. If alloy X has lead and tin in the ratio of 3:2 and alloy Y has tin and copper in the ratio of 1:4, then the amount of tin in the new alloy is

(a)  53 kgs.

(b)  80 kgs.

(c)  36 kgs.

(d)  44 kgs.

Answer: (d)

100. Two consecutive even positive integers, sum of the squares of which is 1060 are:

(a)  12 and 14

(b)  22 and 24

(c)  20 and 22

(d)  16 and 18

Answer: (b)

101. The traffic lights at three different signal points change after every 45 seconds, 75 seconds and 90 seconds respectively. If all change simultaneously at 7:20:15 hours, then they will change again simultaneously at

(a)  7:27:30 hours

(b)  7:28:00 hours

(c)  7:27:50 hours

(d)  7:27:45 hours

Answer: (d)

102. A circular park, 42 m in diameter, has a path 3.5 m wide running around it on the outside. Find the cost of gravelling the path at Rs. 4 per m2

(a)  Rs 1672

(b)  Rs 1652

(c)  Rs 2002

(d)  Rs 2048

Answer: (c)

103. A train which is moving at an average speed of 40 kmph, reaches its destination on time. When its average speed reduces to 35 kmph, then it reaches its destination 15 minutes late. The distance travelled b y the train is:

(a)  80 kms

(b)  40 kms

(c)  70 kms

(d)  30 kms

Answer: (c)

104. The mean of 72 items was found to be 63. If two of the items were misrecorded as 27 and 9 instead of 72 and 90 respectively, find the correct mean.

(a)  64.75

(b)  64.25

(c)  63.25

(d)  65.75

Answer: (a)

105. A man buys Rs. 20 shares paying 9% dividend. The man expects to have an interest of 12% on his money. The market value of each share is

(a)  Rs 18

(b)  Rs 15

(c)  Rs 12

(d)  Rs 21

Answer: (b)

106. A patient in a hospital is given soup daily in a cylindrical bowl of dia meter 7 cm. If the bowl is filled with soup to a height of 4 cm, how much soup the hospital has to prepare daily to serve 250 patients?

(a)  40 litres

(b)  38 litres

(c)  38.5 litres

(d)  39.5 litres

Answer: (c)

107. The angles between the hands of a clock when the time is 4:25 am is:

(a) 

(b) 

(c) 

(d) 

Answer: (c)

108. A shop gives 15% discount on the purchase of a T.V. If paid for in cash immediately, a further discount of 12% is given. If the marked price is Rs. 15,000, what is the price of the T.V if cash purchase is made?

(a)  Rs. 12,750

(b)  Rs. 11,220

(c)  Rs. 10,950

(d)  Rs. 11,475

Answer: (b)

109. If a boy is standing at the top of the tower and another boy is at the ground at some distance from the foot of the tower, then the angle of elevation and depression between the boys when both look at each other will be

(a)  Angle of elevation will be greater

(b)  Cannot be predicted for relation

(c)  Angle of depression will be greater

(d)  Equal

Answer: (d)

110. If the product of zeroes of the polynomial ax2 – 6x – 6 is 4, find the value of ‘a’

(a)  −3/2

(b)  −1/2

(c)  3/2

(d)  1/2

Answer: (a)

Section : Legal Aptitude

This section consists of Fifty (50) questions. Follow the instructions carefully and choose the most appropriate option:

The question number 111-145 in this section consist of legal proposition(s) (hereinafter referred to as ‘principle’) and facts. Such principles may or may not be true in the real and legal sense, yet you have to conclusively assume them to be true for the purpose of this section. In other words, in answering the following question. Further, you must not assume any facts other than those stated in the question. The objective of this section is to test your interest towards study of law, research aptitude and problem solving ability, even if the ‘most reasonable conclusion’ arrived at may e absurd or unacceptable for any other reason.

To answer a question, the given principle is to be applied to the given facts so as to arrive at most reasonable conclusion.

111. Principle : Intentional application of force to another person is action able in law.

Facts : ‘P’ and ‘D’ are unknown to each other. When ‘P’ is about to sit on a chair, ‘D’ intentionally pulls it away as a result of which ‘P’ falls on the floor and is injured.

(a)  ‘D’ is not liable as ‘P’ is not seriously injured.

(b)  ‘D’ is liable as he intentinally caused injury to P.

(c)  ‘D’ is not liable as such jokes are common in the society.

(d)  ‘D’ is not liable as the injury is not directly caused.

Answer: (b)

112. Principle :

1. Wagering agreement are void.

2. Collateral agreement to wagering contracts are valid.

Facts : XYZ Bank lends Rs. 40,000 to Sabu in order to enable him to award as prize to Randeep who is the winner of horse race. Later Sabu refused to pay the prize stating that horse racing is wagering agreement. Can XYZ Bank recover money from Sabu.

(a)  Yes it is only a collateral agreement to horse racing and there fore the bank can recover the money from Sabu.

(b)  Horse racing is illegal and therefore XYZ Bank cannot recover anything from Sabu.

(c)  No, as it is as wagering contract.

(d)  Bank can recover money from Sabu so that payment of prize money can be made to Randeep.

Answer: (a)

113. Principle : Where a person lawfully does anything for another person, or delivers anything to him, not intending to do so or to provide gratuitously, and such other person takes the benefits of that; the latter is bound to compensate the former for something done or thing provided, or ot restore, the thing so delivered.

Facts: Trader ‘A’ delivers certain eatables at ‘B’s house by mistake. ‘B’ consumed the eatables without asking anything. Which of the following derivations is correct?

(a)  ‘B’ is bound to pay ‘A’ for the eatables.

(b)  ‘B’ can be made liable to pay for the eatables, only if ‘A’ establish as an express contract between ‘A’ and ”B’.

(c)  ‘B’ is not bound to pay ‘A’ for the eatables.

(d)  It is the discretion of ‘B’ to make payment to ‘A’

Answer: (a)

114. Principle : Consent is a good defence in a civil action for tort but the act should be the same for which consent was given.

Facts : ‘B’ was formally invited by ‘A’ to his house. ‘B’ after sitting for some time in drawing room, moved to the bed room of the house. ‘A’ sued ‘B’ for trespass.

(a)  ‘B’ has interfered with privacy of ‘A’.

(b)  ‘B’ has committed no trespass as he entered the house with ‘A’s consent.

(c)  ‘B’ has offended ‘A’ by moving to bed room.

(d)  ‘B’ has committed trespass as there was no consent of ‘A’ for entry in the Bed room.

Answer: (d)

115. Principle : Whoever takes away any moveable thing form the land of any person without that person’s consent, he is said to have committed theft.

Facts : During his visit to the house of ‘C’, ‘A’ asked ‘B’ the son of ‘C’, to accompany ‘A’ to the forest.

Neither ‘A’ nor ‘B’ informed ‘C’ in this regard. ‘B’ accompanied ‘A’ to the forest.

(a)  ‘A’ has committed theft as soon as he entered the house of ‘C’.

(b)  ‘A’ has not committed theft.

(c)  ‘A’ has committed theft.

(d)  ‘A’ has not committed theft till ‘B’ did not accompany him.

Answer: (b)

116. Principle: The communication of a proposal is complete when it comes to the knowledge of the person to whom it is made.

Facts : ‘A’ sent a letter making a proposal to ‘B’ to purchase the house of ‘B’.

(a)  The communication of proposal is complete when ‘B’ reads the letter.

(b)  The communication of proposal is complete when A sent the letter.

(c)  The communication of proposal is complete when B’s wife received it.

(d)  The communication of proposal is complete when B’s wife handed over the letter to ‘B’.

Answer: (a)

117. Principle: Law does not penalise for wrongs which are of trivial nature.

Facts: In the course of a discussion, ‘A’ threw a file of papers at the t able which touched the hands of ‘B’.

(a)  ‘A’ is liable for his act, as the file touched ‘B’s hand.

(b)  ‘A’ is liable for his act, as it assaulted ‘B’.

(c)  ‘A’ is liable for insulting ‘B’.

(d)  ‘A’ is not liable for his act, as it was of trivial nature.

Answer: (d)

118. Principle: Copyright law protects only work. ‘Work’ means cinemato graphic film but does not include performance by an actor in a cine matographic film.

Facts: Alia Bhatt acted in a movie

(a)  The acting of Alia Bhatt can be protected under copyright law as professional work.

(b)  The acting of Alia Bhatt cannot be protected under copyright law.

(c)  The acting of Alia Bhatt can be protected under copyright law only as an artistic work

(d)  The acting of Alia Bhatt can be protected as film producer’s work.

Answer: (b)

119. Principle: Import means bringing some consignment into India from a foreign country.

Facts: A consignment from Sri Lanka entered the territorial waters of India. However, this consignment never crossed the Indian custom barrier nor did it enter into the stream of commerce in India.

(a)  The consignment will only be imported into India when it enters into the stream of commerce in India.

(b)  The consignment was not imported into India.

(c)  The consignment was imported into India.

(d)  The consignment was imported into India.

Answer: (c)

120. Principle: A person is said to have committed assault when an apprehension is caused in the mind of a person that he is about to use physical force against his body.

Facts: ‘A’ abuses ‘B’ while he was sitting in a moving train, by aggressively shaking his fists when ‘B’ was standing on the railway platform at a distance.

(a)  ‘A’ has caused apprehension of assault in the mind of ‘B’

(b)  ‘A’ has committed assault against ‘B’.

(c)  ‘A’ has not committed assault against ‘B’.

(d)  ‘A’ has caused fear of assault in the mind of ‘B’.

Answer: (c)

121. Principle: Sale of liquor is illegal. All agreements relating to prohibit ed items do not exist in the eyes of law.

Facts: ‘A’ entered into an agreement with ‘B’ for the sale of liquor. ‘A’ failed to supply the agreed quantity of liquor to ‘B’.

(a)  ‘B’ can bring a legal action against ‘B’.

(b)  ‘A’ can bring a legal action against ‘B’.

(c)  ‘B’ cannot bring any legal action against ‘A’.

(d)  ‘A’ and ‘B’ can initiate appropriate legal proceeding against each other.

Answer: (c)

122. Principle: Nothing is an offence which is done by a child under twelve years of age, who has not attained sufficient maturity of understanding to judge the nature and consequences of his conduct on that occasion.

Facts: Himesh, 11 years old boy, picks up a gold ring worth Rs 5000/-lying on a table in his friend’s house and immediately sells it for Rs 2000/-, and misappropriates the money.

(a)  Himesh would not be protected under the principle stated above because, irrespective of the age, stealing is an offence.

(b)  Himesh would be protected under the principle stated above because he is below 12 years of age.

(c)  Himesh would be protected under the principle stated above because his acts show that he was not sufficiently mature to understand the nature and consequences of his conduct.

(d)  Himesh would not be protected under the principle stated above because his acts show that he was sufficiently mature to understand the nature and consequences of his conduct.

Answer: (d)

123. Principle: One who dishonestly misappropriates or converts to his own use or sells any movable property belonging to another, is guilt y of the offence of misappropriation.

Facts: ‘A’ takes property belonging to ‘Z’ out of Z’s possession, in good faith, believing when he takes it, that the property belongs to himself. Subsequently, ‘A’, on discovering his mistake, without disclosing the actual facts, dishonestly sells the property to a stranger.

(a)  ‘A’ may be guilty of theft but not for misappropriation.

(b)  ‘A’ is guilty of an offence of misappropriation.

(c)  ‘A’ is not guilty because when he took the property, he believed in good faith that it belonged to him.

(d)  ‘A’ is not guilty as the property can be recovered from the stranger.

Answer: (b)

124. Principle: Letters or words not describing quality of things can be registered as a trade mark.

Facts: Ram made an application for registration of alphabet ‘B’ written in a fancy style as trade mark to be applied on packets and cartons of shoes manufactured by him.

(a)  The alphabet ‘B’ can be registered as trade mark because it describes the quality of things.

(b)  The alphabet ‘B’ cannot be registered as trade mark because it is an English letter.

(c)  The alphabet ‘B’ can be registered as trade mark.

(d)  The alphabet ‘B’ cannot be registered as trade mark because it belongs to humanity.

Answer: (c)

125. Principle: Defamation is the publication of a statement which tends to lower reputation of a person in the estimation of other members of the society generally.

Facts: ‘A’ writes a highly offensive and derogatory letter about ‘B’, and sends it directly to ‘B’ in a sealed cover.

(a)  ‘A’ is liable to ‘B’ for defamation, as the letter is highly offensive and derogatory and is directly sent to ‘B’.

(b)  A’ is not liable to ‘B’ for defamation, since there is no publication to any other person in whose estimation the reputation of ‘B’ could be brought down.

(c)  ‘A’ is liable to ‘B’ for defamation, as the letter is highly offensive and derogatory.

(d)  ‘A’ is liable to ‘B’ for defamation, as it has hurt his (B’s) self-esteem.

Answer: (b)

126. Principle: Existence of all the alleged facts is relevant, whether they occurred at the same time and place or at different times and places.

Facts: ‘A’, a citizen of England, is accused of committing murder of ‘B’ in India by taking part in a conspiracy hatched in England.

(a)  Only the fact that ‘A’ citizen of England is accused of committing murder of ‘B’ in India is relevant.

(b)  ‘A’ citizen of England cannot be tried in India.

(c)  Only the fact that ‘A’ is accused of conspiracy hatched in England is relevant.

(d)  The facts that ‘A’ citizen of England is accused of commission of murder in India and of conspiracy hatched in England are relevant facts.

Answer: (d)

127. Principle: An agreement without free consent can be enforces only at the option of the party whose consent was not free.

Facts: A obtains the consent of ‘B’ to enter into an agreement by put ting a gun on the head of B’s girl friend.

(a)  ‘B’ can enforce the agreement.

(b)  ‘B’ cannot enforce the agreement.

(c)  Neither ‘A’ nor ‘B’ can enforce the aggrement.

(d)  ‘A’ can enforce the agreement.

Answer: (a)

128. Principle: Acceptance of proposal must be the exact mirror image of the proposal.

Facts: ‘A’ made a proposal to ‘B’ to sell a chair for Rs. 500. ‘B’ expressed his desire to buy the said chair for Rs. 400.

(a)  It is not clear whether A made a proposal to ‘B’.

(b)  ‘B’ has accepted the proposal of ‘A’.

(c)  ‘B’ has not accepted the proposal of ‘A’.

(d)  It is not clear whether ‘B’ has accepted the proposal of ‘A’ or not.

Answer: (c)

129. Principle: Mere silence as to facts lakely to affect the decision of a person to enter into a contract is not fraud.

Facts: ‘A’ sells to ‘B’ (A’s daughter who is minor) a horse which ‘A’ knows to be unsound. ‘A’ says nothing to ‘B’ about the unsoundness of the horse.

(a)  ‘B’ can take plea of fraud because she is minor.

(b)  There can not be a contract between a father and daughter.

(c)  ‘A’ has not committed fraud.

(d)  ‘A’ has committed fraud.

Answer: (c)

130. Principle: A person, who is usually of unsound mind, but occasionally normal, may make a contract when he is not of unsound mind.

Facts: ‘A’ generally remains in the state of unsound mind and rarely becomes capable of understanding the things.

(a)  ‘A’ can make a contract when normal.

(b)  ‘A’ can make a contract only for his own benefit.

(c)  ‘A’ can never make a contract.

(d)  ‘A’ can make a contract at any time whenever he pleases.

Answer: (a)

131. Principle: In case where there is an infringement of legal right even without any actual loss or damage, the person whose right is Infringed has a cause of action.

Facts: ‘P’ was wrongfully prevented by the Returning officer from exercising his vote in an assembly election. Still he (‘P’) brought an action claiming damages. Which of the following derivations is correct?

(a)  ‘P’ would succeed in his action, as it is mandatory to cast vote.

(b)  ‘P’ would succeed in his action, as he was wrongfully prevented from exercising his legal right of voting in that election.

(c)  ‘P’ would not succeed in his action, as he did not suffer any loss in that election.

(d)  ‘P’ would not succeed in his action, as the candidate for whom he wanted to give his vote won the election.

Answer: (b)

132. Principle: There are certain acts which, though harmful, are not wrongful in law; therefore, do not give legal right to bring action in law, to the person who suffers from such acts.

Facts: ‘Prakash’ has a rice mill. His neighbour, Shanti, sets up another rice mill and offers a tough competition to Prakash. As a consequence, Prakash’s profits fall down. He brings a suit against Shanti for damages.

(a)  Prakash can succeed in his claim as it is a case of actual damages.

(b)  Prakash cannot succeed in his claim for damages, as it is a case of damage without infringement of any legal right.

(c)  Prakash may succeed in his claim for damages, as it is a case of loss to his business.

(d)  Prakash can succeed in his claim for damages, as it is a case of damage as a result of infringement of his legal right.

Answer: (b)

133. Principle: A condition to a contract can also be complied with after the happening of the event to which such a condition is attached.

Facts: ‘A’ promises to pay Rs. 5000 to ‘B’ on the condition that he shall marry with the consent of ‘C’, ‘D’ and ‘E’. ‘B’ marries without the consent of ‘C’, ‘D’ and ‘E’, but obtains their consent after the marriage.

(a)  ‘B’s marriage is not valid.

(b)  ‘B’ has not fulfilled the condition.

(c)  The condition is illegal

(d)  ‘B’ has fulfilled the condition.

Answer: (d)

134. Principle: Killing is not murder if the offender, whilst deprived of the power of self-control by intense and sudden provocation, causes the death of the person who gave the provocation.

Facts: ‘A’, a man found his girl friend sleeping, in her own bed room, with another man named ‘B’. ‘A’ did not do anything but went to his home, picked a gun and cartridges, returned to the girl friend’s bed room with loaded gun but found the place empty. After fifteen days he saw his girl friend dining in a restaurant. Without waiting for even a second, ‘A’ fired five bullets at his girl friend who died on the spot.

(a)  ‘A’ could have killed ‘B’ instead of his girl friend.

(b)  ‘A’ could have killed both ‘B’ and his girl friend.

(c)  ‘A’ did not kill his girl friend under intense and sudden provocation.

(d)  ‘A’ killed his girl friend under intense and sudden provocation.

Answer: (c)

135. Principle: Whoever by words or writing conveys to others any imputation concerning any person’s reputation is said to defame that person.

Facts: During a marriage ceremony, ‘A’ circulated a pamphlet saying that ‘S’, sister of the bride, is a thief, she has stolen the shoes of the bridegroom.

(a)  ‘A’ has defamed the bridegroom.

(b)  ‘A’ defamed the bride.

(c)  ‘A’ has defamed ‘S’.

(d)  ‘A’ did not defame ‘S’ as he never intended it.

Answer: (c)

136. Principle: Causing of an effect partly by an act and partly by an omission is an offence.

Facts: ‘A’ confined her daughter ‘D’ in a room. ‘A’ also did not provide any food to her daughter ‘D’.

Consequently, ‘D’ died of starvation.

(a)  ‘A’ committed the offence of confining ‘D’.

(b)  ‘A’ committed the offence of causing death of ‘D’.

(c)  ‘A’ committed no offence.

(d)  ‘A’ committed the offence of not providing food to ‘D’.

Answer: (b)

137. Principle: Whoever does not arrest the killer and report the matter to the concerned authorities commits an offence.

Facts: ‘A’, a woman, sees ‘B’ , another woman, killing a third woman ‘C’. ‘A’ neither attempted to arrest ‘B’ nor informed the concerned authorities.

(a)  ‘A’ has not committed an offence.

(b)  ‘A’ has committed an offence.

(c)  ‘B’ has not committed an offence.

(d)  ‘B’ has committed an offence.

Answer: (b)

138. Principle: False imprisonment is a tort (wrong) which means the total restraint of a person’s liberty without lawful justification.

Facts: A part of a public road had been closed for spectators of a boat race. ‘P’ wanted to enter but he was prevented by ‘D’ and other policemen because he had not paid the admission fee. ‘P’ was able to enter the enclosure by other means but was unable to go where he wanted to go. The policemen refused access to where he wanted to go but allowed him to remain where he was or to go back. ‘P’ remained within the enclosure and refused to leave. Subsequently, ‘P’ sued ‘D’ for false imprisonment.

(a)  It was a case of false imprisonment, but ‘D’ could not be made liable for it.

(b)  ‘D’ could not be made liable for false imprisonment as he has not touched him.

(c)  ‘D’ could be made liable for false imprisonment, as he did restrict P’s movements.

(d)  ‘D’ could not be made liable for false imprisonment, as he did not totally restrict P’s movements.

Answer: (d)

139. Principle: An independent contractor is one who is employed to do some work of his employer. He is engaged under a contract for services. He undertakes to produce a given result, and in the actual execution of the work, he is not under the direct control or following directions of hi s employer. He may use his own discretion in execution of the work assigned.

In general, an employer is not liable for the torts (wrongful acts) of his independent contractor. But, the employer may be held liable if h e directs him to do some careless acts.

Facts: Ramesh hired a taxi-cab to go to Delhi Airport. As he started l ate from his home, he kept on urging the taxi-driver to drive at a high speed and driver followed the directions; and ultimately due to high speed an accident took place causing injuries to a person.

(a)  Ramesh would be held liable for damages as he exercised the control by giving directions to the driver.

(b)  Ramesh would not be held liable for damages because the drive r was an independent contractor and not his servant.

(c)  Ramesh would not be held liable for damages because Ramesh did not know the consequences of such rash driving.

(d)  Ramesh would not be liable as car was not owned by him.

Answer: (a)

140. Principle: Nothing is an offence, which is done by accident or misfortune, and without any criminal intention or knowledge in the doing of a lawful act in a lawful manner by lawful means and with proper car e and caution.

Facts: ‘A’ takes up a gun, not knowing whether it is loaded or not, points it playfully at ‘B’ and pulls the trigger. Consequently, ‘B’ falls dead.

(a)  ‘B’s death is not accidental, as there was want of proper care and caution on the part of ‘A’.

(b)  ‘B’s death is accidental, as ‘A’ had no intention to kill ‘B’.

(c)  ‘B’s death is accidental, as ‘A’ was just pointing the gun playfully at ‘B’.

(d)  ‘B’s death is accidental, as ‘A’ did not have the knowledge that the gun is loaded.

Answer: (a)

141. Principle: An agreement may be entered into orally or in writing, or by conduct.

Facts: ‘A’ went to the shop of ‘B’ and picked a toothbrush and gave a cheque of Rupees twenty to ‘B’ and left the shop.

(a)  Payment of toothbrush cannot be made through a cheque.

(b)  ‘A’ did not enter into an agreement with ‘B’.

(c)  ‘A’ should have carried a currency note of Rupees twenty to make the payment.

(d)  There was an agreement between ‘A’ and ‘B’.

Answer: (d)

142. Principle: Law never enforces an impossible promise.

Facts: ‘A’ made a promise to ‘B’ to discover treasure by magic.

(a)  Law will not enforce the promise.

(b)  Law will enforce the promise.

(c)  Law will enforce the promise only at the option of ‘A’.

(d)  Law will enforce the promise only at the option of ‘B’.

Answer: (a)

143. Principle: Consent is a good defence for civil action in tort. But consent must include both knowledge of risk and assumption of risk, i.e, readiness to bear harm.

Facts: A lady passenger was aware that the driver of the cab, in which she opted to travel was little intoxicated. The cab met with an accident and lady got injured.

(a)  Driver can take the plea that he was lightly intoxicated.

(b)  Lady is not entitled to claim compensation as she had knowledge of the risk.

(c)  Lady is entitled to claim compensation as she only knew about risk and there was no assumption of risk.

(d)  Lady can refuse to pay the fare as she had suffered injuries.

Answer: (c)

144. Principles:

• A person is said to abet the doing of a thing when he instigates any other person to do that thing.

• Mere acquiescence, however, does not amount to instigation.

Facts: ‘A’ says to ‘B’: I am going to kill ‘C’.” And, ‘B’ replies: “Do as y ou wish and take the consequences”; whereafter ‘A’ kills ‘C’.

(a)  ‘B’ has not abetted ‘A’ to kill ‘C’.

(b)  ‘B’ has abetted ‘A’ by conspiracy.

(c)  ‘B’ abetted ‘A’ to kill ‘C’.

(d)  ‘B’ is jointly liable with ‘A’ for killing ‘C’.

Answer: (c)

145. Principles:

• A servant is one who is employed to do some work for his employ er (master). He is engaged under a contract of service. He works directly under the control and directions of his master.

• In general, the master is vicariously liable for those torts (wrongful acts) of his servant which are done by the servant in the course of his employment.

Facts: ‘M’ appointed ‘D’ exclusively for the purpose of driving his tourist vehicle. ‘M’ also appointed ‘C’ exclusively for the purpose of performing the work of a conductor for the tourist vehicle. During one trip, at the end of the journey, ‘C’, while ‘D’ was not on the driver’s seat, and apparently for the purpose of turning the vehicle in the right direction for the next journey, drove it through the street at high speed, and negligently injured ‘P’.

(a)  ‘M’ could be made liable for the act of ‘C’, as his (C’s) act of driving the vehicle was within his scope of employment.

(b)  ‘M’ is not liable as he was not present at the time of accident.

(c)  ‘M’ could not be made liable for the act of ‘C’, as his (C’s) act of d riving the vehicle was not in the course of his employment.

(d)  ‘M’ could be made liable for the act of ‘C’, as ‘C’ was employed nder a contract of service.

Answer: (c)

Direction for Questions 146 to 160: Choose the most appropriate option:

146. ‘alibi’ means a plea by an accused person that he –

(a)  was facing trial.

(b)  was present elsewhere

(c)  underwent preventive detention.

(d)  remained in judicial custody.

Answer: (b)

147. Under the Constitution of India restriction on freedom of religion can not be placed on the ground of –

(a)  Public order

(b)  Social justice

(c)  Morality

(d)  Health

Answer: (b)

148. If an authority is holding information about another in a ‘fiduciary capacity’, the information under the Right to Information Act, 2005 may not be obtainable. ‘Fiduciary relationship’ is based on:

(a)  Authority

(b)  Law

(c)  Trust

(d)  Contract

Answer: (c)

149. Which one of the following is not a Directive Principle of State Policy under Part IV of the Constitution of India?

(a)  Promotion of adult education.

(b)  Promotion of International peace and security.

(c)  Organisation of village panchayats.

(d)  Provision for just and humane conditions of work and maternity relief.

Answer: (a)

150. ‘audi alteram partem’ means –

(a)  Not connected to facts.

(b)  Giving opportunity of hearing of the other side.

(c)  Following the substantive law.

(d)  A transferee cannot retransfer.

Answer: (b)

151. Which among the following was described by Dr. B. R. Ambedkar as the “heart and soul of the Constitution of India”?

(a)  Right to Constitutional Remedies

(b)  Right to equality

(c)  Freedom of Religion

(d)  Right to move throughout the territory of India

Answer: (a)

152. ‘obiter dicta’ means –

(a)  Direction by a judge.

(b)  Basis of judicial decision.

(c)  Judgment of a court in the case before it.

(d)  An opinion given by the court not necessary for the decision.

Answer: (d)

153. Under the Constitution of India ‘Right to Pollution Free Environment’ has emerged as a fundamental right from the right to-

(a)  Life and personal liberty under Article 21

(b)  Freedom of movement under Article 19

(c)  Conserve culture under Article 29.

(d)  Equality under Article 14

Answer: (a)

154. ‘persona non grata’ means –

(a)  Non-person

(b)  An unacceptable person

(c)  Non-performance of promise.

(d)  Ungrateful person

Answer: (b)

155. The object of which one of the following writs is to prevent a person to hold public office which he is not legally entitled to hold ?

(a)  Quo warranto

(b)  Mandamus

(c)  Prohibition

(d)  Certiorari

Answer: (a)

156. Which among the following does not belong to the ‘right to freedom of religion’?

(a)  Freedom of conscience and free profession, practice and propagation of religion.

(b)  Freedom from attending religious instruction or religious worship in certain educational institutions.

(c)  Freedom from payment of taxes for promotion of any particular religion .

(d)  Freedom of speech and expression .

Answer: (d)

157. ‘lis pendens’ means –

(a)  A pending suit

(b)  On the basis of evidence

(c)  Awaited information

(d)  Decision awaited.

Answer: (a)

158. Which Indian State has prescribed minimum educational qualification for candidates contesting panchayat polls?

(a)  Gujarat

(b)  Kerala

(c)  Punjab

(d)  Haryana

Answer: (d)

159. As per law the minimum age for the marriage of a boy and a girl in India is

(a)  21 years in both cases

(b)  18 years and 21 years respectively

(c)  21 years and 18 years respectively

(d)  18 years in both cases

Answer: (c)

160. The Supreme Court of India has struck down the Constitution (Ninety ninth Amendment) Act, 2014 as unconstitutional. It is related to –

(a)  National Judicial Appointment Commission

(b)  Religious Rights

(c)  Land Exchange between India and Bangladesh

(d)  Jallikattu (Bull Fighting)

Answer: (a)

Section : Logical Reasoning

Direction for Questions 161 – 163

Read the following information carefully and choose the most appropriate option:

A and B are good in driving bus and car. C and A are good in driving car and scooter. C, D and E are good in driving scooter and tractor. E and C are good in driving scooter and auto rickshaw. D and B are good in driving bus and tractor.

161. Who is good in driving scooter, tractor and auto rickshaw but not good in driving car?

(a)  A

(b)  E

(c)  D

(d)  C

Answer: (b)

162. Who is good in driving scooter, tractor and bus?

(a)  C

(b)  D

(c)  B

(d)  A

Answer: (b)

163. Who is good in driving tractor, scooter, car and autorickshaw but no t bus?

(a)  C

(b)  D

(c)  B

(d)  A

Answer: (a)

Direction for Questions 164 – 166

Read the following information carefully and choose the most appropriate option:

In each question, there is a statement and two assumptions numbered as I and II. Read the statement and find which of the given assumptions is implicit:

(A) If assumption I is implicit.

(B) If assumption II is implicit.

(C) If neither assumption I nor Assumption II is implicit.

(D) If both Assumption I and Assumption II are implicit.

164. Statement: The next meeting of the executive board of a company will be held after six months.

Assumptions:

I. Existing executive board will be dissolved before six months

II. The company will remain in function after six months

(a)  Neither assumption I nor Assumption II is implicit.

(b)  Assumption II is implicit.

(c)  Both Assumption I and Assumption II are implicit.

(d)  Assumption I is implicit.

Answer: (b)

165. Statement: In the State of Zuminisia, people prefer to travel by X airline instead of Y airline, as X airline has advanced German security system and 99% on time operations.

Assumptions:

I. Airline X with advanced German security system and record on time operation is perceived better than airline Y.

II. Had advanced German security system and on time performance record of Y airline been implemented, it would have been preferred over airline X.

(a)  Assumption I is implicit.

(b)  Neither assumption I nor Assumption II is implicit.

(c)  Assumption II is implicit.

(d)  Both Assumption I and Assumption II are implicit.

Answer: (a)

167. Statement: To attend a convocation ceremony scheduled to be held on Thursday at GM University, Chennai, Mr X left for Chennai on Tuesday by train.

Assumptions:

I. Mr X may reach home on Saturday

II. Mr X may reach the University on Wednesday

(a)  Assumption I is implicit.

(b)  Assumption II is implicit.

(c)  Both Assumption I and Assumption II are implicit.

(d)  Neither assumption I nor Assumption II is implicit.

Answer: (b)

Direction for Questions 167 – 168

Read the following information carefully and choose the most appropriate option:

In a joint family, A, B, C, D, E, and F are the members. B is the son of C. A and C are husband and wife. C is not the mother of B. E is the brother of C. D is the daughter of A and F is the brother of B.

167. Which of the following is a pair of females in the family?

(a)  AE

(b)  AD

(c)  DF

(d)  BD

Answer: (b)

168. Who is the mother of B?

(a)  D

(b)  F

(c)  A

(d)  E

Answer: (c)

Read the following information carefully and choose the most appropriate option:

In each of the questions, two statements are numbered as I and II. There may be a cause and effect relationship between the two statements. The statements may be the effect of the same cause or an independent cause. The statements may be independent causes without having any relationship. Read both the statements and mark your answer as:

(A) If statement I is the cause and statement II is its effect.

(B) If statement II is the cause and statement I is its effect.

(C) If both statements I and II are independent causes.

(D) If both statements I and II are effects of independent causes.

169. Statement I: In last two years, there is a considerable reduction in cancellation of flights due to fog in North India.

Statement II: In last two years, there is a considerable improvement in passenger amenities on all airports of North India.

(a)  Both statements I and II are effects of independent causes.

(b)  Statement I is the cause and statement II is its effect.

(c)  Statement II is the cause and statement I is its effect.

(d)  Both statements I and II are independent causes.

Answer: (a)

170. Statement I: The Government, by legislation has decided to make all public information available to general public.

Statement II: Before passing of legislation, general public did not have access to public information.

(a)  Both statements I and II are independent causes.

(b)  Statement I is the cause and statement II is its effect.

(c)  Statement II is the cause and statement I is its effect.

(d)  Both statements I and II are effects of independent causes.

Answer: (c)

Direction for Questions 171 – 200: Choose the most appropriate option:

171. Identify the statement which cannot be false.

(a)  Democracy is the best form of government.

(b)  Water evaporates at 100° C.

(c)  All radii of any given circle are of equal length.

(d)  Myopia is a congenital disorder.

Answer: (c)

172. An old woman decided to divide her gold among her daughter and daughters-in-law. She first kept exactly half of the gold for her daughter. Then she divided the rest of her gold among her daughters-in-law. The eldest one got 26 grams more than the youngest daughter-in-law. The middle one got twice as the youngest one. If the eldest daughter-in-law got 66 grams of gold, how much was received by the daughter.

(a)  198 grams

(b)  172 grams

(c)  186 grams

(d)  194 grams

Answer: (c)

173. Find the odd one out from the following group.

WINDSHIELD, SPARK PLUG, CLUTCH PEDAL, CAR, ENGINE

(a)  Car

(b)  Engine

(c)  Windshield

(d)  Spark Plug

Answer: (a)

174. Four statements are given below. Group two of them in such a way that one is logically incorrect and the other is verifiable as a matter of fact

(a) The Sun does not rise in the East.

(b) A straight line is the shortest distance between any two points.

(c) Every circle has a centre.

(d) The maximum duration of a total solar eclipse is about 7.5 minute s

(a)  [a, c]

(b)  [c, d]

(c)  [a, d]

(d)  [a, b]

Answer: (c)

175. This question consists of a related pair of words, followed by four pairs of words. Choose the pair that best represents a similar relations hip as the one expressed in the given pair of words:

SANDERLING : BIRD

(a)  Mastiff: Dog

(b)  Cat : Mare

(c)  Frog : Toad

(d)  Water : Fish

Answer: (a)

176. From among the given options, identify the statement which means t he same as the statement ‘The dual nature of light is an enigma’.

(a)  Two contradictory descriptions of light presuppose a third description.

(b)  Light has distinct properties which makes it unique.

(c)  Light is mysterious

(d)  The nature of light is an enigma.

Answer: (d)

177. Which word in the following group DOES NOT belong to the others?

PROSPER, EXCITE, THRIVE, FLOURISH

(a)  Prosper

(b)  Excite

(c)  Flourish

(d)  Thrive

Answer: (b)

178. Examine the following numbers and identify the next number:

45; 43; 40; 36; 31; 25; ….

(a)  23

(b)  29

(c)  17

(d)  18

Answer: (d)

179. If it is true that ‘All humans are imperfect’, then which one of the following is necessarily true?

(a)  No humans are imperfect.

(b)  Some humans are not imperfect.

(c)  Every human is imperfect.

(d)  All imperfect beings are humans.

Answer: (c)

180. If it is false that ‘Men always pray to God’, then which one of the following statements is true?

(a)  Men seldom pray to God.

(b)  Men always pray to God.

(c)  Some men pray to God.

(d)  All men pray to God.

Answer: (a)

181. Geeta is twice the present age of Seema. If age of Seema is 20 years now, how many years ago Geeta was three times Seema’s age?

(a)  10

(b)  8

(c)  12

(d)  9

Answer: (a)

182. In certain code MAIL is written as ZNVY then how will FILM be written in that code?

(a)  SVYZ

(b)  MLIF

(c)  NORY

(d)  XLMP

Answer: (a)

183. You are given a 60inch long ribbon, and you are instructed to cut 60 1-inch long strips out of this ribbon. The time taken to cut one strip i s one second. So how long will it take to cut 60 strips?

(a)  1 minute

(b)  1 minute and 1 second

(c)  30 seconds

(d)  59 seconds

Answer: (d)

184. If it is true that ‘Religious fundamentalism is dangerous to the society’, then which one of the following statements can also be true?

(a)  Disrespect for other religions is dangerous to the society.

(b)  Religious pluralism is dangerous to the society.

(c)  Religion without reason is dangerous to the society.

(d)  Belief in any religion is dangerous to the society.

Answer: (a)

185. If it is true that ‘Good governance implies law and order in society’ t hen identify the statement which has to be accepted along with the given statement?

(a)  An able government very effectively uses laws to promote peace.

(b)  A healthy society is governed by maximum number of laws.

(c)  A strong government uses force to impose laws.

(d)  Law is indispensable for the society.

Answer: (a)

186. If it is false that ‘There is at least one octogenarian in the room’, then which one of the following is probable?

(a)  No one in the room is an octogenarian.

(b)  All those who are in the room are octogenarians.

(c)  One person in the room is not an octogenarian.

(d)  Some people in the room are not octogenarians.

Answer: (a)

187. A, B, C and D have got some money with them. If A gives 8 rupees to B, B will have as much as C has and A will have 3 rupees less than what C has. Also if A takes 6 rupees from C, A will have twice as much money as D. If B and D together have 50 rupees, how much money does A and B have respectively ?

(a)  29, 32

(b)  27, 40

(c)  32, 29

(d)  40, 27

Answer: (d)

188. Who among the following is the odd one in the following group of persons? Chief Justice of India, Attorney General of India, Solicitor General, Advocate General

(a)  Chief Justice of India.

(b)  Advocate General.

(c)  Solicitor General

(d)  Attorney General of India.

Answer: (a)

189. In a bag, there are some diamonds. In another bag, there are one fou rth the number more than the number of diamonds in the first bag. If the difference in the number of diamonds in the first and second bag is 3, how many diamonds are there in the first bag?

(a)  10

(b)  16

(c)  12

(d)  8

Answer: (c)

190. X, the President of a club arrived in a meeting at 10 minutes to 12 30 hrs. Mr X came earlier by 20 minutes than the other participating members in the meeting, who arrived late by 30 minutes. At what time was the meeting scheduled?

(a)  1210 hrs

(b)  1220 hrs

(c)  1240 hrs

(d)  1250 hrs

Answer: (a)

191. In a class, student X has 8th position from the top and 84th from the bottom. How many students are there in the class?

(a)  96

(b)  98

(c)  91

(d)  88

Answer: (c)

192. There is some relationship between the figures given in the series. Find out the missing one from the alternatives given below.

(a) 

(b) 

(c) 

(d) 

Answer: (d)

193. If Oceans are Deserts, then Waves are:

(a)  Powerful

(b)  Ripples

(c)  Sand Dunes

(d)  Water

Answer: (c)

194. Assume that both premises, ‘No innocent person should be punished’ and ‘Socrates is innocent’ are true. Then which one of the following options is necessarily true?

(a)  Socrates may not be punished.

(b)  Socrates is not punished.

(c)  It is not proper to punish Socrates.

(d)  Socrates should not be punished.

Answer: (d)

195. A 2100 member team consisting of Team Leaders and Athletes is attending a National Athletic Meet. For every 20 Athletes, there is one Team Leader. How many Team Leaders would be there in the team?

(a)  100

(b)  105

(c)  110

(d)  95

Answer: (a)

196. Choose the pair of words from the options that best represents a similar relationship as the one expressed in the following pair of words.

WAITER: RESTAURANT

(a)  Driver : Car

(b)  Teacher : School

(c)  Author : Book

(d)  Actor : Acting

Answer: (b)

197. Examine the series and identify the missing number:

46, 44, 40, 38, 34, …

(a)  26

(b)  32

(c)  28

(d)  30

Answer: (b)

198. Identify the argument which cannot be accepted

(a)  All unmarried women are spinsters. Therefore, all spinsters are women.

(b)  All wives are married. Therefore, all married people are wives.

(c)  All equilateral triangles are equiangular. Therefore, all equiangular triangles are equilateral.

(d)  All rainy days are wet days. Therefore, all wet days are rainy days.

Answer: (b)

199. Identify the statement which cannot be true.

(a)  Snow is white.

(b)  All bachelors are faithful to their wives.

(c)  Black body radiation is a physical phenomenon.

(d)  Every natural number has a successor.

Answer: (b)

200. If it is false that ‘Animals are seldom aggressive’, then which one of t he following statements conveys the same meaning?

(a)  All animals are always aggressive.

(b)  All animals are aggressive.

(c)  At least one animal is aggressive.

(d)  Sometimes animals are aggressive.

Answer: (d)

Common Law Admission Test (CLAT) Online Examination Held on April 14, 2017 Question Paper with Answer Key

CLAT 2017 Online Exam Held on April 14, 2017
Common Law Admission Test (CLAT) Online Examination Held on April 14, 2017 Question Paper with Answer Key

Common Law Admission Test (CLAT) Online Examination

Held on April 14, 2017

Part 1 English including Comprehension

 

Directions (Q. Nos. 1-10) Fill in the blanks by choosing the most appropriate options.

1. If they want to succeed, they……….. have to work very hard.

(a)  must

(b)  ought

(c)  should

(d)  will

Answer: (d)

2. The minister flew ………… the flooded areas in a helicopter.

(a)  about

(b)  over

(c)  along

(d)  in

Answer: (b)

3. You have played a great role, for ……. your help I possibly would have landed myself into a problem.

(a)  without

(b)  despite

(c)  after

(d)  although

Answer: (a)

4. Would anybody ……… a mother have risked her life for the baby?

(a)  rather

(b)  but

(c)  than

(d)  however

Answer: (b)

5. Kanak is endowed ……… many great qualities.

(a)  of

(b)  by

(c)  in

(d)  with

Answer: (d)

6. The passengers were very happy ………… friendly and warm treatment.

(a)  from

(b)  by

(c)  about

(d)  to

Answer: (c)

7. Sunita decided to set ……… some time every day for prayers.

(a)  aside

(b)  on

(c)  up

(d)  in

Answer: (a)

8. She stood …….. Amit, but could not utter a single word for quite some time.

(a)  for

(b)  before

(c)  about

(d)  to

Answer: (b)

9. We shall fail ……….. we are industrious.

(a)  until

(b)  unless

(c)  though

(d)  whether

Answer: (b)

10. The doctor advised him to go ……… several medical tests.

(a)  through

(b)  into

(c)  about

(d)  under

Answer: (a)

Directions (Q. Nos. 11-15) Read the given passage carefully and choose the most appropriate option to the questions given below.

The Word Trade Organization (WTO) was created in the early 1990s as a component of the Uruguay Round negotiation. However, it could have been negotiated as part of the Tokyo Round of the 1970s, since negotiation was an attempt at a ‘constitutional reform’ of the General Agreement on Tariffs and Trade (GATT). Or it could have been put off to the future, as the US government wanted. What factors led to the creation of the WTO in the early 1990s?

One factor was the pattern of multilateral bargaining that developed late in the Uruguay Round. Like all complex international agreements, the WTO was a product of a series of trade-offs between principal actors and groups. For the United States, which did not want a new organization, the disputed settlement part of the WTO package achieved its longstanding goal of a more effective and m ore legal dispute settlement system. For the Europeans, who by the 1990s had come to view GATT dispute settlement less in political terms and more as a regime of legal obligations, the WTO package was acceptable as a means to discipline the resort to unilateral measures by the United States. Countries like Canada and other middle and smaller trading partners were attracted by the expansion of a rule-based system and by the symbolic value of a trade organization, both of which inherently support the weak against the strong. The developing countries were attracted due to the provisions banning unilateral measures. Finally, and perhaps most important, many countries at the Uruguay Round came to put a higher  priority on the export gains than on the import losses that the negotiation would produce, and they came to associate the WTO and a rule-based system with those gains. This reasoning-replicated in many countries-was contained in US Ambassador Kantor’s defence of the WTO, and it announced to a recognition that international trade and its benefits cannot be enjoyed unless trading nations accept the discipline of a negotiate rule-based environment.

A second factor in the creation of the WTO was pressure from lawyers and the legal process. The dispute settlement system of the WTO was seen as a victory of legalists but the matter went deeper than that. The GATT, and the WTO, are contract organizations based on rules, and it is inevitable that an organization creating a further rule will in turn be influenced by legal process. Robert Hudee has written of the ‘momentum of legal development’, but what is this precisely? Legal development can  be defined as promotion of the technical legal values of consistency, clarity (or certainty) and effectiveness; these are values that those responsible for administering any legal system will seek to maximize. As it played out in the WTO, consistency meant integrating under one roof the whole lot of separate agreements signed under GATT auspices; clarity meant removing ambiguities about the powers of contracting parties to make certain decisions or to undertake waivers; and effectiveness meant eliminating exceptions arising out of grandfather-rights and resolving defects in dispute settlement procedures and institutional provisions. Concern for these values in inherent in any rule-based system of cooperation, since without these value rules would be meaningless in the first place, therefore, create their own incentive for fulfillment.

The moment of legal development has occurred in other institutions besides the GATT, most notably in the European Union (EU). Over the past two decades, the European Court of Justice (ECJ) has consistently rendered decisions that have expanded incrementally the EU’s internal market, in which the doctrine of ‘mutual recognition’ handed down is Cassis de Dijon case in 1979 was a key turning point. The court is now widely recognized as a major player in European integration, even though arguably such a strong role was not originally envisaged in the Treaty of Rome, which initiated the current European Union. One mans the Court used to expand integration was the ‘teleological method of interpretation’, whereby the actions of member States were evaluated against ‘the accomplishment of the most elementary goals set forth in the Preamble to the (Rome) treaty. The teleological method represents an effort to keep current policies consistent with slated goals, and it is analogous to the effort in GATT to keep contracting party trade practices consistent with slated rules. In both cases, legal concerns and procedures are in independent force of further cooperation.

In the large part, the WTO was an exercise in consolidation. In the context of a trade negotiation that created a near-revolutionary expansion of international trade rules, the formation of the WTO was a deeply conservative act needed to ensure that the benefits of the new rules would not be lost. The WTO was all about institutional structure and dispute settlement, these are the concerns of conservatives and not revolutionaries, that is why lawyers and legalists took the lead on these issues. The WTO codified the GATT institutional practice that had developed by custom over three decades, and it incorporated a new dispute settlement system that was necessary to keep both old and new rules from becoming a sham. Both the international structure and the dispute settlement system were necessary to preserve and enhance the integrity of the multilateral trade regime that had been built incrementally from the 1940s to the 1990s.

11. In the statement ‘… it amounted to a recognition that international trade and its benefits cannot be enjoyed unless trading nations accept the discipline of a negotiated rule-based environment’, it refers to:

(a)  The export gains many countries came to associate with a rule-based system.

(b)  The higher priority on export gains placed by many countries at the Uruguay Round.

(c)  The provision of a rule-based system by the WTO.

(d)  Ambassador Kantor’s defence of the WTO.

Answer: (a)

12. What would be the closest reason why WTO was not formed in 1970s?

(a)  The US government did not like it.

(b)  Important players did not find it in their best interest to do so.

(c)  Lawyers did not work for the dispute settlement system.

(d)  The Tokyo Round negotiations was an attempt at constitutional reform.

Answer: (b)

13. In the method of interpretation of the European Court of Justice:

(a)  Actions against member states needed to be evaluated against the said community goals.

(b)  Enunciation of the most elementary community goals needed to be emphasized.

(c)  Current policies need to be consistent with stated goals.

(d)  Contracting party trade practices need to be consistent with stated rules.

Answer: (a)

14. According to the passage, WTO promoted the technical legal values partly through

(a)  Integrating under one roof the agreements signed under GATT.

(b)  Rules that create their own incentive for fulfilment.

(c)  Ambiguities about the powers of contracting parties to make certain decisions.

(d)  Grandfather-rights exceptions and defects in dispute settlement procedures.

Answer: (a)

15. The most likely reason for the acceptance of the WTO package by nations was that :

(a)  It has the means to prevent the US from taking unilateral measures.

(b)  Its rule-based system leads to export gains.

(c)  It settles disputes more legally and more effectively.

(d)  They recognized the need for a rule-based environment to protect the benefits of increased trade.

Answer: (d)

Direction (Q. Nos. 16 – 20) : In each of the following sentences, some part of the sentence or the entire sentence is underlined. Beneath each sentence, you will find four ways of phrasing the underlined part. Choose the most appropriate option given in each of the sentences given below that is the best version than the underlined part of the sentence.

16. Two valence states of uranium, one with a deficit of four electrons and the other one with a deficit of six occurs in nature and contributes to the diversity of uranium’s behaviour.

(a)  the other one a deficit of six, occur in Nature and contribute

(b)  the other with a deficit of six, occurs in Nature and contributes

(c)  the other with a deficit of six, occur in Nature and contribute

(d)  one with six occurs in Nature and contributes

Answer: (b)

17. Initiative and referendum, is a procedure that allows voters to propose and pass laws as well as to repeal them.

(a)  allows voters to propose, pass and to repeal laws

(b)  will allow laws on be proposed, passed, as well as repealed by voters

(c)  allows voters to propose to pass, and repeal laws

(d)  will allow voter to propose, pass, as well as to repeal laws.

Answer: (a)

18. Plausible though it sounds, the weakness of the hypothesis is that it does not incorporate all relevant facts.

(a)  the weakness of the hypothesis which sounds plausible.

(b)  though the hypothesis sounds plausible, its weakness

(c)  even though it sounds plausible, the weakness of the hypothesis

(d)  though plausible, the hypothesis’ weakness

Answer: (b)

19. Many of them chiselled from solid rock centuries ago the mountainous regions are dotted with hundreds of monasteries:

(a)  The mountainous regions are dotted with hundreds of monasteries, many of which are chiselled from solid rock centuries ago.

(b)  The mountainous regions are dotted with hundreds of monasteries, many of them chiselled from solid rock centuries ago.

(c)  Hundreds of monasteries, many of them chiselled from solid rock centuries ago, are dotting the mountainous regions.

(d)  chiselled from solid rock centuries ago, the mountainous regions are dotted with many hundreds of monasteries.

Answer: (b)

20. During her lecture, the speaker tried to clarify directional terms, for not everyone in attendance was knowledgeable that winds are designed by the direction from which they come.

(a)  With everyone in attendance not knowing.

(b)  For everyone in attendance did not know.

(c)  With everyone attending not knowledgeable.

(d)  For not everyone attending knew.

Answer: (d)

Direction (Q. Nos. 21 – 24): Choose the correct spellings in questions given below.

21. Choose the correct spellings in options given below.

(a)  Accommedation

(b)  Accommodation

(c)  Accomadation

(d)  Accomedation

Answer: (b)

22. Choose the correct spellings in options given below.

(a)  Ghallows

(b)  Gallows

(c)  Ghellows

(d)  Gellows

Answer: (b)

23. Choose the correct spellings in options given below.

(a)  Renumeration

(b)  Remuneration

(c)  Remunaration

(d)  Renumaration

Answer: (b)

24. Choose the correct spellings in options given below.

(a)  Blashphemy

(b)  Bleshphemy

(c)  Blasphamy

(d)  Blasphemy

Answer: (d)

Direction for Questions 25 – 28: Choose the correct spellings in questions given below.

25. Choose the correct spellings in options given below.

(a)  Hyphothecation

(b)  Hypathecation

(c)  Hypothecation

(d)  Hypthacation

Answer: (c)

26. Choose the correct spellings in options given below.

(a)  Gratuitous

(b)  Gratitious

(c)  Gratetious

(d)  Gratuitus

Answer: (a)

27. Choose the correct spellings in options given below.

(a)  Interrogation

(b)  Interogetion

(c)  Interogation

(d)  Interagation

Answer: (a)

28. Choose the correct spellings in options given below.

(a)  Annulement

(b)  Anulment

(c)  Annulment

(d)  Annulmant

Answer: (c)

Direction (Q. Nos. 29 and 30) : Choose the correct spellings in questions given below.

29. Choose the correct spellings in options given below.

(a)  Abhayence

(b)  Abheyance

(c)  Abeyance

(d)  Abeyence

Answer: (c)

30. Choose the correct spellings in options given below.

(a)  Moratarium

(b)  Moretorium

(c)  Maratorium

(d)  Moratorium

Answer: (d)

Directions (Q. Nos. 31 – 40): In each of the following sentences four words or phrases are underlined. If there is any mistake with regard to grammar or usage, it is in the underlined part only. Identify the incorrect part.

31. Evidently our product is the most unique in the market.

            A                                 B                      C      D

(a)  C

(b)  D

(c)  B

(d)  A

Answer: (c)

32. The most difficult job is to bend and then lifting the weight.

             A                                B                           C             D

(a)  C

(b)  D

(c)  B

(d)  A

Answer: (d)

33. The hope to go through the book today I have almost read half of it.

          A                   B                                         C             D

(a)  A

(b)  B

(c)  C

(d)  D

Answer: (c)

34. Our boss always asks us to pay  full attention to the work at hand.

                          A                      B        C                                       D

(a)  B

(b)  A

(c)  D

(d)  C

Answer: (c)

35.  On listening to the confession of love she blushed until she was purple.

              A                                   B                                     C               D

(a)  A

(b)  C

(c)  D

(d)  B

Answer: (c)

36. After being finished the last chapter of the book, return it to me.

                  A                    B                                              C          D

(a)  C

(b)  A

(c)  D

(d)  B

Answer: (b)

37. Since I have forgotten all the equations I will have to start from the scratch.

         A                B                                                 C                                 D

(a)  B

(b)  A

(c)  C

(d)  D

Answer: (d)

38. He gave me a ticket so that I may visit the book fair.

                   A                        B           C         D

(a)  C

(b)  A

(c)  B

(d)  D

Answer: (a)

39. Five gallons of petrol are not enough to cover the distance.

                          A              B             C                           D

(a)  A

(b)  B

(c)  D

(d)  C

Answer: (b)

40. The officer asked that the report  be submitted  immediately.

                                    A           B                 C                     D

(a)  D

(b)  C

(c)  A

(d)  B

Answer: (b)

Part 2: General Knowledge and Current Affairs

 

41. Juno is the name of a:

(a)  Juno is the name of a:

(b)  Hydrogen fuelled space craft

(c)  Solar powered space craft

(d)  Atomic powered submarine

Answer: (c)

42. Who has been selected for 2016 BC Roy Award?

(a)  Dr. P. Raghu Ram

(b)  Dr. J. Rajendra

(c)  Dr. N. Bhaskaran

(d)  Dr. Jagat Ram

Answer: (a)

43. The first country to have announced euthanasia of a child is:

(a)  Belgium

(b)  Denmark

(c)  Finland

(d)  Norway

Answer: (a)

44. The Japanese Prime Minister who offered ‘sincere and everlasting condolences’ to the people of the United States for killing more than 2,400 soldiers in the attack on Pearl harbour was:

(a)  Kakuei Tanaka

(b)  Shinzo Abe

(c)  Juniciro Koizumi

(d)  Hayato Ikeda

Answer: (b)

45. In terms of steel production in the world during 2015 – 2016, India stood at:

(a)  Fourth

(b)  Second

(c)  Fifth

(d)  Third

Answer: (d)

46. According to the Survey Report released by Transparency International during March 2017 on India, the most corrupt are

(a)  Government officials

(b)  Business executives

(c)  Local Councillors

(d)  Police

Answer: (d)

47. Among the following who was crowned as ‘Miss Supernational’ during 2016?

(a)  Srinidhi Shetty

(b)  Shilpa Shetty

(c)  Alia Butt

(d)  Aishwarya Rai

Answer: (a)

48. Japan is threatening to drag India to W.T.O on issues relating to the export of its:

(a)  Electronic goods

(b)  Tea

(c)  Small ships

(d)  Steel

Answer: (d)

49. COIN, a software programme developed by J. P. Morgan supports:

(a)  Robotic surgery

(b)  Financial accounting

(c)  Bitcoin

(d)  Interpreting commercial documents

Answer: (d)

50. Which of the following country enacted a law during August 2016 providing for the right to register the marriages of Hindus?

(a)  Saudi Arabia

(b)  Iran

(c)  Afghanistan

(d)  Pakistan

Answer: (d)

51. The first elected civilian President in Myanmar is :

(a)  Aung San

(b)  Khin Ayi

(c)  Htin Kyaw

(d)  Aung San Suu Kyi

Answer: (c)

52. Among the following professors of Indian origin who has received Knighthood for the work as a co-inventor of next generation DNA Sequencing called Solexa Sequencing is?

(a)  P. C. Mahalanobis

(b)  Shankar Balasubramanian

(c)  Mehnad Saha

(d)  Satyendra Nath Bose

Answer: (b)

53. Which court has stayed the execution of death sentence of Kulbhushan Jadhav in May 2017?

(a)  International Criminal Court

(b)  International Court of Justice

(c)  Supreme Court of India

(d)  Supreme Court of Pakistan

Answer: (b)

54. World’s longest rail tunnel is about:

(a)  32 kms.

(b)  57 kms.

(c)  23 kms.

(d)  47 kms.

Answer: (b)

55. The Hubble telescope of NASA is located in

(a)  Iceland

(b)  Space

(c)  Canada

(d)  NASA headquarters

Answer: (b)

56. The first Commercial Court and Commercial Disputes Resolution Centre was inaugurated at

(a)  Ahmedabad, Gujarat

(b)  Raipur, Chattisgarh

(c)  Mumbai, Maharastra

(d)  Visakapatinam, Andhra Pradesh

Answer: (b)

57. As on 31st January 2016, the highest number of law colleges were present in:

(a)  Madhya Pradesh

(b)  Andhra Pradesh

(c)  Uttar Pradesh

(d)  Maharashtra

Answer: (c)

58. The bowler who has claimed the fastest 250 wickets in Cricket test matches is:

(a)  Ravichandran Ashwin

(b)  Dennis Lillee

(c)  Rangana Herath

(d)  Anil Kumble

Answer: (a)

59. The top destination for domestic tourists in India for the past three consecutive years has been:

(a)  Tamil Nadu

(b)  Orissa

(c)  Rajasthan

(d)  Kerala

Answer: (a)

60. Name the President elect of France who is likely to take the oath on 14 May 2017.

(a)  Francois Fillon

(b)  Emmanuel Macron

(c)  Francois Holland

(d)  Marine Le Pen

Answer: (b)

61. With the development of Terahertz (THz) transmitter, it is expected to be faster than 5G mobile networks by:

(a)  Two times

(b)  Ten times

(c)  Four times

(d)  Five times

Answer: (b)

62. India’s third largest trading partner during 2016 is

(a)  Dubai

(b)  Saudi Arabia

(c)  Kuwait

(d)  U.A.E

Answer: (d)

63. The first country in the world to have begun shutting down the entire Frequency Modulation (FM) radio network to be replaced by Digital Audio Broadcasting is:

(a)  Norway

(b)  Switzerland

(c)  United States of America

(d)  China

Answer: (a)

64. The Happiness Index Department or a Wing has been established in the states of:

(a)  Madhya Pradesh and Tamil Nadu

(b)  Andhra Pradesh and Madhya Pradesh

(c)  Andhra Pradesh and Sikkim

(d)  Madhya Pradesh and Goa

Answer: (b)

65. Volvo has launched the world’s largest bus that can carry up to:

(a)  320 passengers

(b)  260 passengers

(c)  300 passengers

(d)  150 passengers

Answer: (c)

66. Donald Trump is _____________ President of the United States.

(a)  43rd

(b)  44th

(c)  46th

(d)  45th

Answer: (d)

67. The 2016 Nobel Peace Prize was won by the President of :

(a)  United States of America

(b)  Columbia

(c)  Sri Lanka

(d)  South Africa

Answer: (b)

68. The World Bank had cut India’s GDP growth for 2016 – 2017 to:

(a)  7%

(b)  8.6%

(c)  8%

(d)  7.6%

Answer: (a)

69. Solar Impulse–2 is:

(a)  Solar powered ship

(b)  Solar powered airplane that completed the first around the world

(c)  Hybrid airplane

(d)  Impact of climate change for overall increase of 2º C a year

Answer: (b)

70. The Union Cabinet has recently approved the setting up of a Permanent Tribunal for resolving:

(a)  Complaints from three services

(b)  Inter-state water disputes

(c)  Inter-state boundary disputes

(d)  Election disputes

Answer: (b)

71. Which country 3D – Printed a home of 37 sq.mts?

(a)  Spain

(b)  South Korea

(c)  Russia

(d)  Taiwan

Answer: (c)

72. ‘World’s longest-all women Non-stop flight’ from New Delhi to San Francisco covering 14,500 kms was operated by:

(a)  Air India

(b)  American Airlines

(c)  Jet Airways

(d)  United Airlines

Answer: (a)

73. Which country offered asylum seekers 1,200 Euros to leave by withdrawing their application for protection?

(a)  Italy

(b)  Germany

(c)  Denmark

(d)  France

Answer: (b)

74. During 2017, which SAARC country has notified the Right to Information Act?

(a)  Nepal

(b)  Sri Lanka

(c)  Afghanistan

(d)  Bhutan

Answer: (b)

75. Highest number of open prisons in India as on 2015 are in

(a)  Kerala

(b)  Maharashtra

(c)  Tamil Nadu

(d)  Rajasthan

Answer: (d)

76. India’s voting rights at the International Monetary Fund increased from 2.3 % to

(a)  2.5%

(b)  2.6%

(c)  2.8%

(d)  2.4%

Answer: (b)

77. Which of the following individuals was called a ‘deceptive actor’ by China’s foreign ministry during March 2017?

(a)  Dalai Lama

(b)  Donald Trump

(c)  Narendra Modi

(d)  Sirisena

Answer: (a)

78. ‘Scorpion kick’ is a phrase used in

(a)  Kalari Fight

(b)  Kung Fu

(c)  Kick Boxing

(d)  Kabaddi

Answer: (d)

79.Immediately before Antonio Guterres was appointed the U.N Secretary General in October 2016, he was

(a)  United Nations High Commissioner for Refugees

(b)  Commissioner General of UNRWA

(c)  The Prime Minister of Portuguese

(d)  United Nations High Commissioner for Human Rights

Answer: (a)

80. How many billionaires India has lost since demonetization on November 8, 2016?

(a)  Eleven

(b)  Eight

(c)  Two

(d)  Eighteen

Answer: (a)

81. The world’s first artificial intelligence lawyer, a robot, is named as:

(a)  Boss

(b)  Watson

(c)  Ross

(d)  IBM-LaBrain

Answer: (c)

82. Till the end of 2016, the total number of UNESCO’s World Heritage Sites in India is:

(a)  18

(b)  21

(c)  42

(d)  35

Answer: (d)

83. India emerged as ___________ largest holder of the U.S Government Securities at the end of 2016:

(a)  Twelfth

(b)  Twenty eighth

(c)  Fifteenth

(d)  Twenty fourth

Answer: (a)

84. Prithvi Defence Vehicle is

(a)  The name of a newly developed tank

(b)  The name of a single seated aircraft developed by HAL

(c)  The name of an amphibious Naval vessel

(d)  The name of India’s Nuclear Intercepter Missile

Answer: (d)

85. Which shoe company in the United States of America has won an IPR dispute against China recently for using their logo?

(a)  Adidas

(b)  Reebok

(c)  Nike

(d)  New Balance

Answer: (d)

86. Among the following, who has won the maximum number of titles?

(a)  Gopichand

(b)  P. V. Sindhu

(c)  Saina Nehwal

(d)  Srikant

Answer: (c)

87. NASA rediscovered India’s lunar spacecraft that was lost in the space during the past eight years known as:

(a)  Mangalayaan – I

(b)  Chandrayan – II

(c)  Chandrayan – I

(d)  Mangalayaan – II

Answer: (c)

88. Who called the immigration the ‘Trojan horse of Terrorism’?

(a)  Theresa May, Prime Minister of U.K

(b)  Angela Merkel, Chancellor of Germany

(c)  Donald Trump, President of the U.S

(d)  Viktor Orban, Prime Minister of Hungary

Answer: (d)

89. The first statue of a woman in Parliament Square in England is that of:

(a)  Queen Elizabeth – II

(b)  Margaret Thatcher

(c)  Millicent Fawcett

(d)  Mother Theresa

Answer: (c)

90. Among the following M.L.As who was disqualified by the Governor during January 2017 under Article 192 of the Constitution for undertaking government contracts?

(a)  P.C. George of Kerala

(b)  Jayalalithaa of Tamil Nadu

(c)  Srinivas Prasad of Karnataka

(d)  Uma Shankar Singh of Bihar

Answer: (d)

Part 3: Elementary Mathematics (Numerical Ability)

 

91. Keerthi’s father gave him some money to buy books. He spent half of the money equally to buy books and entertaining his friends. Whatever amount left with him, he deposited half in his savings account and gave Rs. 5 to a poor person as charity. Finally, Keerthi was left with Rs. 20 which he returned to his father. What amount did his father give him initially?

(a)  Rs. 120

(b)  Rs. 100

(c)  Rs. 200

(d)  Rs. 160

Answer: (b)

92. Praveen has Rs. 4,662 in the form of 2, 5 and 10 rupee notes. If these notes are in the ratio of 3:5:8, the number of five rupees notes with him is:

(a)  250

(b)  336

(c)  210

(d)  84

Answer: (c)

93. A train ‘X’ leaves station ‘A’ at 3 p.m and reaches station ‘B’ at 4.30 p.m., while another train ‘Y’ leaves station ‘B’ at 3.00 p.m and reaches station ‘A’ at 4.00 p.m. These two trains cross each other at:

(a)  3.30 p.m.

(b)  3.20 p.m.

(c)  3.40 p.m.

(d)  3.36 p.m.

Answer: (d)

94. The difference between simple interest and compound interest at the same rate for rupees 5,000 for two years is rupees 98. The rate of interest is:

(a)  10%

(b)  14%

(c)  12%

(d)  

Answer: (b)

95. Gold and copper are as heavy as water by 19 and 9 times respectively. The ratio in which these two metals be mixed so that the mixture is 17 times as heavy as water is:

(a)  2:3

(b)  3:2

(c)  4:1

(d)  3:4

Answer: (c)

96. There are two urns. One contains two white balls and four red balls, the other contains three white and nine red balls. All balls are of the same shape and size. From each urn, one ball is drawn. What is the probability of getting both the balls of the same colour?

(a)  1/24

(b)  7/12

(c)  1/12

(d)  1/2

Answer: (b)

97. Two men and seven boys can do a work in 14 days. Three men and eight boys can do the same work in 11 days. Further eight men and six boys can do three times the amount of this work in:

(a)  21 days

(b)  18 days

(c)  34 days

(d)  30 days

Answer: (a)

98. A piece of cloth costs rupees 75. If the piece is four meters longer and each meter costs rupees 5 less, the cost remains unchanged. What is the length of the piece?

(a)  8 meters

(b)  10 meters

(c)  12 meters

(d)  6 meters

Answer: (d)

99. A man rows to a place 45 k.ms distant and back in 12 hours. He realises that he can row 5 k.ms downstream in the same time as 3 k.ms against the stream. The velocity of the stream is:

(a)  2 k.ms/hr

(b)  1.5 k.ms/hr

(c)  1 k.m/hr

(d)  4 k.ms/hr

Answer: (a)

100. In an office, 1/3 of the workers are Men, 1/2 of the men are married and 1/3 of the married men have children. If 3/4 of the women are married and 2/3 of the 2/3 of the married women have children, then the part of workers without children are:

(a)  17/36

(b)  4/9

(c)  5/18

(d)  11/18

Answer: (d)

101. Taps ‘A’ and ‘B’ can fill a tank in 37 ½ minutes and 45 minutes respectively. Both taps are opened and after some time tap ‘B’ is turned off. The tank is filled completely in exactly 30 minutes, if tap ‘B’ is turned off after:

(a)  9 minutes

(b)  10 minutes

(c)  15 minutes

(d)  12 minutes

Answer: (a)

102. A boat travels upstream from A to B and back from B to A in 5 hours. The speed of the boat in still water is 8 km/hour and the speed of the current is 4 km/hour. Then, the distance from A to B is:

(a)  9 kms.

(b)  10 kms.

(c)  12 kms.

(d)  15 kms.

Answer: (d)

103. Age of father 10 years ago was three times the age of his son. After 10 years, father’s age is twice that of his son. The ratio of their present ages is:

(a)  7:3

(b)  9:5

(c)  7:4

(d)  11:7

Answer: (a)

104. A clock was set correct at 12 O’ clock. It loses 10 minutes per hour. What will be the angle between the hour and minute hands of the clock after one hour?

(a)  75°

(b)  85°

(c)  105°

(d)  90°

Answer: (b)

105. A trader sells rice at a profit of 20% and uses weights which are 10% less than the correct weight. The total gain earned by him is:

(a)  

(b)  

(c)  30%

(d)  35%

Answer: (a)

106. The average weight of three men ‘X’, ‘Y’ and ‘Z’ is 75 kgs. Another man ‘A’ joins the group and the average weight now becomes 80 kgs. If another person ‘B’ whose weight is 5 kgs more than ‘A’ replaces ‘X’, then the average weight of ‘Y’, ‘Z’, ‘A’ and ‘B’ will be 85 kgs. What is the weight of ‘X’?

(a)  80 kgs.

(b)  84 kgs.

(c)  82 kgs.

(d)  78 kgs.

Answer: (a)

107. ‘A’ and ‘B’ complete a work in 12 days, ‘B’ and ‘C’ in 8 days and ‘C’ and ‘A’ in 16 days. ‘A’ left after working for 3 days. In how many days more will ‘B’ and ‘C’ finish the remaining work?

(a)  6 & frac56;

(b)  

(c) 

(d) 

Answer: (d)

108. A vessel contains a mixture of milk and water in the ratio of 5:3 respectively. How much of the mixture must be siphoned off and replaced with water, so that the mixture may be half milk and half water?

(a)  1/3

(b)  1/4

(c)  1/7

(d)  1/5

Answer: (d)

109. A can do a piece of work in 8 days and B alone can do the same work in 10 days. A and B agreed to do the work together for Rs. 720. With the help of C, they finished the work in 4 days. How much C is to be paid?

(a)  Rs. 80

(b)  Rs. 82

(c)  Rs. 72

(d)  Rs. 70

Answer: (c)

110. The Banker’s discount on a sum of money for 18 months is Rs. 600 and the true discount on the same sum for 3 years is Rs. 750/-. The rate percentage is:

(a)  10%

(b)  20%

(c)  12%

(d)  15%

Answer: (b)

Part 4 Legal Aptitude

This section consists of fifty (50) questions. Follow the instructions carefully and answer the questions.

Direction (Q. Nos. 111-145) consists of legal proposition(s)/ principle(s) (hereinafter referred to as ‘principle’) and facts. Such principles may or may not be true in the real and legal sense, yet you have to conclusively assume  them to be true for the purposes of this Section. In other words, in answering these questions, you must not rely on any principle except the principles those are given herein below for every question. Further, you must not assume any facts other than those stated in the question. The objective of this section is to test your interest towards study of law, research aptitude and problem solving

ability, even if the ‘most reasonable conclusion’ arrived at may be absurd or

unacceptable for any other reason. It is not the objective of this section to test your knowledge of law.

Therefore, to answer a question, principle is to be applied to the given facts and to choose the most appropriate option.

111. Principle: The concept of natural justice is against bias and for the right to a fair hearing. While the term natural justice is often retained as a general concept, and it has largely been replaced and extended by the general ‘duty to act fairly’.

Fact: ‘X’, a male employee of a company was dismissed by the employer just on the basis of a complaint by ‘Y’, a female employee of the company that ‘X’ was trying to be too friendly with her and often requested her to accompany him to the canteen.

Is the dismissal of ‘X’ valid?

(a)  No, because in the modern times this type of behaviour is common

(b)  No, because the employer did not give a chance to ‘X’ to explain his side, thereby violated the principles of natural justice.

(c)  Yes, moral law is antique and therefore, not applicable in modern times, therefore the termination is valid and no violations of the principles of natural justice occurred

(d)  Yes, because men are not supposed to behave improperly with women and hence there is no violation of any principles of law

Answer: (b)

112. Principle: Ownership in property consists of right to possess, right to use, right to alienate and right to exclude others. Sale is complete when property gets transferred from the seller to the buyer on sale.

Facts: ‘A’ sold his car to ‘B’. After this, ‘B’ requested ‘A’ to keep the car in his care on behalf ‘B’ for one month. ‘A’ agreed.

(a)  Sale of car is not complete

(b)  Sale will be completed when ‘B’ will take the delivery of the car.

(c)  Sale of car is complete.

(d)  Sale will be automatically completed after the expiry of one month

Answer: (c)

113. Principle: Every agreement, by which any party is restricted absolutely from enforcing his right in respect of any contract, by the usual legal proceedings in the ordinary Tribunals, is void to that extent. The law also provides that nobody can confer jurisdiction to a civil court by an agreement between parties.

Facts: A and B entered into a valid contract for rendering certain service. A clause in the contract was that in case of any dispute arose out of the contract; it shall be referred to for Arbitration only. Is the contract valid?

(a)  Arbitration is also a valid dispute settlement machinery recognized by law and hence the entire contract is valid.

(b)  The parties were trying to confer jurisdiction to some authority to decide a dispute and hence the clause would be invalid.

(c)  Arbitrator cannot be termed as an ordinary Tribunal. Hence, the agreement is void and would be unenforceable.

(d)  The contract is valid but the clause regarding Arbitration is void.

Answer: (a)

114. Principle: It is a case of fraud where a party to a contract knows or believes a fact to be true, but conceals it actively from the other party with a view to induce that person to enter into the contract.

Facts: While taking a life insurance policy, in reply to questions by the insurance company during the inquiry into his proposal, Zameer deliberately concealed the fact of his medical treatment for a serious ailment, which he had undergone only a few weeks ago.

(a)  The act of Zameer did not amount to fraud, as disclosing the fact would have resulted in exposure of his privacy.

(b)  The act of Zameer amounted to innocent misrepresentation

(c)  The act of Zameer did not amount to any misrepresentation.

(d)  The concealment of fact by Zameer amounted to fraud.

Answer: (d)

115. Principle: Contract is a written or spoken agreement, with specific terms between two or more persons or entities in which there is a promise to do something in return for a valuable benefit known as consideration. Such an agreement is intended to be enforceable by law. A unilateral contract is one in which there is a promise to pay or give other consideration in return for actual performance.

Facts: A Toilet Soap Manufacturing Company in India in order to promote the sale of their product, published an advertisement in all the Newspapers on January 1, 2017 that the Company has kept a model ignition key of an Audi A3 Car. The advertisement also stated that whoever gets the said key before December 31, 2017 from a soap bar will be gifted with the Audi A3 Car. Mr. Martin, a foreigner who came to India as a Tourist who was staying in a Hotel found a Key similar to same Car Ignition Key. Mr. Martin brought this matter to the notice of the Hotel Manager. The Manager informed Mr. Martin about the Company’s advertisement on January 1, 2017. Mr. Martin wants to claim the Car. Will he succeed?

(a)  No. The Soap Company has not entered into a contract with Mr. Martin as he was not in India on January 1, 2017 when the advertisement was published.

(b)  No. Actual intention of the Company was to promote the sale of the Soap.

(c)  The Hotel Manager who could legally claim the Car as he was the one actually purchased the soap for the use in the Hotel.

(d)  Mr. Martin obtained the Key before the stipulated date from the Soap Bar. So he is covered by the offer of the Soap Company and can claim the car.

Answer: (d)

116. Principle: When a person who has made a promise to another person to do something does not fulfill his promise, the other person becomes entitled to receive, from the person who did not fulfill his promise, compensation in the form of money.

Facts: ‘X’ made a promise to ‘Y’ to repair his car engine. ‘Y’ made the payment for repair. After the repair, ‘Y’ went for a drive in the same car. While driving the car, ‘Y’ met with an accident due to bursting of a tyre.

(a)  ‘Y’ will be entitled to receive compensation from ‘X’ in the form of money.

(b)  ‘X’ will not be entitled to receive compensation.

(c)  ‘X’ will be entitled to receive compensation from ‘Y’ in the form of money.

(d)  ‘Y’ will not be entitled to receive compensation from ‘X’.

Answer: (d)

117. Principle: In criminal law, misappropriation is the intentional, illegal use of the property or funds of another person for one’s own use or other unauthorized purpose, particularly by a public official, a trustee of a trust, an executor or administrator of a dead person’s estate or by any person with a responsibility to care for and protect another’s assets. Embezzlement is misappropriation when the funds involved have been lawfully entrusted to the embezzler. On the contrary, theft is the illegal taking of another person’s property or services without that person’s permission or consent with the intent to deprive the rightful owner of it.

Facts: A went for swimming at the Municipal Swimming Pool. A handed over all his valuables, including some cash to X, the guard on duty for safe custody, as notified by the Municipality. After swimming for an hour, A came out and searched for X. He found another guard on duty and that guard informed A that X had gone home after completing his shift and did not hand over anything to be given to A. A registered a complaint with the police. X was traced but he told the police that he sold all the valuables and the entire cash was used for drinking liquor. What offence, if any, was/were committed by X?

(a)  X is not guilty of criminal misappropriation as he did not make any personal gain out of those items with him.

(b)  X is liable for criminal misappropriation and embezzlement.

(c)  X is liable for theft as he took A’s property without X’s permission.

(d)  If at all X is liable, it is for criminal misappropriation only.

Answer: (b)

118. Principle: Under the Employees Compensation Act, 1923, an employer is liable to pay compensation to his workmen for injuries sustained by them by an accident arising out of and in the course of employment.

Facts: M, the Manager of SRK Industries asked his secretary S to submit a report at the Government Labour Office. ‘S’ submitted the report as directed. On his way back S met one of his class mates. He then decided to have a cup of tea together on a way side restaurant. Sometime later, ‘S’ got a message from his office to report back as it was long time since he left the office. ‘S’ rushed back on his Motor Cycle. On his way back a Truck which was coming from a side road hit ‘S’. He was admitted in a nearby hospital with multiple injuries. He claims compensation under the Employees Compensation Act

from his employer.

(a)  The Employer is not liable as the truck driver was negligent.

(b)  The Employer is liable as S had to rush back to the office, because of the message from the office.

(c)  The Employer is liable to pay compensation as the accident took place arising out of and in the course of employment.

(d)  The Employer is not liable as he was admitted in a private hospital and not a Government Hospital.

Answer: (c)

119. Principle: If a party to a contract agrees to it under undue influence of any other party then the party under the undue influence may refuse to perform in accordance with the agreement.

Facts: A, a rich youngster became a member of a religious group and soon he was appointed by P the head of the group as his personal secretary. As per the rules of the group, all officials and staff of the group were supposed to stay in the group’s official premises itself. Some days later, A was asked by P to execute a Gift deed in favour of P, in which it was mentioned that all immovable properties in his name are being gifted to P. A was unwilling to execute the deed, but he was forcefully restrained by P and his body guards in P’s office and made A sign the gift deed. Soon after this A left the group and refused to hand over the property as agreed to in the gift deed. Is A’s action valid?

(a)  A executed the deed, under compulsion and undue influence, and was right in withdrawing from the contract.

(b)  It is illegal for religious groups acquire property from its members.

(c)  As the gift deed was executed by A, he cannot refuse.

(d)  As Gift is also a contract, the consent of A was not obtained by P while executing the deed.

Answer: (a)

120. Principle: Penal laws provide that whoever voluntarily has carnal intercourse against the order of nature with any man or woman, shall be punished for rape.

Facts: A Police Officer found a man engaged in carnal intercourse with an animal. The Police Officer arrested the man and produced him before the Court.

(a)  Court will punish the police officer.

(b)  Court will not punish the man for rape.

(c)  Court will punish the man for rape.

(d)  Court will not punish the police officer.

Answer: (b)

121. Principle: According to law, a person who find goods belonging to another and takes them into his custody, is subject to the same responsibility as a bailee. Bailee is a person or party to whom goods are delivered for a purpose, such as custody or repair, without transfer of ownership. The finder of the goods legally can sell the goods found by him under certain circumstances including the situation that the owner refuses to pay the lawful charges of the finder.

Facts: P, a college student, while coming out of a Cricket stadium found a necklace, studded with apparently precious diamonds. P kept it for two days thinking that the owner would notify it in a local newspaper. Since he did not notice any such notification, P published a small classified advertisement in a local newspaper. In two days’ time, P was contacted by a film actor claiming that it was her Necklace and requested P to return it to her. P told her that she should compensate him for the advertisement charges then only he would return it otherwise he will sell it and make good his expenses. The film star told P that she had advertised in a national newspaper about her lost Necklace which was lost somewhere in the Cricket Stadium. The advertisement was published for three consecutive days incurring a large expenditure for her. Mentioning all this she refuses to pay P and claims the Necklace back. Which

among the following is the most appropriate answer to this?

(a)  P was requesting the film star for the actual expenditure incurred by him before returning the Necklace. This request is legally sustainable.

(b)  The film star was right in refusing P, as she did not offer any reward for anyone who would return the Necklace.

(c)  As it was wrong on the part of P to bargain over a property belonging to a celebrity and he should have accepted some gift which might have been given by the film star and returned the Necklace instead of threatening her that he would sell it.

(d)  As the film star had notified in the newspaper, P ought to have read it and contacted her instead of publishing another notification. So he cannot claim any compensation.

Answer: (a)

122. Principle: A violation of a legal right of someone, whether results in a legal injury or not, gives rise to an action in tort for compensation. At the same time, an action by someone, which results in some loss or damage to somebody else is not actionable, if there is no violation of a right of that somebody.

Facts: AB Coaching Centre was a popular CLAT coaching academy with several good trainers. A lot of aspirants used to attend its coaching classes from all over and was making good profit. This was going on for the past several years. During a session, T, one of the very good and popular trainers of ABCC, had some difference of opinion with the owner of ABCC and left the coaching centre. In August 2016, T started another Entrance Coaching Centre closer to ABCC which resulted in a substantial drop in its students and huge financial loss. The owner of ABCC wants to file a case against T for the loss sustained by ABCC. What do you think is the right legal position?

(a)  T has not violated any of ABCC’s legal right though they sustained some financial loss, and not legally bound to compensate ABCC.

(b)  T will be liable to compensate the loss to ABCC.

(c)  T started the new coaching centre near ABCC intentionally, and shall be liable to compensate the loss of ABCC.

(d)  ‘T’ should have consulted ABCC before starting his coaching centre.

Answer: (b)

123. Principle: An offer made by one party when accepted by another makes it a contract.

Transactions:

1. P offered to sell his house for Rs. 20 lakhs to R; R told P that he was interested to buy a house for 15 lakhs only.

2. C was looking for a house for not more than 25 lakhs; P informed C that his house was available for 20 lakhs.

3. K wanted to buy some old furniture; L told K that he would sell his furniture for Rs. 10, 000.

4. R advertised to sell his old car for a price of Rs. Three lakhs; S found the

advertisement and offered to buy it for Rs. 2 lakhs 50 thousand; R agrees to sell itto S.

Which among the above is actually a contract?

(a)  Situations 1 and 2 are contracts

(b)  Situation 4 only is a contract

(c)  Situation 3 only is a contract

(d)  Situations 2 and 4 are contracts

Answer: (b)

124. Principle: Every agreement, of which the object or consideration is opposed to public policy, is void. An agreement which has the tendency to injure public interest or public welfare is one against public policy. What constitutes an injury to public interest or public welfare would depend upon the times and the circumstances.

Facts: ‘A’ promises to obtain for ‘B’ an employment in the public service, and ‘B’ promises to pay rupees 5,00,000/- to ‘A’.

(a)  The agreement is void, as the object and consideration for it is opposed to public policy.

(b)  The agreement is void because rupees 5,00,000/- is excessive.

(c)  The agreement is valid, as it is with consideration for public service.

(d)  The agreement is valid, as it is a contract between two parties with their free consent.

Answer: (a)

125. Principle: According to the law of trade unions in India, no suit or other legal proceeding shall be maintainable in any civil court against any registered trade union or any officer or member thereof in respect of any act done in contemplation or in furtherance of a trade dispute.

Facts: Soloman, the Secretary of a registered Trade Union took a loan from a Bank for the higher education of his daughter. Soon after completing the course she was married to an NRI Engineer. Solomon did not repay the loan. The Bank demanded the payments from Soloman and warned him that the Bank will take suitable legal action against him. Identify the legal position in this regard.

(a)  The Bank can file a suit for recovery of the loan amount against Soloman as he took the loan for a personal purpose and in such case no immunity will work.

(b)  The Bank can recover the loan amount from the Trade Union as Soloman is the Secretary of the Union.

(c)  The Bank cannot initiate any action against Soloman as he is the Secretary of a Registered Trade Union.

(d)  As Soloman did not use the loan amount for his use and hence, no action can be initiated against him.

Answer: (a)

126. Principle: When a person makes such a statement which lowers other person’s reputation in the estimation of other persons, is liable for committing defamation.

Facts: ‘A’ writes a letter to ‘B’ in which he uses abusive language against ‘B’ and also states that ‘B’ is a dishonest person. ‘A’ put the letter in a sealed envelope and delivered it to ‘B’.

(a)  ‘A’ has committed defamation

(b)  ‘A’ has committed a moral wrong

(c)  ‘A’ has not committed moral wrong

(d)  ‘A’ has not committed defamation

Answer: (d)

127. Principle: Nothing is an offence which is done in the exercise of the right of private defence.

Facts: ‘A’, under the influence of madness, attempts to kill ‘B’. ‘B’ to save his life kills ‘A’.

(a)  ‘B’ has not committed any offence.

(b)  ‘B’ has committed an offence.

(c)  ‘A’ has not committed an offence because he was mad.

(d)  ‘A’ has committed the offence of attempt to murder.

Answer: (a)

128. Principle: An agreement, the terms of which are not certain, or capable of being made certain, is void.

Facts: Sunder agreed to take Bhola’s penthouse on rent for three years at the rate of rupees 12, 00, 000/- per annum provided the house was put to thorough repairs and the living rooms were decorated according to contemporary style.

(a)  There is a valid contract because there is an offer from Sunder and acceptance from Bhola

(b)  There is a valid contract because all the terms of contract are certain and not vague as the rent is fixed by both of them and the term ‘present style’ only can be interpreted to mean the latest style.

(c)  There is no valid contract because it has vague and uncertain terms, as the term ‘present style’ may mean one thing to Sunder and another to Bhola.

(d)  It is voidable contract at the option of Bhola.

Answer: (c)

129. Principle: A master shall be liable for the fraudulent acts of his servants committed in the course of employment. However, the master and third parties must exercise reasonable care in this regard.

Facts: Rahul was a door to door salesman with United Manufacturing Company (the Company). The Company was manufacturing Water Purifiers. Rahul, along with the Company’s products, used to carry Water Purifiers manufactured by his Cousin in a local Industrial Estate. He used to sell the local product at a lower rate giving the impression to the buyers that he is offering a discount on the Company’s product. The Company Management detected the fraudulent activity of Rahul and dismissed him from service. Rahul still continued to carry on with his activity of selling the local product pretending that he was still a salesman of the Company. Several customers got cheated in this process. The fraud was noticed by the Company when the customers began to complain about the product. The customers demanded the Company to compensate their loss.

(a)  The Company is liable to the customers who purchased the local product from Rahul only till he remained as a salesman of the Company.

(b)  The Company is not liable as Rahul was dismissed by the Company.

(c)  The Company is liable to compensate all the customers as it did not inform the public about Rahul’s fraudulent conduct and the subsequent dismissal.

(d)  The liability rests with the local manufacturer as it was a defective product.

Answer: (c)

130. Principle: According to law, a person is deemed to have attained the age of majority when he completes the age of 18 years, except in the case of a person where a guardian of a minor’s person or property has been appointed under the Guardians and Wards Act, 1890 or where the superintendence of a minor’s property is assumed by a Court of Wards. Indian law expressly forbids a minor from entering into a contract. Hence, any contract entered into by a minor is void-ab-initio regardless of whether the other party was aware of his minority or not. Further, though a minor is not competent to contract, nothing in the Contract Act prevents him from making the other party bound to the minor.

Facts: Lal executed a promissory note in favour of Gurudutt, aged 16 years stating that he would pay Gurudutt a sum of Rs. 2 Lakhs when he attains the age of majority. On attaining the age of 18, Gurudutt demanded the amount from Lal, who refused to pay. Gurudutt wants to take legal action against Lal. Identify the most appropriate legal position from the following:

(a)  A promissory note duly executed in favour of a minor is not void and can be sued upon by him, because he though incompetent to contract, may yet accept a benefit.

(b)  Gurudutt should not have entered into a contract with Lal when he was a minor.

(c)  Lal was not aware of the fact that Gurudutt was a minor.

(d)  Lal argues that as per the Guardians and Wards Act, 1890, Gurudutt can claim the money only after he attains the age of 21.

Answer: (a)

131. Principle: Section 34 of Indian Penal Code provides that ‘When a criminal act is done by several persons in furtherance of the common intention of all, each of such persons is liable for that act in the same manner as if it were done by him alone.’

Facts: Three vagabonds, Sanju, Dilbag and Sushil decided to commit burglary. In the night, Sushil opened the lock and they broke into a rich man’s house when the entire family was on a pilgrimage. Sanju had gone to that house earlier in connection with some cleaning job. There was only a servant lady in the house. Hearing some sounds from the master bed room, the servant switched on the lights and went up to the room from where she heard the sound. Noticing that the servant was going to cry for help, Sanju grabbed her and covered her mouth with his hands and dragged her into the nearby room. The other two were collecting whatever they could from the room. When they were ready to go out of the house, they looked for Sanju and found him  committing rape on the servant. They all left the house and the servant reported the matter to the police and identified Sanju. Subsequently, all three were arrested in connection with the offences of house breaking, burglary and rape. Identify the legal liability of the three.

(a)  All three are liable for all the offences as there was common intention to commit the crimes.

(b)  Sanju will be liable only for housebreaking and rape as he did not participate in the burglary.

(c)  Only Sanju will be liable for rape as he was the one who actually committed the offence.

(d)  Only Dilbag and Sushil are liable for burglary in looting the house, and all three will be liable for housebreaking and rape as they did not stop Sanju from committing the offence and hence were accomplice to the offence.

Answer: (c)

132. Principle: Assault is causing bodily injury to another person by use of physical force.

Facts: Rustum while entering into compartment of a train raised his fist in anger towards a person Sheetal, just in front of him in the row, to get way to enter into the train first, but did not hit him. Rustum has:

(a)  committed an assault on Sheetal

(b)  insulted Sheetal

(c)  not committed an assault on Sheetal

(d)  Rightly showed his anger

Answer: (c)

133. Principle: According to Sec. 2 of the Industrial Disputes Act, 1947, ‘Industrial dispute means any dispute or difference between employers and employers or between employers and workmen or between workmen and workmen, which is connected with the employment or non-employment or the terms of employment or with the conditions of labour of any person’.

Facts: The employees of DK Enterprises met the management and requested half a day leave to allow them to celebrate a lunar eclipse, which was going to happen two days later. The management refused the request. Does this situation amount to an ‘industrial dispute’?

(a)  Yes, because there is some difference of opinion it would be an industrial dispute.

(b)  No as declaring holidays is a prerogative of the employer. So no industrial dispute.

(c)  No as Lunar eclipse is unconnected with employment.

(d)  As the difference of opinion between the employees and employer is on declaration of holiday it amounts to an issue connected with employment or with the terms of employment and hence, an industrial dispute.

Answer: (d)

134. Principle: When a person falsifies something with the intent to deceive another person or entity is forgery and is a criminal act. Changing or adding the signature on a document, deleting it, using or possessing the false writing is also considered forgery. In the case of writing/painting to fall under the definition, the material included must have been fabricated or altered significantly in order to represent something it is actually not.

Facts: David made a living traveling from city to city, selling paintings that he claimed were done by great artists. Since the artists’ signatures were in place, many people fell for them and purchased the paintings. One of these artists saw three of his alleged paintings in a City gallery containing his name. He knew these were not his works and he complained to the police. Police traced David and initiated legal proceedings. Is David guilty of any offence?

(a)  David is guilty of forgery as the addition of the signature was with an intention to make people believe that those were the paintings of the great artists.

(b)  There is no point in taking legal action against David as the signature has not done any alteration to the art work.

(c)  Those who buy the art pieces from David ought to have been careful in checking it and ensuring that they were originals before purchasing it.

(d)  David is not guilty of any offence as he was selling the art pieces for his living.

Answer: (a)

135. Principle: The Constitution of India guarantees certain fundamental rights to its citizens. The Constitution also provides that these rights cannot be taken away by state even by a law. For violation of this, the person adversely affected by the law may approach the High Court or the Supreme Court for the issuance of an appropriate writ. One of these rights includes the freedom to form association that implies the right to join an association or not to join such an association.

Facts: Owing to some industrial disturbances created by XATU, one of the several trade unions in AB Chemicals (Pvt) Ltd., the Company issued a circular to all its employees that as far as possible the employees may disassociate with XATU. Navin is an employee of AB Chemicals and the current General Secretary of XATU. Aggrieved by this circular, which affected the fundamental rights of his and other members of the Union, approaches the High Court of the state for a relief. Identify the most reasonable legal proposition.

(a)  The Company’s circular is illegal and has to be quashed by the Court.

(b)  The prohibition against any imposition of restriction against a fundamental right is not applicable to anybody other than the state and hence Navin will not get any relief from the High Court.

(c)  Circular issued by a Company amounts to law in the constitutional sense and hence the High Court can issue a writ as pleaded for by Navin.

(d)  The circular interferes with the freedom guaranteed by the Constitution and hence the High Court can issue an appropriate writ.

Answer: (b)

136. Principle: When a person interferes with peaceful possession of another person without the permission of the person in possession of those premises, commits trespass to land.

Facts: ‘T’ just walked over the land of ‘P’ to reach his house as it was a short cut. ‘P’ had displayed a notice that it is not a thoroughfare. ‘P’ did not cause any damage to the land.

(a)  ‘T’ has violated privacy of ‘P’

(b)  ‘T’ has not committed any trespass on the land of ‘P’.

(c)  ‘T’ has committed trespass to land

(d)  ‘T’ has created nuisance for ‘P’

Answer: (c)

137. Principle: A contract would be invalid and unlawful, if the contract is for an immoral or illegal purpose.

Facts: P, was a young and helpless widow, living on the pavement. R, a neighbour gave her a house, registered in her name, on the condition that she should allow R to keep his smuggled goods and drugs in her house. After the registration was done, according to the condition in the contract, R’s agents went to keep some packets in her house, she refused. R told her the condition under which the house was given to her. She still refused. Is P justified in her action?

(a)  P is not justified as she did not have the right to deny R’s request.

(b)  As R was making the contract for illegal activities, P’s stand is valid in law.

(c)  R can take back the house by cancelling the transfer deed.

(d)  P is right as she did not like smuggled goods to be kept in her house.

Answer: (b)

138. Principle: When a person falsifies something with the intent to deceive another person or entity is forgery and is a criminal act. Changing or adding the signature on a document, deleting it, using or possessing the false writing is also considered forgery. In the case of writing to fall under the definition, the material included must have been fabricated or altered significantly in order to represent something it is actually not.

Facts: John was a publisher of ancient books and papers. In one of his books on the World Wars, he gave photograph of some letters written by famous historic personalities. A researcher in history noted that in the pictures of some of the letters printed in the book, John had added some words or sentences in his own handwriting to give completeness to the sentences, so that the readers will get a clear picture of the writer’s intention. The researcher challenges the originality of those pictures and claims that the book containing the forged letters should be banned. Examine the validity of the researcher’s demand.

(a)  The additions in the letters were made by the publisher in his own handwriting would have made material alteration to the original meaning and hence amounted to forgery.

(b)  Allowing forged publications to be circulated among the public is as good as committing fraud on the public, so the publication should be banned.

(c)  As forgery amounts to adding or deleting anything from an original document, the demand of the researcher is valid.

(d)  The additions were made to give clarity to the original document and did not in any sense change the contents of the documents and hence there is no forgery as alleged by the researcher.

Answer: (d)

139. Principle: Whoever takes away with him any minor less than sixteen years of age if a male, or less than eighteen years of age if a female, out of the custody of parents of such minor without the consent of such parents, is said to commit no offence.

Facts: ‘A’, a man, took away a girl below sixteen years to Mumbai without informing the parents of the girl.

(a)  ‘A’ committed no offence against the parents of the girl.

(b)  ‘A’ committed no offence against the girl as well as her parents.

(c)  ‘A’ committed an offence against the girl as well as her parents

(d)  ‘A’ committed an offence against the girl.

Answer: (b)

140. Principle: Acceptance of a proposal must be absolute and unqualified.

Facts: ‘A’ made a proposal to sell his motorcycle to ‘B’ for rupees 25,000/-. ‘B’ agreed to buy it for rupees 24,000/-. ‘A’ sold his motorcycle to ‘C’ for 26,000/- the next day. ‘B’ sues ‘A’ for damages.

(a)  ‘B’ will get damages from ‘A’

(b)  ‘B’ will get the difference of rupees 1,000/- only

(c)  ‘B’ can proceed against ‘C’

(d)  ‘B’ will not get any damages from ‘A’

Answer: (d)

141. Principle: A person is said to do a thing fraudulently, if he does that thing with intent to defraud, but not otherwise.

Facts: ‘A’ occasionally hands over his ATM card to ‘B’ to withdraw money for ‘A’. On one occasion ‘B’ without the knowledge of ‘A’, uses ‘A’s ATM card to find out the balance in ‘A’s account, but does not withdraw any money.

(a)  ‘B’ has not committed the act fraudulently

(b)  ‘B’ has committed the act fraudulently

(c)  ‘B’ has committed breach of faith

(d)  ‘B’ has committed misappropriation

Answer: (a)

142. Principle: Where one of the parties to a contract was in a position to dominate the decision of the other party, the contract is enforceable only at the option of the party who was in a position to dominate the decision of the other party.

Facts: A doctor asked his patient to make a payment of rupees Ten Lakh for treatment of his fever. The patient paid an amount of rupees Five Lakh and promised to pay the remaining amount after the treatment. After treatment the patient recovered from fever. The doctor demanded the remaining amount from the patient. The patient refused to pay.

(a)  The contract is not enforceable without the consent of the patient.

(b)  The contract is not enforceable as doctor was in dominating position.

(c)  The contract is enforceable against the doctor.

(d)  The contract is enforceable against the patient by the doctor.

Answer: (d)

143. Principle: Negligence is actionable in law. In simple terms, negligence is the failure to take proper care over something.

Facts: A, a doctor, conducted a hysterectomy sincerely on B and left a small cotton swab inside the abdomen. As a consequence of which B developed some medical problems and had to undergo another surgery. Is A liable?

(a)  A is not liable as he did not foresee any consequences at the time of surgery.

(b)  A is liable for the negligence as he failed to take proper care during the surgery.

(c)  Liability for negligence does not arise here as A performed the operation sincerely

(d)  As only a small swab was left in the abdomen, there was no negligence.

Answer: (b)

144. Principle: When a person consented to an act to be done by another, he cannot claim any damages resulting from doing that act, provided the act done is the same for which consent is given.

Facts: ‘P’ submitted a written consent to a surgeon ‘S’ for undergoing a surgical operation for removal of appendicitis. The surgeon while doing surgery also removed the gall bladder of ‘A’:

(a)  ‘P’ can claim damages from ‘S’

(b)  ‘P’ is required to pay expenses for surgery for Appendicitis but not for Gall Bladder

(c)  ‘P’ cannot claim damages from ‘S’

(d)  ‘P’ is not bound to pay expenses of the surgery

Answer: (a)

145. Principle: There are legal provisions to give authority to a person to use necessary force against an assailant or wrong-doer for the purpose of protecting one’s own body and property as also another’s body and property when immediate aid from the state machinery is not readily available; and in so doing he is not answerable in law for his deeds.

Facts: X, a rich man was taking his morning walk. Due to the threat of robbers in the locality, he was carrying his pistol also. From the opposite direction, another person was coming with a ferocious looking dog. All of a sudden, the dog which was on a chain held by the owner, started barking at X. The owner of the dog called the dog to be calm. They crossed each other without any problem. But suddenly, the dog started barking again from a distance. X immediately took out his pistol. By seeing the pistol the dog

stopped barking and started walking with the owner. However, X shot at the dog which died instantly. The owner of the dog files a complaint against X, which in due course reached the Magistrate Court. X pleads the right of private defence. Decide.

(a)  Shooting a fierce dog is not to be brought under the criminal law. So the case should be dismissed.

(b)  There was no imminent danger to X as the dog stopped barking and was walking with the owner. Hence, shooting it amounted to excessive use of the right of private defence and hence liable for killing the dog.

(c)  The right of private defence is available to persons against assailants or wrongdoers only and a dog does not fall in this category.

(d)  As there was no guarantee that the dog would not bark again, shooting it was a precautionary measure and hence within the right available to X under law.

Answer: (b)

Direction (Q. Nos. 36 – 50) : Legal phrases are followed by four meanings. Choose the most appropriate option:

146. Perincuriam

(a)  Mistaken identity

(b)  Mistaken decision

(c)  Supremacy of the Constitution

(d)  Supremacy of law

Answer: (b)

147. Autrefois convict

(a)  Formerly convicted

(b)  Failed prosecution

(c)  To be convicted

(d)  Doubtful conviction

Answer: (a)

148. Lex loci

(a)  Domestic laws

(b)  Law of a place

(c)  Latin regualtions

(d)  Italian laws

Answer: (b)

149. Lis pendens

(a)  Pending suit

(b)  Decided case

(c)  No legal issues involved

(d)  Facts of case proved

Answer: (a)

150. Faux pas

(a)  Passage of time

(b)  Tactless mistake

(c)  Pausing for a while

(d)  Cheating

Answer: (b)

151. Bona vacantia

(a)  Order of the court for eviction

(b)  Vacant land

(c)  Goods that have no owner

(d)  Vacant building

Answer: (c)

152. In pari delicto

(a)  Where the lawyer is at fault

(b)  Where the petitioner is at fault

(c)  Where the judge is at fault

(d)  Where both parties to a dispute are equally at fault

Answer: (d)

153. Pari passu

(a)  Diverse nature

(b)  On an unequal status

(c)  Supremacy of law

(d)  On equal footing

Answer: (d)

154. ‘Jus Gentium’ means:

(a)  Law of Societies

(b)  Law among Nations

(c)  Global justice

(d)  Global administrative law

Answer: (b)

155. ‘Punctum Temporis’ means:

(a)  Point of time

(b)  Temporary position

(c)  Timely assistance

(d)  Functional authority

Answer: (a)

156. Turpis arbiter’ means:

(a)  Corrupt prosecutor

(b)  Inefficient judge

(c)  Corrupt judge

(d)  Inefficient lawyer

Answer: (c)

157. Caveat venditor

(a)  Buyer beware

(b)  Seller beware

(c)  Transporter beware

(d)  Manufacturer beware

Answer: (b)

158. ‘Sine die’ means:

(a)  Adjourned without fixing any date for the next meeting.

(b)  Adjourned for the day and scheduled to meet next day again.

(c)  Adjourned for the day and meet after one month.

(d)  Adjourned for the day and meet after one week.

Answer: (a)

159. Animus posssidendi’ means:

(a)  Intention to harm

(b)  Intent to contract

(c)  Intention to possess

(d)  Intention to return

Answer: (c)

160. Malus animus

(a)  Good intention

(b)  Bad intention

(c)  Animal farm

(d)  Physical force

Answer: (b)

Part 5 Logical Reasoning

Direction (Q. Nos. 161 – 163) Read the following information carefully and choose the appropriate option in the questions given below.

I. There is a group of five persons – A, B, C, D and E.

II. One of them is a Singer, one is a Dancer, one is a Painter, one is a Teacher and one is a Doctor.

III. Three of them – A, C and Doctor prefer rice to chapatti and two of them – B and the Painter prefer chapatti to rice.

IV. The Teacher, D and A are friends to one another but two of these prefer chapatti to rice.

V. The Singer is C’s brother.

161. Who is a Dancer?

(a)  D

(b)  C

(c)  A

(d)  E

Answer: (a)

162. Who is a Teacher?

(a)  B

(b)  E

(c)  C

(d)  D

Answer: (c)

163. Who is a Singer?

(a)  C

(b)  D

(c)  A

(d)  B

Answer: (c)

Direction (Q. Nos. 164 – 166) Read the following information carefully and choose the most appropriate option in the questions given below.

I. Six flats on a floor in two rows facing North and South are allotted to P, Q, R, S, T and U.

II. Q gets a North-facing flat and is not next to S.

III. S and U get diagonally opposite flats.

IV. R, next to U gets a South-facing flat and T gets a North-facing flat.

164. If the flats of T and P are interchanged, who’s flat will be next to that of U?

(a)  R

(b)  Q

(c)  P

(d)  T

Answer: (a)

165. Whose flat is between Q and S?

(a)  T

(b)  R

(c)  U

(d)  P

Answer: (a)

166. Which of the following combination gets South-facing flats?

(a)  U, R, P

(b)  Q, T, S

(c)  U, P, T

(d)  data inadequate

Answer: (a)

Direction (Q. Nos. 167-168) Which alternative applies to the following

Statement or Assumptions? Choose the most appropriate option.

167. ‘Only ignorant people believe in witchcraft’ is equivalent to:

(a)  All persons who believe in witchcraft are ignorant.

(b)  No ignorant persons are those who do not believe in witchcraft.

(c)  Some ignorant persons are not those who believe in witchcraft.

(d)  There is no link between ignorance and witchcraft.

Answer: (a)

168. ‘There is no man that is not naturally good’ is equivalent to the proposition:

(a)  Some men are naturally good.

(b)  Some men are not naturally good.

(c)  All men are naturally good.

(d)  No men are good.

Answer: (c)

Direction (Q. Nos. 169-170) : Find the odd one out from the following:

169. Find the odd one out from the following:

(a)  Exact estimate

(b)  Only choice

(c)  Clearly visible

(d)  Open secret

Answer: (c)

170. Find the odd one out from the following:

(a)  Expedition

(b)  Crusade

(c)  Cruise

(d)  Campaign

Answer: (c)

171. How many times from 4 pm to 10 pm, the hands of a clock are at right angles?

(a)  9

(b)  10

(c)  11

(d)  6

Answer: (c)

172. ————– is a hater of knowledge and learning.

(a)  Misologist

(b)  Misogynist

(c)  Mystique

(d)  Moroccan

Answer: (a)

173. Ravi was showing a photograph to his friend, Gopi. Pointing at a boy in the photograph, Ravi said: ‘The boy sitting at the left is the son of the wife of the only son of the grand-mother of my younger brother’.

What is the relation between the boy in the photograph and Ravi?

(a)  First Cousins

(b)  Ravi’s brother-in-law

(c)  Brothers

(d)  Nephew and uncle

Answer: (c)

174. In a company, 60 % workers are males. If the number of female workers in the company is 800, what is the number of male workers in the company?

(a)  1200

(b)  1900

(c)  1400

(d)  1600

Answer: (a)

175. ‘Some of the valuable books are seldom read’, means:

(a)  Some of the valuable books are not read.

(b)  All the valuable books are not read.

(c)  Some of the valuable books are read.

(d)  All the valuable books are read.

Answer: (a)

176. Coding and decoding 9: 72 : : 8 : ?

(a)  34

(b)  64

(c)  18

(d)  43

Answer: (b)

177. A girl introduced a boy as the son of the daughter of the father of her uncle. The boy is girl`s:

(a)  Uncle

(b)  Son

(c)  Brother

(d)  Son-in-law

Answer: (c)

178. Sunil’s school bus is facing North when it reaches his school. After starting from Sunil’s house, it turned right twice and then left before reaching the school. What direction the bus was facing when it left the bus stop in front of Sunil’s house?

(a)  East

(b)  South

(c)  North

(d)  West

Answer: (d)

179. When Ravi saw Ramesh, he recalled, ‘He is the son of the father of my daughter’. Who is Ramesh?

(a)  Uncle

(b)  Brother-in-law

(c)  Cousin

(d)  Brother

Answer: (b)

180. If in a certain code, the word MILITARY is written as 12324567, then in the same code, the word TAIL will be written as:

(a)  2345

(b)  5432

(c)  3254

(d)  4523

Answer: (d)

181. Crumb : Bread is as

(a)  Water : Vessel

(b)  Inch : Unit

(c)  Splinter : Wood

(d)  Powder : Face

Answer: (c)

182. A man walks 1 km. towards East and then he turns to South and walks 5 kms. Again he turns to East and walks 2 kms. After this he turns to North and walks 9 kms. Now, how far is he from his starting point?

(a)  0 kms.

(b)  4 kms.

(c)  9 kms.

(d)  5 kms.

Answer: (d)

183. John wants to go the university. He starts from his house which is in the East and comes to a crossing. The road to his left ends in a theatre, straight ahead is the hospital. In which direction is the University?

(a)  East

(b)  West

(c)  South

(d)  North

Answer: (d)

184. If 27th March, 2011 was Sunday, what was the day on 27th June, 2011?

(a)  Sunday

(b)  Tuesday

(c)  Monday

(d)  Saturday

Answer: (c)

185. Pointing to a girl in the photograph, Ram said, ‘Her mother`s brother is the only son of my mother’s father’. How is the girl`s mother related to Ram?

(a)  Sister

(b)  Mother

(c)  Grandmother

(d)  Aunt

Answer: (d)

186. wave: crest as _________ : peak.

(a)  land

(b)  Water

(c)  River

(d)  Mountain

Answer: (d)

187. Vaishnavi prefers Economics to Maths, English to Social science, and Political Science to History. If she prefers Maths to History, and Social science to Maths, which is Vaishnavi’s least preferred subject?

(a)  History

(b)  Economics

(c)  Maths

(d)  Social science

Answer: (a)

188. Pointing to a photograph, Prakash said, ‘She is the daughter of my grandfather’s only son’

How is Prakash related to the girl in the photograph?

(a)  Brother

(b)  Uncle

(c)  Father

(d)  Cousin

Answer: (a)

189. In a military secret service map, South-East is shown as North, North-East as West and so on. What will West become?

(a)  North-East

(b)  South-West

(c)  North-West

(d)  South-East

Answer: (d)

190. No parrots are black.

All crows are black.

From the above premises which one of the following

(a)  No crows are parrots.

(b)  Some parrots are not crows.

(c)  Some crows are not parrots.

(d)  No conclusion can be drawn.

Answer: (a)

191. If South-East becomes North, North-East becomes West and so on, what will West become?

(a)  North-West

(b)  North

(c)  South-East

(d)  East

Answer: (c)

192. If in a code language, ‘ABANDON’ is written as ‘aramoim’; ‘BORE’ is written as ‘rits’ and ‘BASIL’ is written as ‘rabut’, then what is the original word for the code: ‘bituo’?

(a)  SOMAD

(b)  SOLID

(c)  NASIA

(d)  SOFIA

Answer: (b)

193. Mare is to Horse as –

(a)  Sow is to Boar

(b)  Geese is to Duck

(c)  Pony is to Donkey

(d)  Deer is to Buck

Answer: (a)

194. There were twelve dozens of chocolates with a shopkeeper. Ten chocolates were distributed by the shopkeeper to the children of his colony. The shopkeeper then added two more dozens of chocolates in his stock. If the shopkeeper divided the total chocolates equally in two different packets, then how many chocolates were there in each packet?

(a)  152

(b)  89

(c)  79

(d)  158

Answer: (c)

195. In the series of alphabets given below, which is the missing alphabet series? AX, DU, GR, ____, ML

(a)  JN

(b)  JO

(c)  IK

(d)  HQ

Answer: (b)

196. What is meant by ‘Alliteration’?

(a)  Act of literary modification.

(b)  Acts of an environmentally conscious person.

(c)  The occurrence of the same letter or sound at the beginning of adjacent or closely connected words.

(d)  Words which sound alike but have different meanings.

Answer: (c)

197. Two ladies and two men are playing bridge and seated at North, East, South and West of a table. No lady is facing East. Persons sitting opposite to each other are not of the same sex. One man is facing South. Which direction are the ladies facing to?

(a)  South and East.

(b)  None of these.

(c)  East and West.

(d)  North and West.

Answer: (d)

198. Identify the statement which cannot be accepted

(a)  Odyssey is an ancient epic

(b)  Almost one third of the human body is made up of water

(c)  The earth revolves around the sun in 366 days

(d)  Human race will become extinct sooner or later

Answer: (b)

199. The birthday of Ms. Y was celebrated six days before Ms. X, who was born on 4th October 1999. The independence day of that year fell on Sunday. On which day did Ms. Y celebrate her birthday, if it was not a leap year?

(a)  Sunday

(b)  Monday

(c)  Wednesday

(d)  Tuesday

Answer: (d)

200. A person who renounces religious or political belief or principle is called:

(a)  Ascetic

(b)  apostate

(c)  Antiquarian

(d)  Apostle

Answer: (b)

Career in Law in India

Career in Law in India:

Law is one of the most prestigious and popular career choices in our Country. People whose family generations have been into this profession for years usually choose this career.

However, to choose this career it is not compulsory that the student must be from Law or its related background.

There are different types of branches available in the Law courses. They are civil law, criminal law, Intellectual Property Right, Patent law and corporate law, etc.,

A degree of law not only allows you to practice as a lawyer in courts but also to work in various sectors such as corporate management, legal services and administrative services.

Students who wish to choose this career must check the following points:

  • Is this the right career for me?

Before choosing up this career, please ask this question to yourself. The career in law requires more patience and logical thinking. Think twice whether you are suitable for this profession. Also, the Law profession is a difficult and a challenging one when compared to other professions. It requires more hard work and intelligence to become a successful lawyer. Even for a successful lawyer the starting stage is not that easy. They must be trained under an experienced senior lawyer or advocate to become successful.

Those who are choosing this career must have excellent communication skills, critical thinking and intelligence.

  • Start preparing for Law entrance examination:

        If you have decided to choose this career, then start preparing for the national level Law entrance examinations as soon as you complete the senior secondary education. The national entrance examination like CLAT generally tests the candidate’s ability of thinking, general English, legal aptitude, general awareness and logical skills. The other competitive Law exams are AILET, LSAT, SET, etc.,

  • What type of course can I choose for?

        Any students who decide to choose their career in Law can either do 3 years LLB course or 5 years BA LLB course in this field. 3 years course can be done after any graduation in any discipline whereas 5 years course can be done directly after your 12th standard.

Nowadays, 3 years degree in law is considered less important when compared to the 5 years course in Law. This is because 5 years law course is treated as professional degree in Law and students who really aspire to become an advocate take up this course. 3 years law course is just treated as additional degree nowadays.

  • What is the cost of these courses?

        The cost of pursuing these courses depending on the type and reputation of the institution. A 3 years LLB course usually less expensive where as the five years BA LLB course is little more expensive. There will also be hostel for students opting hostel.

  • What is my salary after I complete the degree?

This is actually one of the highest paying professions but depends upon the Lawyer’s caliber, popularity and success. The lawyer who has just started practicing can get a stipend of Rs.5000 to Rs.40,000 depending upon the performance, caliber, senior advocate he is associated with, etc.,

Outsourcing receives an attractive salary package of Rs.20000 to Rs.200000 per month.

  • International focus:

The Law education in India is similar to that of Britain. Thus, many Universities in Britain offer Law education to many students from India. As a recent trend, students from India study in this these foreign universities by enjoying attractive salary package while studying.

  • Demand and supply:

An acute shortage of well qualified lawyers is a major problem in India. Among ten Lakh lawyers in India only 20 percent of them are said to be fit enough to practice law in courts. Several Law schools like NLSU, NALSAR are established to improve the level of legal education in India and to produce skilled lawyers to meet the requirements.

Also, those who decide to take up this career must sincerely work hard in this profession. Also, they must have an intention to serve their own country through this valuable and respected profession.

  • Job prospects:

There are huge numbers of jobs in Law career. The person taking up Law as a profession can either practice as a Lawyer in the court or can work for the corporate firms. By clearing exams like Public service commission the Law graduate will get a chance to become a Judge.  After good and considerable experience a Law graduate can also become a solicitor general or public prosecutor or work for any Government or public firms.

The other opportunities for a Law graduate is that he/she can also work as a legal adviser to any private/public firms, work with NGO’s or work as a reporter in News paper or television channel.

  • What are the different designations possible in this field?

There are different roles available for a Law graduate to choose for: They are

Civil Litigation Lawyer:

The Lawyer who solely concentrates only one civil cases such as corporate, family, probate, intellectual property tax, personal injury law, real estate and so on.

Criminal Lawyer:

Criminal lawyers who are otherwise called as criminal defense lawyers who are responsible for defending individuals, organizations and entities that have been charged with crime.

They handle diverse types of criminal cases such as domestic violence crime, sex crimes, theft, drug crimes, fraudulent act, etc.,

Legal Analyst:

Legal analyst who works for corporate or law firms and analyses laws related to the company and its operation. He/she deals with all legal problems faced by the concern they are working for.

Document Drafting Lawyer:

The creation of legal documents is an integral part of Law profession. But those who specialize in this field are called as document drafting Lawyers. Such persons are responsible for drafting legal agreements, documents and all related legal papers for the company or the firm they work for.

Legal Journalist:

Legal journalism is a specialization within journalism. Legal journalist research and report every information pertaining to the Law. These people can be very successful when they work as “news reporters” since they will be able to perform well than a journalist without legal knowledge.

 Government Lawyer:

Government Lawyers are the qualified Lawyers or Solicitors who work on behalf of government and give legal advice to the Government ministers and administrative staffs.

Judge:

Judge plays main role in the court room. They interpret law, deal with the evidence presented and offer judgment for all cases.

Different types of Law Exam:

The candidates those who are interested in the Law profession have to qualify any entrance examination below:

CLAT: (Common Law Admission Test)

This is national level entrance test for the admission to any 14 top National Law Universities of India. They are NLUs, TNNLS, Tiruchirappalli, DSNLU Visakhapatnam, Nirma Ahemedabad etc.,

LSAT: (Law School Admission Test)

This is a common test which tests the candidate’s verbal and logical ability skills. This test is conducted by USA based Law School Admission Council (LSAC) for use by Law schools in India.

DU LLB/LLM:

Faculty of Law, University of Delhi conducts a separate entrance exam for the admission to the courses like LLB and LLM.

SET Symbiosis:

Symbiosis International University conducts this exam for the admission to the various UG courses in the Symbiosis University.

ULSAT-UPES Law Studies Aptitude Test:

ULSAT is conducted by University of Petroleum and Energy Studies (UPES). This is a common entrance test conducted for the admission to the courses like BA LLB and LLB in corporate Laws, cyber laws and intellectual property rights.

Top Law schools in India:

  1. Faculty of Law, University of Delhi
  2. Banaras Hindu University, Varanasi
  3. ILS Law college, Pune
  4. Government Law college, Mumbai
  5. Amity Law School, Delhi
  6. Aligarh Muslim University, Aligarh
  7. National Law University, Jodhpur
  8. Gujarat National Law of University (GNLU)
  9. NALSAR University of Law, Hyderabad
  10. Symbiosis Society’s Law College (SSLC), Pune

Candidates who do 5 years BA LLB from top law schools like NALSAR or NLSUI Bangalore get huge job opportunity from the ITES firms and other law firms. These graduates can also start to practice in courts. But those who wish to practice in the court have to register their name in the bar council of India (BCI) and practice under the senior lawyers.

CLAT UG Seat Matrix 2014 Assam

 

NLUJA Assam
Category of Seats No. of Seats
Grand Total 60
All India Category Seats
All India Total 45
General 30
S.C. 04
S.T./ ST (Plains) ST (Hills) 06

03

P.W.D (Horizontal) 02
P.W.D (Vertical)
O.B.C
State Category Seats Based on Domicile
State Total 15
General 15
SC
ST
P.W.D(Horizontal)
Dependants of Freedom Fighters (DFF)
O.B.C
B.C
E.B.C
W.B.C
Ancestral residents of village Sidhuwal(Punjab)
Son/Daughters of Defence Personnel Deployed in U.P. or
Defence Personnel of U.P. either retired or killed disabled in action
Special Category Seats
NRI/NRI Sponsored
Foreign Nationals



 

CLAT UG Seat Matrix 2014 Ranchi

 

NUSRL Ranchi
Category of Seats No. of Seats
Grand Total 100
All India Category Seats
All India Total 50
General 24
S.C. 08
S.T./ ST (Plains) ST (Hills) 04
P.W.D (Horizontal)
P.W.D (Vertical)
O.B.C 14
State Category Seats Based on Domicile
State Total 50
General 25
SC 05
ST 13
P.W.D(Horizontal)
Dependants of Freedom Fighters (DFF)
O.B.C
B.C 03
E.B.C 04
W.B.C
Ancestral residents of village Sidhuwal(Punjab)
Son/Daughters of Defence Personnel Deployed in U.P. or Defence
Personnel of U.P. either retired or killed disabled in action
Special Category Seats
NRI/NRI Sponsored
Foreign Nationals



 

CLAT UG Seat Matrix 2014 Odisha

 

NLUO Odisha
Category of Seats No. of Seats
Grand Total 120
All India Category Seats
All India Total 106
General 78
S.C. 10
S.T./ ST (Plains) ST (Hills) 14
P.W.D (Horizontal) 04
P.W.D (Vertical)
O.B.C
State Category Seats Based on Domicile
State Total …..
General
SC
ST
P.W.D(Horizontal)
Dependants of Freedom Fighters (DFF)
O.B.C
B.C
E.B.C
W.B.C
Ancestral residents of village Sidhuwal(Punjab)
Son/Daughters of Defence Personnel Deployed in U.P. or
Defence Personnel of U.P. either retired or killed disabled in action
Special Category Seats
NRI/NRI Sponsored 10
Foreign Nationals 04



 

CLAT UG Seat Matrix 2014 Patna

 

CNLU Patna
Category of Seats No. of Seats
Grand Total 140
All India Category Seats
All India Total 60
General 60
S.C.
S.T./ ST (Plains) ST (Hills)
P.W.D (Horizontal) 3%#
P.W.D (Vertical)
O.B.C
State Category Seats Based on Domicile
State Total 60
General
SC 19
ST 01
P.W.D(Horizontal)
Dependants of Freedom Fighters (DFF)
O.B.C
B.C 14
E.B.C 22
W.B.C 04
Ancestral residents of village Sidhuwal(Punjab)
Son/Daughters of Defence Personnel Deployed in U.P. or
Defence Personnel of U.P. either retired or killed disabled in action
Special Category Seats
NRI/NRI Sponsored 10
Foreign Nationals 10



 

CLAT UG Seat Matrix 2014 Kochi

 

NUALS Kochi
Category of Seats No. of Seats
Grand Total 120
All India Category Seats
All India Total 77
General 61
S.C. 10
S.T./ ST (Plains) ST (Hills) 02
P.W.D (Horizontal) 04
P.W.D (Vertical)
O.B.C
State Category Seats Based on Domicile
State Total 43
General 12
SC
ST
P.W.D(Horizontal)
Dependants of Freedom Fighters (DFF)
O.B.C 31
B.C
E.B.C
W.B.C
Ancestral residents of village Sidhuwal(Punjab)
Son/Daughters of Defence Personnel Deployed in U.P. or Defence
Personnel of U.P. either retired or killed disabled in action
Special Category Seats
NRI/NRI Sponsored
Foreign Nationals



 

CLAT UG Seat Matrix 2014 Raipur

 

HNLU Raipur
Category of Seats No. of Seats
Grand Total 180
All India Category Seats
All India Total 80
General 60*
S.C. 12*
S.T./ ST (Plains) ST (Hills) 06*
P.W.D (Horizontal) 02@
P.W.D (Vertical)
O.B.C
State Category Seats Based on Domicile
State Total 80
General 31*
SC 10*
ST 26*
P.W.D(Horizontal) 02@
Dependants of Freedom Fighters(DFF)
O.B.C 11*
B.C
E.B.C
W.B.C
Ancestral residents of village Sidhuwal(Punjab)
Son/Daughters of Defence Personnel Deployed in U.P. or Defence
Personnel of U.P. either retired or killed disabled in action
Special Category Seats
NRI/NRI Sponsored 16
Foreign Nationals 04



 

CLAT UG Seat Matrix 2014 Patiala

 

RGNLU Patiala
Category of Seats No. of Seats
Grand Total 120
All India Category Seats
All India Total 102
General 71
S.C. 18
S.T./ ST (Plains) ST (Hills) 09
P.W.D (Horizontal) 04
P.W.D (Vertical)
O.B.C
State Category Seats Based on Domicile
State Total 13
General 12
SC
ST
P.W.D(Horizontal)
Dependants of Freedom Fighters (DFF)
O.B.C
B.C
E.B.C
W.B.C
Ancestral residents of village Sidhuwal(Punjab) 01
Son/Daughters of Defence Personnel Deployed in U.P. or Defence
Personnel of U.P. either retired or killed disabled in action
Special Category Seats
NRI/NRI Sponsored
Foreign Nationals 05



 

CLAT UG Seat Matrix 2014 Lucknow

 

RMLNLU Lucknow
Category of Seats No. of Seats
Grand Total 160
All India Category Seats
All India Total 80
General 80**
S.C.
S.T./ ST (Plains) ST (Hills)
P.W.D (Horizontal) 3%#
P.W.D (Vertical)
O.B.C
State Category Seats Based on Domicile
State Total 80
General
SC 34**
ST 03**
P.W.D(Horizontal)
Dependants of Freedom Fighters (DFF) 2%##
O.B.C 43**
B.C
E.B.C
W.B.C
Ancestral residents of village Sidhuwal(Punjab)
Son/Daughters of Defence Personnel Deployed in U.P. or Defence
Personnel of U.P. either retired or killed disabled in action
5%##
Special Category Seats
NRI/NRI Sponsored
Foreign Nationals



 

CLAT UG Seat Matrix 2014 Gandhinagar

 

GNLU Gandhinagar
Category of Seats No. of Seats
Grand Total 180
All India Category Seats
All India Total 113
General 68*
S.C. 26*
S.T./ ST (Plains) ST (Hills) 13*
P.W.D (Horizontal) 06*
P.W.D (Vertical)
O.B.C
State Category Seats Based on Domicile
State Total 44
General 44*
SC
ST
P.W.D(Horizontal)
Dependants of Freedom Fighters (DFF)
O.B.C
B.C
E.B.C
W.B.C
Ancestral residents of village Sidhuwal(Punjab)
Son/Daughters of Defence Personnel Deployed in U.P. or Defence
Personnel of U.P. either retired or killed disabled in action
Special Category Seats
NRI/NRI Sponsored 17
Foreign Nationals 06



 

CLAT UG Seat Matrix 2014 Jodhpur

 

NLU Jodhpur
Category of Seats No. of Seats
Grand Total 100
All India Category Seats
All India Total 100
General 77
S.C. 15
S.T./ ST (Plains) ST (Hills) 08
P.W.D (Horizontal) 3%#
P.W.D (Vertical)
O.B.C
State Category Seats Based on Domicile
State Total …..
General
SC
ST
P.W.D(Horizontal)
Dependants of Freedom Fighters(DFF)
O.B.C
B.C
E.B.C
W.B.C
Ancestral residents of village Sidhuwal(Punjab)
Son/Daughters of Defence Personnel Deployed in U.P. or
Defence Personnel of U.P. either retired or killed disabled in action
Special Category Seats
NRI/NRI Sponsored
Foreign Nationals



 

CLAT UG Seat Matrix 2014 Kolkata

 

WBNUJS Kolkata
Category of Seats No. of Seats
Grand Total 125
All India Category Seats
All India Total 103
General 78
S.C. 15
S.T./ ST (Plains) ST (Hills) 08
P.W.D (Horizontal) 02
P.W.D (Vertical)
O.B.C
State Category Seats Based on Domicile
State Total …..
General
SC
ST
P.W.D(Horizontal)
Dependants of Freedom Fighters(DFF)
O.B.C
B.C
E.B.C
W.B.C
Ancestral residents of village Sidhuwal(Punjab)
Son/Daughters of Defence Personnel Deployed in U.P. or
Defence Personnel of U.P. either retired or killed disabled in action
Special Category Seats
NRI/NRI Sponsored 18
Foreign Nationals 04



 

CLAT UG Seat Matrix 2014 Hyderabad

 

NALSAR Hyderabad
Category of Seats No. of Seats
Grand Total 80
All India Category Seats
All India Total 56
General 41*
S.C. 09*
S.T./ ST (Plains) ST (Hills) 04*
P.W.D (Horizontal)
P.W.D (Vertical) 02
O.B.C
State Category Seats Based on Domicile
State Total 14
General 11*
SC 02*
ST 01*
P.W.D(Horizontal)
Dependants of Freedom Fighters(DFF)
O.B.C
B.C
E.B.C
W.B.C
Ancestral residents of village Sidhuwal(Punjab)
Son/Daughters of Defence Personnel Deployed in U.P. or
Defence Personnel of U.P. either retired or killed disabled in action
Special Category Seats
NRI/NRI Sponsored
Foreign Nationals 10



 

CLAT UG Seat Matrix 2014 Bhopal

 

NLIU Bhopal
Category of Seats No. of Seats
Grand Total 120
All India Category Seats
All India Total 51
General 39*
S.C. 08*
S.T./ ST (Plains) ST (Hills) 04*
P.W.D (Horizontal) 3%#
P.W.D (Vertical)
O.B.C
State Category Seats Based on Domicile
State Total 51
General 25*
SC 08*
ST 11*
P.W.D(Horizontal) 3%#
Dependants of Freedom Fighters(DFF)
O.B.C 07*
B.C
E.B.C
W.B.C
Ancestral residents of village Sidhuwal(Punjab)
Son/Daughters of Defence Personnel Deployed in U.P. or
Defence Personnel of U.P. either retired or killed disabled in action
Special Category Seats
NRI/NRI Sponsored 18
Foreign Nationals



 

CLAT UG Seat Matrix 2014 Bangalore

 

NLSIU Bangalore
Category of Seats No. of Seats
Grand Total 80
All India Category Seats
All India Total 75
General 55
S.C. 12
S.T./ ST (Plains) ST (Hills) 06
P.W.D (Horizontal) 02
P.W.D (Vertical)
O.B.C
State Category Seats Based on Domicile
State Total ……
General
SC
ST
P.W.D(Horizontal)
Dependants of Freedom Fighters(DFF)
O.B.C
B.C
E.B.C
W.B.C
Ancestral residents of village Sidhuwal(Punjab)
Son/Daughters of Defence Personnel Deployed in U.P. or Defence
Personnel of U.P. either retired or killed disabled in action
Special Category Seats
NRI/NRI Sponsored
Foreign Nationals 05



 

CLAT – UG 2014 Seat Matrix

 

NLUJA   Assam
Category of Seats No. of Seats
Grand Total 60
All India Category Seats
All India Total 45
General 30
S.C. 04
S.T./ ST (Plains) ST (Hills) 0603
P.W.D (Horizontal) 02
P.W.D (Vertical)
O.B.C
State Category Seats Based on Domicile
State Total 15
General 15
SC
ST
P.W.D(Horizontal)
Dependants of Freedom Fighters (DFF)
O.B.C
B.C
E.B.C
W.B.C
Ancestral residents of village Sidhuwal(Punjab)
Son/Daughters of Defence Personnel Deployed in U.P. or Defence
Personnel of U.P. either retired or killed disabled in action
Special Category Seats
NRI/NRI Sponsored
Foreign Nationals
NUSRL   Ranchi
Category of Seats No. of Seats
Grand Total 100
All India Category Seats
All India Total 50
General 24
S.C. 08
S.T./ ST (Plains) ST (Hills) 04
P.W.D (Horizontal)
P.W.D (Vertical)
O.B.C 14
State Category Seats Based on Domicile
State Total 50
General 25
SC 05
ST 13
P.W.D(Horizontal)
Dependants of Freedom Fighters (DFF)
O.B.C
B.C 03
E.B.C 04
W.B.C
Ancestral residents of village Sidhuwal(Punjab)
Son/Daughters of Defence Personnel Deployed in U.P. or
Defence Personnel of U.P. either retired or killed disabled in action
Special Category Seats
NRI/NRI Sponsored
Foreign Nationals
NLUO   Odisha
Category of Seats No. of Seats
Grand Total 120
All India Category Seats
All India Total 106
General 78
S.C. 10
S.T./ ST (Plains) ST (Hills) 14
P.W.D (Horizontal) 04
P.W.D (Vertical)
O.B.C
State Category Seats Based on Domicile
State Total …..
General
SC
ST
P.W.D(Horizontal)
Dependants of Freedom Fighters (DFF)
O.B.C
B.C
E.B.C
W.B.C
Ancestral residents of village Sidhuwal(Punjab)
Son/Daughters of Defence Personnel Deployed in U.P. or
Defence Personnel of U.P. either retired or killed disabled in action
Special Category Seats
NRI/NRI Sponsored 10
Foreign Nationals 04
CNLU   Patna
Category of Seats No. of Seats
Grand Total 140
All India Category Seats
All India Total 60
General 60
S.C.
S.T./ ST (Plains) ST (Hills)
P.W.D (Horizontal) 3%#
P.W.D (Vertical)
O.B.C
State Category Seats Based on Domicile
State Total 60
General
SC 19
ST 01
P.W.D(Horizontal)
Dependants of Freedom Fighters (DFF)
O.B.C
B.C 14
E.B.C 22
W.B.C 04
Ancestral residents of village Sidhuwal(Punjab)
Son/Daughters of Defence Personnel Deployed in U.P. or
Defence Personnel of U.P. either retired or killed disabled in action
Special Category Seats
NRI/NRI Sponsored 10
Foreign Nationals 10
NUALS   Kochi
Category of Seats No. of Seats
Grand Total 120
All India Category Seats
All India Total 77
General 61
S.C. 10
S.T./ ST (Plains) ST (Hills) 02
P.W.D (Horizontal) 04
P.W.D (Vertical)
O.B.C
State Category Seats Based on Domicile
State Total 43
General 12
SC
ST
P.W.D(Horizontal)
Dependants of Freedom Fighters (DFF)
O.B.C 31
B.C
E.B.C
W.B.C
Ancestral residents of village Sidhuwal(Punjab)
Son/Daughters of Defence Personnel Deployed in U.P. or
Defence Personnel of U.P. either retired or killed disabled in action
Special Category Seats
NRI/NRI Sponsored
Foreign Nationals
HNLU   Raipur
Category of Seats No. of Seats
Grand Total 180
All India Category Seats
All India Total 80
General 60*
S.C. 12*
S.T./ ST (Plains) ST (Hills) 06*
P.W.D (Horizontal) 02@
P.W.D (Vertical)
O.B.C
State Category Seats Based on Domicile
State Total 80
General 31*
SC 10*
ST 26*
P.W.D(Horizontal) 02@
Dependants of Freedom Fighters(DFF)
O.B.C 11*
B.C
E.B.C
W.B.C
Ancestral residents of village Sidhuwal(Punjab)
Son/Daughters of Defence Personnel Deployed in U.P. or
Defence Personnel of U.P. either retired or killed disabled in action
Special Category Seats
NRI/NRI Sponsored 16
Foreign Nationals 04
RGNLU   Patiala
Category of Seats No. of Seats
Grand Total 120
All India Category Seats
All India Total 102
General 71
S.C. 18
S.T./ ST (Plains) ST (Hills) 09
P.W.D (Horizontal) 04
P.W.D (Vertical)
O.B.C
State Category Seats Based on Domicile
State Total 13
General 12
SC
ST
P.W.D(Horizontal)
Dependants of Freedom Fighters (DFF)
O.B.C
B.C
E.B.C
W.B.C
Ancestral residents of village Sidhuwal(Punjab) 01
Son/Daughters of Defence Personnel Deployed in U.P. or
Defence Personnel of U.P. either retired or killed disabled in action
Special Category Seats
NRI/NRI Sponsored
Foreign Nationals 05
RMLNLU    Lucknow
Category of Seats No. of Seats
Grand Total 160
All India Category Seats
All India Total 80
General 80**
S.C.
S.T./ ST (Plains) ST (Hills)
P.W.D (Horizontal) 3%#
P.W.D (Vertical)
O.B.C
State Category Seats Based on Domicile
State Total 80
General
SC 34**
ST 03**
P.W.D(Horizontal)
Dependants of Freedom Fighters (DFF) 2%##
O.B.C 43**
B.C
E.B.C
W.B.C
Ancestral residents of village Sidhuwal(Punjab)
Son/Daughters of Defence Personnel Deployed in U.P. or
Defence Personnel of U.P. either retired or killed disabled in action
5%##
Special Category Seats
NRI/NRI Sponsored
Foreign Nationals
GNLU   Gandhinagar
Category of Seats No. of Seats
Grand Total 180
All India Category Seats
All India Total 113
General 68*
S.C. 26*
S.T./ ST (Plains) ST (Hills) 13*
P.W.D (Horizontal) 06*
P.W.D (Vertical)
O.B.C
State Category Seats Based on Domicile
State Total 44
General 44*
SC
ST
P.W.D(Horizontal)
Dependants of Freedom Fighters (DFF)
O.B.C
B.C
E.B.C
W.B.C
Ancestral residents of village Sidhuwal(Punjab)
Son/Daughters of Defence Personnel Deployed in U.P. or
Defence Personnel of U.P. either retired or killed disabled in action
Special Category Seats
NRI/NRI Sponsored 17
Foreign Nationals 06
NLU   Jodhpur
Category of Seats No. of Seats
Grand Total 100
All India Category Seats
All India Total 100
General 77
S.C. 15
S.T./ ST (Plains) ST (Hills) 08
P.W.D (Horizontal) 3%#
P.W.D (Vertical)
O.B.C
State Category Seats Based on Domicile
State Total …..
General
SC
ST
P.W.D(Horizontal)
Dependants of Freedom Fighters(DFF)
O.B.C
B.C
E.B.C
W.B.C
Ancestral residents of village Sidhuwal(Punjab)
Son/Daughters of Defence Personnel Deployed in U.P. or
Defence Personnel of U.P. either retired or killed disabled in action
Special Category Seats
NRI/NRI Sponsored
Foreign Nationals
WBNUJS   Kolkata
Category of Seats No. of Seats
Grand Total 125
All India Category Seats
All India Total 103
General 78
S.C. 15
S.T./ ST (Plains) ST (Hills) 08
P.W.D (Horizontal) 02
P.W.D (Vertical)
O.B.C
State Category Seats Based on Domicile
State Total …..
General
SC
ST
P.W.D(Horizontal)
Dependants of Freedom Fighters(DFF)
O.B.C
B.C
E.B.C
W.B.C
Ancestral residents of village Sidhuwal(Punjab)
Son/Daughters of Defence Personnel Deployed in U.P. or
Defence Personnel of U.P. either retired or killed disabled in action
Special Category Seats
NRI/NRI Sponsored 18
Foreign Nationals 04
NALSAR   Hyderabad
Category of Seats No. of Seats
Grand Total 80
All India Category Seats
All India Total 56
General 41*
S.C. 09*
S.T./ ST (Plains) ST (Hills) 04*
P.W.D (Horizontal)
P.W.D (Vertical) 02
O.B.C
State Category Seats Based on Domicile
State Total 14
General 11*
SC 02*
ST 01*
P.W.D(Horizontal)
Dependants of Freedom Fighters(DFF)
O.B.C
B.C
E.B.C
W.B.C
Ancestral residents of village Sidhuwal(Punjab)
Son/Daughters of Defence Personnel Deployed in U.P. or
Defence Personnel of U.P. either retired or killed disabled in action
Special Category Seats
NRI/NRI Sponsored
Foreign Nationals 10
NLIU   Bhopal
Category of Seats No. of
Seats
Grand Total 120
All India Category Seats
All India Total 51
General 39*
S.C. 08*
S.T./ ST (Plains) ST (Hills) 04*
P.W.D (Horizontal) 3%#
P.W.D (Vertical)
O.B.C
State Category Seats Based on Domicile
State Total 51
General 25*
SC 08*
ST 11*
P.W.D(Horizontal) 3%#
Dependants of Freedom Fighters(DFF)
O.B.C 07*
B.C
E.B.C
W.B.C
Ancestral residents of village Sidhuwal(Punjab)
Son/Daughters of Defence Personnel Deployed in U.P. or
Defence Personnel of U.P. either retired or killed disabled in action
Special Category Seats
NRI/NRI Sponsored 18
Foreign Nationals
NLSIU   Bangalore
Category of Seats No. of Seats
Grand Total 80
All India Category Seats
All India Total 75
General 55
S.C. 12
S.T./ ST (Plains) ST (Hills) 06
P.W.D (Horizontal) 02
P.W.D (Vertical)
O.B.C
State Category Seats Based on Domicile
State Total ……
General
SC
ST
P.W.D(Horizontal)
Dependants of Freedom Fighters(DFF)
O.B.C
B.C
E.B.C
W.B.C
Ancestral residents of village Sidhuwal(Punjab)
Son/Daughters of Defence Personnel Deployed in U.P. or
Defence Personnel of U.P. either retired or killed disabled in action
Special Category Seats
NRI/NRI Sponsored
Foreign Nationals 05



 

© Copyright Entrance India - Engineering and Medical Entrance Exams in India | Website Maintained by Firewall Firm - IT Monteur